Anda di halaman 1dari 99

Hence;

��������������
Volume of KMnO4
Topic 1 Atoms, Molecules &
Stoichiometry ​ 2 ​  × 2.0 × 10–3
__
= ​ ____________
5   
 ​  × 1000
0.020
PAPER 1 = 40 cm . 3
Section A
1. B 6. B
From the relative abundance given in option B, 4Na + 3CO2 → 2Na2CO3 + C
(92.2 × 28) + (4.7 × 29) + (3.1 × 30)
______________________________ ________
1.1 × 107
Ar of Si = ​  No. of mole of CO2 = ​  22.4 ​   mol
92.2 +   
4.7 +   
3.1 ​ Mole of Na CO
= 28.109 ______________
2 3 __
2
​       ​= ​ 3 ​ 
Mole of CO2
2. C ________
1.1 × 107
∴ Mole of Na2CO3 = __ ​ 2 ​  × ​  22.4 ​  
Reaction: Fe + 2Fe3+ → 3Fe2+ 3
In order for the resultant solution to contain equal = 3.274 × 105 mol
numbers of moles of Fe3+(aq) and Fe2+(aq), 3 mol Mr of Na2CO3 = (2)(23) + 12 + (3)(16) = 106
of excess Fe3+(aq) is required. ∴ Mass of Na2CO3 = Mole × Mr
Reaction involves 1 mol of Fe and 5 mol of = (3.274 × 105) × 106
Fe3+(aq), and the resultant solution contains 3 mol = 3.5 × 107 g
of Fe2+(aq) and 3 mol of unreacted Fe3+(aq).�
Common Mistakes
3. B Molar volume of gas,Vm =22.4 dm3 mol–1 at s.t.p. and (not
At r.t.p., 24 dm3 of gas  1 mol of gas 24 dm3 mol–1 under room conditions)
_______
1
1 cm3 of gas  ​     ​ 
mol of gas
24 × 103 7. C
Avogadro's constant states that 1 mole of gas ______
48.0
contains 6.02 × 1023 molecules. No. of moles of CO2= ​ 24000  ​ 
= 0.00200 mol
∴ No. of O2 molecules = No. of moles of O2 × No. of moles of (Na2CO3)x•y(H2O2)
Avogadro's constant _____
10.0
= ​ 1000  ​ × 0.100 = 0.00100 mol
_______
1
= ​     ​ 
× 6.02 × 1023
24 × 103 Evolution of CO2 is due to:
_____________
1 × 6.02 × 1023 Na2CO3 → Na2O + CO2
= ​  24 000    ​ 
Since 2 moles of CO2 is released from 1 mole of
4. A
(Na2CO3)x•y(H2O2), ∴ x = 2
Since density of 'atmospheric N2' is higher than _____
24.0
that of chemically pure N2, the gas that causes No. of moles of KMnO4 = ​ 1000   ​ × 0.0500
this discrepancy would have high Ar/Mr greater = 0.00120 mol
than the Ar of N2. Since KMnO4 reacts with H2O2 in mole ratio 2:5,
Ar of N2 = 28 Mole of H2O2
______________ __
5
Ar of Ar = 39.9 ∴ ​    
   ​= ​   ​ 
Mole of KMnO4 2
Ar of He = 4 __
5
Ar of CH4 = 16 Mole of H2O2 = ​ 2 ​  × Mole of KMnO4
Ar of Ne = 20.2 __
5
= ​ 2 ​  × 0.00120
5. B = 0.00300 mol
KHC2O4·H2C2O4 → 2C2O42– + 3H+ + K+
1mol 2 mol Since 0.00100 mol of (Na2CO3)x•y(H2O2) contains
0.00300 mol of H2O2, ∴ y = 3
∴ 1.0 × 10–3 mol of KHC2O4·H2C2O4 will __y __ 3
dissociate to give 2.0×10–3 mol of C2O42–. Hence, ratio ​ x ​= ​ 2 ​ 
2MnO4–(aq) + 5C2O42–(aq) + 16H+(aq) → 8. C
2Mn2+(aq) + 10CO2(g) + 8H2O(l) Reaction: Fe + 2Fe3+ → 3Fe2+
In order for the resultant solution to contain
5 mol of C2O42– will react with 2 mol of MnO4–.
equal numbers of moles of Fe3+(aq) and Fe2+(aq),
∴ 2.0 × 10–3 mol of C2O42– will react with 3 mol of excess Fe3+(aq) is required.
__
2 Reaction involves 1 mol of Fe and 5 mol of
​ 5 ​  × 2.0 × 10–3 mol of MnO4–.

Answers to A Level H2 Topical Chemistry A


© Singapore Asia Publishers Pte Ltd Topic 1

H2 Topical Chemistry 2014.indb 1 3/27/2014 1:57:12 PM


Fe3+(aq), and the resultant solution contains ∴ Mass of HOC6H4CO2H
3 mol of Fe2+(aq) and 3 mol of unreacted = Mole × Mr
Fe3+(aq). = 0.07407 × 138
= 10.2 g
Section B
9. A Mole of (CH3CO)2O
____
54.5 ___ 9.1 = Mole of CH3CO2C6H4CO2H
Mole ratio of C : H = ​ 12.0 ​ : ​ 1.0  ​
= 4.54 : 9.1 = 0.07407 mol
= 1: 2 ∴ Mass of (CH3CO)2O
= Mole × Mr
Option Molecular Formula C:H mole ratio
= 0.07407 × 102
1 C4H8O2 1:2
= 7.56 g
2 C4H8O2 1:2
3 C4H8S 1:2 (b)
reflux condenser
10. D water out
1 – Correct statement.
2 – It should be the ratio of the average mass
___
1 water in
of a molecule to ​ 12  ​ of mass of an atom of retort
12
C. stand
3 – Relative molecular mass should be compared round-bottom flask
with the mass of a 12C atom, and not 1H stirrer bar
atom. hot plate
magnetic stirrer
PAPER 2 speed heat
1. Planning (P)
(a) (i) HOC6H4CO2H + (CH3CO)2O → Steps:
1. Weigh out accurately 10.2 g of
CH3CO2C6H4CO2H + CH3COOH
2-hydroxybenzene carboxylic acid and
Tips add into a round bottom flask.
The question hinted that the reaction between 2. Add 7 cm3 of ethanoic anhydride and
2-hydroxybenzene carboxylic acid and ethanoic anhydride 8 – 10 drops of 85% phosphoric acid into
gives aspirin and ethanoic acid.
the round bottom flask
3. Shake the reaction mixture well and cool
(ii) % yield of Pure Aspirin occasionally in an ice water bath.
Actual yield
______________ 4. Set up a reflux system as shown above
= ​       ​× 100%
Theoretical yield and heat the reaction mixture under reflux
____
100 for around fifteen minutes.
∴ Theoretical yield = ​  75 ​ × Actual yield
5. Add 2 – 3 cm3 of water to the hot reaction
____
100 mixture to hydrolyse any unreacted
= ​  75 ​ × 10 g
ethanoic anhydride.
= 13.33 g
6. Reaction is allowed to subside and
No. of moles of aspirin reaction mixture is poured into about
Mass of aspirin
_____________ 50 cm3 of cold water to precipitate aspirin.
= ​    
    ​
Mr of aspirin 7. Crude aspirin product is then purified by
13.33 g
_______ recrystallisation from water.
= ​  180 ​  
(a) Dissolve aspirin in small amount of
= 0.07407 mol water.
Based on the balanced chemical (b) Heat the solution until all aspirin just
equation in (a)(i); dissolves, i.e. saturated solution.
Mole of HOC6H4CO2H (c) Allow the solution to cool, and pure
= Mole of CH3CO2C6H4CO2H aspirin will crystallise as temperature
= 0.07407 mol drops.

Answers to A Level H2 Topical Chemistry A


© Singapore Asia Publishers Pte Ltd Topic 1

H2 Topical Chemistry 2014.indb 2 3/27/2014 1:57:13 PM


(d) Pure aspirin crystals are obtained by ∴ Mole of Na2O
filtration. __
1
= ​ 2 ​  × Mole of NaOH
Tips __
1
= ​ 2 ​  × (5 × 10–3 mol)
_______
Mass 7.56 g
_________
Volume of ethanoic anhydride = ​    ​ =
 ​    ​ 

Density 1.08 g/cm3 = 2.5 × 10–3 mol
=7g
Mole of I2
____________ __
1
We can check the purity of the sample by 2. ������
(a) ​       ​= ​   
Mole of S2O32– 2
measuring its melting point.
(c) Handling of corrosive contents. __
1
 ​ ∴ Mole of I2 = ​ 2 ​  × Mole of S2O32–
Wear safety gloves and goggles.
__
1 _____
15.0
= ​ 2 ​  × (0.100 × ​ 1000  ​ 
)
PAPER 3 = 7.50 × 10–4 mol
1. �����������������
(a) (i) NaOH + HCl → NaCl + H2O
Mole of NaOH in 25.0 cm3 (b) Mole of O3 = Mole of I2
= Mole of HCl = 7.50 × 10–4 mol
_____
22.5 At s.t.p, the molar gas volume, Vm = 22.4 dm3
= (0.100)(​ 1000  ​ 
)
∴ Vol. of O3 = (7.50 × 10–4) × (22.4 × 1000)
= 2.25 × 10 mol
–3
= 16.8 cm3
Mole of NaOH in 100 cm3 Hence, % of O3 in O2/O3 mixture
____
100
= ​ 25.0  ​ × (2.25 × 10–3 mol) ____
16.8
= ​ 500 ​ × 100%
= 9 × 10–3 mol
= 3.36%
(ii) 2Mn​O​–4​​ + 5H2O2 + 6H+
→ 2Mn2+ + 5O2 + 8H2O
Mole of H2O2 in 25.0 cm3
__
5
= ​ 2 ​  × Mole of MnO4–
__
5 _____
10.0
= ​ 2 ​  × (0.0200)(​ 1000  ​ 
)
= 5.00 × 10 mol –4

∴ Mole of H2O2 in 100 cm3


____
100
= ​ 25.0  ​ × (5.00 × 10–4 mol)
= 2 × 10–3 mol
(iii) Na2O2 + 2H2O → 2NaOH + H2O2
Mole of Na2O2 = Mole of H2O2
= 2 × 10–3 mol
Mole of NaOH (from Na2O2)
= 2 × Mole of H2O2
= 2 × (2 × 10–3 mol)
= 4 × 10–3 mol
∴ Mole of NaOH (from Na2O)
= Total no. of moles of NaOH
– Mole of NaOH (from Na2O2)
= 9 × 10–3 – 4 × 10–3
= 5 × 10–3 mol
Na2O + H2O → 2NaOH

Answers to A Level H2 Topical Chemistry A


© Singapore Asia Publishers Pte Ltd Topic 1

H2 Topical Chemistry 2014.indb 3 3/27/2014 1:57:13 PM


Topic 2 Atomic Structurec Structure ∴ To form Al2+, ∆H = 577 + 1820
= 2397 kJ mol–1
PAPER 1 757 1640
Section A Co(g) Co+(g) Co2+(g)
1. D ∴ To form Co2+, ∆H = 757 + 1640
no. of p no. of n no. of e– = 2397 kJ mol–1
 
36
​16 ​S
  2– 16 20 18 8. D
 
37 –
   l
​ ​C 17 20 18 A – Cu atom has 29 electrons while Mg atom
17
has 12 electrons
2. C ∴ Cu atom has more electrons than Mg
1st 2nd 3rd 4th 5th 6th
I.E. 950 1800 2700 4800 6000 12300 atom
B – E.C. of Cu2+: 1s22s22p63s23p63d9
∆ 850 900 2100 1200 6300
outer E.C.
The big jump from the 5th I.E. to the 6th I.E.
E.C. of Mg2+: 1s22s22p6
shows that the 6th electron is removed from the
outer E.C.
next inner electron shell.
C – E.C. of Cu2+: 1s22s22p63s23p63d9
Hence, element X is a Group V element with 5
valence electrons and can form compounds with E.C. of Mg: 1s22s22p63s2
formula XCl3. ∴ Both particles have 3 electron shells which
are occupied.
3. D
D – E.C. of Cu: 1s22s22p63s23p63d104s1
Cu2+ has configuration of [Ar] 3d9 and thus has
E.C. of Mg2+: 1s22s22p6
a single unpaired electron.
∴ Cu has 4 electron shells occupied whereas
4. C Mg2+ has only 2 electron shells that are
The element in Option C contains 3 valence occupied.
electrons (3d14s2) and would lose all 3 valence
9. B
electrons most easily to form X3+ ion which has
From the Data Booklet:
a stable electronic configuration. A – 1st I.E. is 799, 1090, 1400 and 1310 kJ mol–1
5. D respectively
When a neutron changes to a proton, the nucleon  does not follow the trend in the bar
number does not change. The proton number chart
increases by one. B – 1st I.E. is 757, 736, 745 and 908 kJ mol–1
respectively
Tips  follows the trend in the bar chart
Nucleon No.(Mass No.) = No. of Proton + No. of Neutron C – 1st I.E. is 736, 590, 548 and 502 kJ mol–1
respectively
 
40
​ ​K
    ​C
​40  does not follow the trend in the bar
19 20   a
• 19 protons • 20 protons chart
• Nucleon No. = 40 • Nucleon No. = 40 D – 1st I.E. is 786, 1060, 1000 and 1260 kJ mol–1
respectively
6. C  does not follow the trend in the bar

A half-filled set of p orbitals has np 3 chart
configuration.
10. B
A: C– (1s22s22p3) → C (1s22s22p2)
No. of protons of Po = 84
B: N (1s22s22p3) → N+ (1s22s22p2)
C: N– (1s22s22p4) → N (1s22s22p3) No. of nucleons of Po = 210
D: O+ (1s22s22p3) → O2+ (1s22s22p2) ∴ No. of neutrons of Po = 210 – 84 = 126
When an -particle is emitted;
7. A
No. of protons of X = 84 – 2 = 82
To form one mole of an ion with a 2+ charge, the
1st and 2nd I·E. must be involved. No. of neutrons of X = 126 – 2 = 124
577 1820  
210
​ ​P   o → ​4 ​H  
   e + 206
​ ​X  
84 2 82
Al(g) Al+(g) Al2+(g)

Answers to A Level H2 Topical Chemistry A


© Singapore Asia Publishers Pte Ltd Topic 2

H2 Topical Chemistry 2014.indb 4 3/27/2014 1:57:13 PM


11. D
Topic 3 Chemical Bonding
Neon and fluorine are in the same period of the
Periodic Table. PAPER 1
Ionisation energy increases across a period due to Section A
increasing nuclear charge and decreasing atomic 1. B CH3
radius. O=C
δ– δ+ CH
Since the electrons all go into the same shell, 3

the shielding of the ionising electrons is about Molecule B has the largest dipole due to presence
the same. The outer electrons are, therefore, of a very electronegative O atom.
increasingly more strongly attracted by the
positive nucleus, and so, more energy is required Common Mistakes
to remove an electron. Molecule C has a smaller dipole than
Cl
Molecule B because the C=O and C–Cl
O=C
bonds have very similar dipoles and tends
Common Mistakes Cl
to cancel each other out�.
C – The atomic radius of fluorine is less than that of
neon.
2. B
In ice, each water molecule is hydrogen bonded
12. D
to four other water molecules.
Electronic 2 lone pairs (or 4 electrons) from each oxygen
Particle Remarks
Configurations atom are involved in forming hydrogen bonds.
Cr [Ar]3d54s1 Unpaired s electron
Ge [Ar]3d104s24p2 Unpaired p electron Tips
A – All the angles surrounding oxygen atoms are 109.5º
S –
ls 2s 2p 3s 3p
2 2 6 2 5
Unpaired p electron
(tetrahedral shape around oxygen atom).
Sc [Ar]3d14s2 Unpaired d electron C – The hydrogen bonds (intermolecular bonding)
are weaker than covalent bonds (intramolecular
13. B bonding).
The big jump from the 2nd I.E. to the 3rd I.E. D – The open structure causes ice to be less dense than
shows that the 3rd electron is removed from the water i.e. ice floats on water.
next inner electron shell.
Hence, element X is a Group II element with 2 3. C
valence electrons. N–1 and N–3 exists as N– ion and form ionic
bonds with Mg2+ ion.
PAPER 3 N–2 and N–4 form co-ordinate (dative covalent)
1. (a) Nucleon number refers to the total number bonds with Mg2+ ion.
–N– N–
of neutrons and protons in the nucleus of an
atom. Mg2+
(b)  Electrons which are negatively charged –N N––
are deflected towards the positive
terminal, while protons being positively 4. D
charged are deflected towards the negative Copper has a metallic structure and its giant lattice
terminal. consists of positive metal ions surrounded by 'sea
 Electrons (being lighter than protons) are of delocalised valence electrons'.
deflected more than protons. Iodine has a simple/discrete molecular structure
and its lattice will consist of small covalent I2
molecules held together by weak intermolecular
(van der Waals) forces.
5. C
Delocalised electrons are electrons in a molecule
that are not associated with a particular atom but
extend over several adjacent atoms.
All four ions contain C=C double bond(s) and
so, have delocalised � electrons.

Answers to A Level H2 Topical Chemistry A


© Singapore Asia Publishers Pte Ltd Topic 2/Topic 3

H2 Topical Chemistry 2014.indb 5 3/27/2014 1:57:14 PM


O– H O O O– D – Carbon 1 and 4 are sp3 hybridised which has
O=C H O=C O←N tetrahedral geometry.
O– H O– O–
carbonate ethanoate nitrate phenoxide 13. C
Both H2O and HF exist as small covalent
6. B molecules and their boiling points are affected
Dry hydrogen chloride does not ionise in by the intermolecular forces between its
methylbenzene. Hence, HCl exists as covalent molecules.
molecules in the resultant solution and does not Both H2O and HF are able to form hydrogen
show any acidic properties since no H+ ions are bonding between its molecules and contribute
dissociated. significantly to the intermolecular forces.
A – pH less than 7 due to presence of H+ ions. Such H2O molecule is hydrogen-bonded to
B – HCl does not dissociate into ions, thus no four other H2O molecules, in a tetrahedral
mobile ions to conduct electricity. arrangement:
C – No H+ ions to have any acidic properties. δ+ H H δ+
D – No H+ ions to have any acidic properties. O δ–

7. B H δ+
Each nitrogen atom in di-imine has one lone pair δ+ O δ–
and two bond pairs. δ– O
H Hδ+ δ– H δ+
H δ+ O
It has a bent shape with bond angle of 118º since δ+ H
bond pair – lone pair repulsion is greater than δ– O H
bond pair – bond pair repulsion. δ+ H δ+
Each HF molecule is hydrogen-bonded to two
N=N other HF molecules:
H 118º H δ– δ–
F δ+ F δ+
H H
8. D F
H δ+ F
H δ+
F
H δ+
Weak van der Waals’ forces between the δ– δ– δ–
microscopic hairs and the non-polar surface of Therefore, on average, there are more hydrogen
glass allows interaction to be formed and broken bonds between H2O molecules than there are
easily. between HF molecules. Hence, more energy
9. B is required to overcome these hydrogen bonds
between H2O molecules, as compared to HF
No. of electron pairs = 4
No. of bond pairs = 3 H P H molecules.
107º
No. of lone pairs = 1 H 14. C
Molecular Shape = Trigonal Pyramidal bond formed by
Bond  = 107º σ bond formed by 2p– 2p overlap
10. C 1s – 2sp overlap
���������������������������������������������������
The lattice structure of MgO is similar to that of H H H
σ σ σ
NaCl. Each Mg2+ ion is surrounded by 6 other H σ
C σ C σ
C σ
C C σ
H
σ
O2– ions, and every O2– ion is surrounded by 6 σ

other Mg2+ ions. H σbond formed by


2sp2 – 2sp3 overlap
11. B
C1 and C2 are sp hybridised while C3 is sp3 Section B
hybridised. Hence, the sigma bond between C2 15. C
and C3 is a sp–sp3 overlap. In graphite, each carbon uses 3 valence electrons
12. C for covalent bonding. The fourth valence electron
A – Carbon 2 and 3 are sp hybridised. is used for � bonding between adjacent carbon
B – The alcohol will be oxidised to a carboxylic atoms, resulting in delocalisation.
acid. KMnO4 will decolorised from purple In the giant metallic structure of sodium,
colour, as it is being reduced to Mn2+ ions. positive sodium ions are surrounded by a 'sea
of delocalised valence electrons' throughout the
C – Carbon 1 and 4 are sp3 hybridised.
giant lattice.

Answers to A Level H2 Topical Chemistry A


© Singapore Asia Publishers Pte Ltd Topic 3

H2 Topical Chemistry 2014.indb 6 3/27/2014 1:57:14 PM


PAPER 2
Tips 1. ����
(a)
In cyclohexene, all the valence electrons are used for
Cl
covalent bonding between the atoms. O C
Cl
16. B
(b) Cl–C–Cl bond angle is 120ºC.
CO2, COS and COSe are all linear molecules.
Electronegativity of Group VI elements decreases Tips
down the group. Molecular shape is Trigonal Planar
Electronegativity : O > S > Se
Bond Polarity : C=O > C=S > C=Se 2. (a) Co-ordination number refers to the maximum
17. D number of neighbouring ions that can be
1 – Diamond has tetrahedral shape and C–C–C placed around another ion of the opposite
bond angle is 109.5º, whereas graphite has charge in a crystal lattice.
(b) The ionic radii of Cs+ > K+ > Na+.
trigonal planar shape with C–C–C bond
The larger ionic radii of Cs+ ion allows
angle of 120º.
more Cl– ions to be packed around it. Thus,
2 – The C–C bond length in diamond is
the co-ordination number in CsCl lattice is
0.154 nm and in graphite is 0.142 nm.
larger than those in NaCl and KCl.
3 – Diamond has a giant covalent structure and
all C–C covalent bonds are of the same 3. ���
[PCl4]+
Cl +
strength.
Graphite has a giant structure of hexagonal P
rings layer over another layer held by weak Cl Cl
van der Waals' forces. The C–C covalent Cl
bonds within the layer are of the same Shape: Tetrahedral
strength. [PCl6]–
18. B Cl –
1 & 2 – Dative covalent bonds are formed Cl Cl
P
between Mg and two of the N atoms Cl
Cl
(shown by line). The other two Cl

N atoms form sigma bonds to the Mg
Shape: Octahedral
atom (shown by line ).
3 – Since Mg atom is in a planar arrangement PAPER 3
with the four N atoms, it cannot be sp3 1. ����
(a) Example of Hydrogen Bonding:
hybridised since sp3 hybridisation will give Hydrogen bonding between H 2 O
molecule a tetrahedral arrangement/shape. molecules
19. A
Hydrogen δ+
Graphite has a giant molecular structure and is O δ– Bonding
δ– H
made up of flat layers of carbon atoms arranged
hexagonally. H H O
δ+ δ+
Within the layer, each carbon atom is bonded H
covalently to three other carbon atoms, using δ+
three of the valence electrons. The fourth valence Requirements for Hydrogen Bonding are:
electron is delocalised throughout the layer.  An electron-deficient hydrogen atom
The valency of each carbon atom in the giant bonded to a very electronegative atom
lattice is 3, since the definition of ‘�����������
������������
valency����
’���
is (such as N, O or F) in one molecule.
the number of valence bonds a given atom has  A lone pair of electrons on a very
formed, or can form, with one or more other electronegative atom (such as N, O or
atoms. F) in the other molecule.
(b) The intermolecular forces in carboxylic

Answers to A Level H2 Topical Chemistry A


© Singapore Asia Publishers Pte Ltd Topic 3

H2 Topical Chemistry 2014.indb 7 3/27/2014 1:57:14 PM


acids includes both van der Waals' forces F
(between the alkyl chains) and hydrogen
bonding (between the carboxyl groups) while O O
that between water molecules are mainly The above structure of FO2 does not exist
hydrogen bonding. because F is a Period 2 element and can
The first four members of the series of only accomodate a maximum of 8 electrons
carboxylic acids are fully soluble in water (octet).
because they form hydrogen bonds with F
water molecules.
O O
As the alkyl chain (of the carboxylic acids)
gets longer, the acids become increasingly The above structure also does not exist
insoluble because there is stronger van der because F cannot form 2 dative bonds to
Waals' forces between the acid molecules. oxygen atoms since F is too electronegative
As such, the energy released from the for dative bonding.
formation of hydrogen bonds between
3. For ClF5:
the carboxylic acids and water molecules F
No. of electron pairs = 6
is insufficient to compensate the energy F F
No. of bond pairs = 5 Cl
required to break the stronger van der Waals' F F
forces and hydrogen bonding between the No. of lone pairs = 1
acid molecules, as well as hydrogen bonding Molecular Geometry = Octahedral
between water molecules. Based on VSEPR Theory;
Hence, the solubility of carboxylic acids LP–LP repulsion > LP–BP repulsion > BP–BP
in water decreases as the chain length repulsion
increases. ∴ Molecular Shape = Square Pyramidal
2. ����
(a) For ICl4–: –
O N O O O O No. of electron pairs = 6 Cl Cl
NO2 O3 No. of bond pairs = 4 I
Cl Cl
No. of lone pairs = 2
(b) Bond angle of NO2 : 118º
Molecular Geometry = Octahedral
Bond angle of O3 : 116º
Both central atoms are surrounded by 3 Based on VSEPR Theory;
electron pairs. LP–LP repulsion > LP–BP repulsion > BP–BP
∴ Basic angle is about 120º. repulsion
Since both has 1 'lone pair' and 2 bond pairs, ∴ Molecular Shape = Square Planar
Bond angle < 120º.
The single unpaired electron on NO 2
molecule will be less effective in causing
lone pair – bond pair repulsion, and so the
bond angle for NO2 should be greater than
that for O3.
(c) Cl

O O
In the structure above, there are 9 electrons
around the central chlorine atom. ClO2
exists because Cl is a Period 3 element
and has available low-lying 3d orbitals to
expand its octet to accomodate more than
8 electrons.

Answers to A Level H2 Topical Chemistry A


© Singapore Asia Publishers Pte Ltd Topic 3

H2 Topical Chemistry 2014.indb 8 3/27/2014 1:57:15 PM


PAPER 2
Topic 4 The Gaseous State
PV
___
1. (a) n = ​ RT  ​
PAPER 1
Section A (6 × 100 × 103) × (670 × 10–6)
_________________________
1. D = ​        
 ​
8.31× (20 + 273)
Using the Ideal Gas Equation: = 0.165 mol
PV = nRT whereby: P in Pa
V in m3 Tips
T in K • Pressure must be converted into Pa (or Nm–2).
• Volume must be converted into m3.
R = 8.31 J K–1 mol–1
____
nRT
∴ P = ​  V ​ 
  nRT
____
(b) P = ​  V   
​ 
(1.6 × 10–3)(8.31)(273 + 273)
________________________
= ​       ​
   0.165 × 8.31 × (5 + 273)
____________________
(3.0 × 10–3) = ​    
 ​   
670 × 10–6
= 569 000 Pa
Common Mistakes
= 5.69 bar
• Units of T must be in K.
• Units of V must be in m3.
(c) Maximum safe pressure difference between
internal and external pressure = 800 – 101
2. A = 699 kPa
At 10,000 m, the pressure difference across
The stronger the intermolecular forces of
the wall of the tyre in flight
attraction between the molecules, the greater the
= (5.69 × 102 – 0.28 × 102) kPa
deviation from ideality.
= 541 kPa
Methane, ethane and nitrogen are non-polar Therefore, it is not necessary to reduce the
molecules which are held together by the weaker air pressure inside the tyre since difference
induced dipole – induced dipole attractions. between the internal and external pressure
Ammonia molecules are held together by the (541 kPa) at 10,000 m is well within the safe
stronger hydrogen bonding. pressure range (699 kPa) that the tyre can
withstand before it burst.
3. D
For Ideal gas, PV = nRT PAPER 3
__
P ___ nR
∴ ​ T  ​= ​ V ​ = constant; since same mass & same volume 1. �����������������
For an ideal gas:
P1 __ P2  Gas particles have negligible volume (or
__
​ T   ​ = ​ T   ​ size)
1 2
 Gas particles have negligible intermolecular
T1
__
T2 = P2 × ​ P  ​  forces of attraction
1  Collisions of gas particles are perfectly
(27 + 273)
_________ elastic.
= 2p × ​  p ​   
2. �����������������������������
(a) Total mass of gas mixture
= 600 K = 1.00 – 0.438
= 0.562 g
Common Mistakes For an Ideal Gas; PV = nRT
Temperature must be in Kelvin (K) when using the Ideal ___
m
PV = ​ M   ​ RT
Gas Equation. If it is used as ºC, you will end up with a r
different answer. _____
mRT
_____
27ºC
∴ Mr = ​  PV ​ 
i.e. T2 = 2p × ​  p ​ 
 
(0.562)(8.31)(298)
___________________
= 54ºC (which is equivalent to 327K) = ​       ​
(1.00 ×105)(386 × 10–6)
T/K = T/ºC + 273
= 36.055
= 36.1 (3 s.f.)

Answers to A Level H2 Topical Chemistry A


© Singapore Asia Publishers Pte Ltd Topic 4

H2 Topical Chemistry 2014.indb 9 3/27/2014 1:57:15 PM


(b) Carbon dioxide ∴ y = mole fraction of AlCl3 in mixture
= 0.390
Tips
From ③: x = 1 – y
CO2(g) is an acidic oxide and will react with the alkaline
∴ y = mole fraction of Al2Cl6 in mixture
solution, NaOH (aq).
= 1 – 0.390
= 0.610
(c) Mr of both gases
(f) Partial pressure of Al2Cl6
Mr of CO2 + Mr of other gas
_______________________ = Mole fraction of Al2Cl6  Total Pressure
= ​  2 ​
      
= (0.610)(1.16  10­5 Pa)
∴ Mr of other gas = 70,760 Pa
= (2 × Mr of both gas) – Mr of CO2 Partial Pressure of AlCl3
= (2)(36.1) – 44 = Mole fraction of AlCl3  Total Pressure
= 28.2 = (0.390)(1.16  10­5 Pa)
 28.0 = 45,240 Pa
Carbon Monoxide. (PAlCl )2
_______
(g) Kp = ​   ​ 
Pa
3

(d) E is CaC2O4. (PAl Cl )


2 6

CaC2O4 → CaO + CO2 + CO (45,240)2


________
= ​   ​ 
Pa
(70,760)
3. ����
(a) Cl Cl Cl = 28,923.93 Pa
Dot-and-cross
≈ 28,900 Pa
Al Al diagram of
Al2Cl6
Cl Cl Cl
(b) Trigonal planar
(c) Consider the equilibrium,
Al2Cl6(g)  2AlCl3(g)  ∆H = +63 kJ mol–1.
If temperature is increased, the system will
try to decrease it by absorbing the extra
heat energy. This will favour the above
endothermic reaction and forward reaction
to form AlCl3(g) will be favoured.
(d) Using the general gas equation for an ideal
gas,
pV = nRT
___
m
pV = ​ Mr  ​ RT
_____
mRT
∴ Mr = ​  pV ​ 
(1.50g)(8.31JK–1 mol–1)(500K)
_________________________
= ​     
    ​
(1.16  105Pa)(250  10–6 m3)
= 214.91
≈ 215
Tips
1000 cm3 = 10–3 m3

x + y = 1
(e) Using: ①
215 = 267x + 133.5y ②

From ①:
x = 1 – y ③
③ into ②: 215 = 267(1 – y) + 1335.5
215 = 267 – 267y + 133.5y
133.5y = 52

Answers to A Level H2 Topical Chemistry A10


© Singapore Asia Publishers Pte Ltd Topic 4

H2 Topical Chemistry 2014.indb 10 3/27/2014 1:57:16 PM


Since ∆G = ∆H – T∆S
Topic 5 Chemical Energetics
___
∆H ____________
–6000 J mol–1
∴ ∆S = ​  T ​ = ​     ​  = –22.0 J K–1 mol–1
PAPER 1 (0 + 273) K
Section A _____
Mass ____54 g
No. of moles of H2O = ​  M  ​   = ​ 18.0 ​ = 3 mol
1. A r

A: Combustion reaction is always exothermic Hence, the entropy change


i.e. ∆Hc is always negative. = –22.0 J K–1 mol–1 × 3 mol
B: Enthalpy change of solution can be either = –66 J K–1
exothermic or endothermic. Common Mistakes
C: Forming an ion from an atom is normally Students tend to make the mistake of having
endothermic. ∆H = +6.0 kJ mol–1 which will result in the answer being
Option D.
D: Enthalpy change of formation may be Note also that the unit for Temp.(T) must be K.
exothermic or endothermic, depending on T/ºC + 273 = T/K
the energetic stability of product with respect
to the reactants. 6. A
Enthalpy change of atomisation is always
2. C
endothermic since energy is absorbed to pull the
∆H is +ve since dissolving of NH4NO3 is
atoms far apart and to break all the bonds between
endothermic which is why it is used as instant them, in order to convert them into free gaseous
'coldpacks'. ions.
∆G is –ve since the reaction is spontaneous. Enthalpy change of combustion is always
∆S is +ve since entropy increases when a highly exothermic.
ordered ionic solid dissolves in water i.e. system Enthalpy change of formation can be endothermic
becomes more random. or exothermic.
3. C Enthalpy change of solution can be endothermic
A – If calcium ethanedioate has a higher or exothermic.
solubility product, Ksp, it will be more soluble 7. A
than magnesium ethanedioate. ∆H = –44 kJ mol–1 when steam condenses
B – Calcium ethanedioate has a lower lattice ∆G = 0 since equilibrium exists
energy than magnesium ethanedioate since ∆G = ∆H – T∆S
Ca2+ ion is larger than Mg2+ ion. ___
∆H __________
–44
∴ ∆S = ​  T ​ = ​     ​ 
× 1000
C – Ca2+ is larger than Mg2+ ion. Its hydration (273 + 100)
enthalpy is lower ( i.e. less exothermic), This = –118J K–1 mol–1
cause the enthalpy of solution to become less _____
Mass
No. of moles of steam condensed = ​  M  ​  
exothermic and hence, solubility is lower. r
54 g
____
= ​ 18.0 ​ 
∆Hsoln = ∆Hhyd – ∆Hlatt
= 3 mol
D – Electropositivity does not affect solubility. ∴ Entropy change = 3 × (–118 J K–1 mol–1)
4. B = –354 J K–1 mol–1
In the Reverse Osmosis process, pure water is Common Mistakes
separated from the impurities (other molecules Must consider that 3 moles (54 g) of steam are involved.
and ions) and thus, there is an increase in the
orderliness i.e. ∆S<0. 8. C
Since ∆G = ∆H – T∆S and that ∆H = 0, ∆G will ∆​Hθr​​ ​= y kJ mol–1
be +ve. 2C2H4(g) C4H8(g)

5. A +6O2 +6O2
Given: ∆H = –6.0 kJ mol–1
= –6,000 J mol–1 2∆​HθC​ ​(​  C2H4) ? ∆​H​θC ​(​  C4H8) = x kJ mol–1
∆G = 0 since equilibrium exists at the
4CO2(g) + 4H2O(l)
freezing point

Answers to A Level H2 Topical Chemistry A11


© Singapore Asia Publishers Pte Ltd Topic 5

H2 Topical Chemistry 2014.indb 11 3/27/2014 1:57:16 PM


By Hess’ Law: Section B
∆​Hθr​​ ​= 2∆​Hθc​​ ​(C2H4) – ∆​Hθc​​ ​(C4H8) 12. D
Ca(s) + 2H2O(l) → Ca(OH)2(s) + H2(g)
2∆​Hθc​​ ​(C2H4) = ∆​Hθr​​ ​+ ∆​Hθc​​ ​(C4H8)
∆Hr = ​∑∆H​θf​ ​(products) – ​∑∆H​θf​ ​(reactants)
∆​Hθr​​ ​+ ∆​Hθc​​ ​(C4H8)
_______________
∴ ∆​Hθc​​ ​(C2H4) = ​  = [∆​H​θf​ ​for Ca(OH)2(s) + ∆​H​θf​ ​for H2(g)]
2 ​     
y+x
_____ – [​∆H​θf​ ​for Ca(s) + 2​∆H​θf​ ​for H2O(l)]
= ​  2 ​   
Since ∆
​ H​θf​ ​for Ca(s) and H2(g) is zero;
(x + y)
______
= ​  2 ​   kJ mol–1 ∴ ∆​Hθf​​ ​for Ca(OH)2(s) = ∆​Hr + 2∆​H​θf​ ​for H2O(l)
9. C Thus, information needed is ∆​Hθf​​ ​for H2O(l).
q •q
_______
Lattice Energy  ​  + –   ​  13. D
(r+ + r–)
At 298K (25ºC), carbon exists in the solid state
The bigger the ionic charge, the bigger (i.e. more
and water exists in the liquid state.
exothermic) is the lattice energy.
∴ Option 2 and 3 are eliminated.
The smaller the ionic radius, the bigger (i.e. more
exothermic) is the lattice energy. 14. A
Lattice energy of Mz = 13.3 ∆S will be positive when the system becomes
Lattice energy of Jx = 3.57 less orderly (i.e. Sfinal > Sinitial).
Lattice energy of Ly = 2.78 1 – Entropy increases (∆S > 0) when a solute
Mz > Jx > Ly dissolves in a solvent. The resulting solution
Most Least
Exothermic Exothermic is less ordered than the original salt crystal
because the particles (solute) are scattered
10. D throughout the whole solution, mixed with
ΔGθ = ΔHθ – TΔSθ the solvent molecules.
= (–2090 J mol–1) – (12 + 273)(44.1 – 51.4) 2 – Entropy increases (∆S > 0) when solvent
= –2090 – 285(–7.3) is evaporated from the solution. There is a
change of state of solvent molecules from
Common Mistakes
liquid to gaseous, resulting in a less ordered
Temperature must be in Kelvin, K.
T/K = T/ºC + 273 system.
Unit for ΔHθ should be converted to J mol–1 (from the 3 – Entropy increases (∆S > 0) during osmosis
given kJ mol–1) since the passage of solvent through a semi-
permeable membrane cause the solute to be
11. A
more dispersed in its solution, resulting in a
Forward reaction: CO2(g) + nH2O(l ) → CO2(aq)
less ordered system.
∆H is –ve since strong intermolecular hydrogen
bonds are formed between the CO2 and H2O
molecules. PAPER 2
The entropy, S, of a gas is very big since gas 1. ��������
(a) 2CHCl3(l) + O2(g) → 2COCl2(g) + 2HCl(g)
molecules have free movements and are not ∆Hθ = –356 kJ mol–1
constrained to be close to each other. When CO2 ∆Hr = ∑∆Hf (products) – ∑∆Hf (reactants)
gas dissolves in water, its molecules associate
with water molecules through intermolecular –356 = [2∆Hf(COCl2) + 2∆Hf(HCl)] – [2∆Hf(CHCl3)
hydrogen bonding, and this results in an increased + ∆Hf(O2)]
degree of order for the system. Hence, we would –356 = [2∆Hf(COCl2) + 2(–92)] – [2(–134) + 0]
expect a decrease in the entropy i.e. ∆S is –ve.
–356 = 2∆Hf(COCl2) + 84
∴ ∆Hf(COCl2) = –220 kJ mol–1

(b) This oxidation requires UV light and free


radicals are involved.

Answers to A Level H2 Topical Chemistry A12


© Singapore Asia Publishers Pte Ltd Topic 5

H2 Topical Chemistry 2014.indb 12 3/27/2014 1:57:17 PM


2. ������������
(a) (i) ∆​Gθppt
​  ​ ​ = 2.303RT logKsp q •q
_______
(ii) L.E.  ​  + –  ​ 
= 2.303(8.31)(298) log(2.0 × 10–10) (r+ + r–)
= –55 310 J mol–1 From data booklet, ionic radii/nm:
= –55.3 kJ mol–1 Mg2+ = 0.065, Na+ = 0.095
(ii) Given: ∆H​θ​ppt  ​ ​= –66.0 kJ mol–1 Higher charge of Mg2+ ion (over Na+
ion) and a smaller ionic radii (Mg2+ <
∆​G​θppt  ​​ = ∆​Hθppt
​  ​​ – T∆​Sθppt
​  ​​ 
Na+ and O2– < halide ion) results in a
∆​Hθppt
​  ​​ – ∆​Gθppt
___________ ​  ​​  much stronger electrostatic forces of
∴ ∆​Sθppt
​  ​ ​ = ​  T ​  
 
attraction between Mg2+ and O2– ions
–66.0 – (–55.3)
_____________ than that between Na+ and halide
= ​  298 ​    
ions. Hence, lattice energy of MgO is
= –35.9 J mol–1 K–1 considerably larger than those of the
(iii) Negative ∆​S​θppt  ​​ suggests that there is a sodium halides.
decrease in entropy due to the formation (c) I– ion (ionic radii = 0.216 nm) is much larger
of more ordered lattices as precipitate than F– ion (ionic radii = 0.136 nm) and can
forms. be easily polarised by the Ag+ ion. There is
(b) ∆​Gθppt
​  ​ =
​  2.303RT logKsp some covalent character in the ionic bond
= 2.303(8.31)(298) log(1.006) of AgI.
= +14.8 J mol–1
4. ��������������������������������������������
(a) (i) Lattice energy of Group II sulfates
Positive ∆​Gθ​ppt  ​​ suggests that precipitation will
decreases from MgSO4 to BaSO4.
not occur. This means that AgF is soluble in
water at 298K. q •q
_______
(ii) Lattice Energy  ​  + –   ​ 
(r+ + r–)
3. (a) The lattice energy of magnesium oxide is As Group II cation increases in size from
the heat energy evolved when one mole of Mg2+ to Ba2+, it makes the lattice energy
MgO(s) ionic compound is formed from to decrease from MgSO4 to BaSO4.
its gaseous Mg2+(g) & O2–(g) ions, under (b) (i)
standard conditions.
δ+ H δ– δ+
Mg2+(g) + O2–(g) → MgO(s) ∆H = L.E. O Mg2+ δ– H SO42–
δ+ H O ion-dipole
Tips ion-dipole H interactions
interactions
δ+
Remember to mention Mg2+(g) and O2–(g) gaseous ions to
form the MgO(s) since the question ask you to reference
Mg2+ cation SO42- anion
to magnesium oxide.

q •q (ii) As the Group II cation size increases


_______
(b) (i) L.E.  ​  + –  ​  from Mg2+ to Ba2+, the strength of ion-
(r+ + r–)
dipole interaction with water molecules
From data booklet, ionic radii/nm:
decreases. As such, less heat energy is
Cl– = 0.181, Br– = 0.195, I– = 0.216
released i.e. ∆Hhydration of M2+ becomes
Charge of cations and anions remain
less negative (decreases) from Mg2+ to
the same for all three NaCl, NaBr and
Ba2+.
NaI.
With increase in the ionic radii from 5. (a) (i) Hydrocarbons
Cl– to Br– to I–; the lattice energies (ii) In the internal combustion engine,
decreases from NaCl to NaBr to NaI. where the temperature is very high,
This is due to weaker electrostatic forces nitrogen combines with oxygen in the
of attraction between Na+ ions and the air to form nitrogen monoxide, NO.
halide ions. (iii) Amino acids

Answers to A Level H2 Topical Chemistry A13


© Singapore Asia Publishers Pte Ltd Topic 5

H2 Topical Chemistry 2014.indb 13 3/27/2014 1:57:17 PM


(c)
Tips
Sulfur is a component of all living cells. The amino acids CH3CH = CH2 + CO + H2 → CH3CH2CH2CHO
cysteine, methionine, homocysteine and taurine contain Energy absorbed during bond breaking (of reactants):
sulfur as do some common enzymes. It is a component
1 × C=C bond +610
of most proteins.
6 × C–H bonds +6(410)
Total energy absorbed
1 × C=O bond +1077
(b) (i) ∆​Hθr​​ ​ = Σ∆​Hθf​​ ​(products) – Σ∆​Hθf​​ ​(reactants) = +4,933 kJ mol–1
1 × H–H bond +436
= (–396) – (–297 + 0)
1 × C–C bond +350
= –99 kJ mol–1
(ii) ∆G = ∆Hθ – T∆Sθ
θ
Energy released during bond forming (of product):
= (–99  103 J mol–1) –
8 × C–H bonds –8(410)
(298K)(–93.5 JK–1 mol–1) Total energy released
1 × C=O bond –740
= –71,137 J mol–1
3 × C–C bonds –3(350) = –5,070 kJ mol
–1

(c) (i) ∆Gθ = –2.303 RT logKC


________
–∆Gθ ∴ ∆H = HBond–Breaking + HBond–Forming
​ 2.303RT   ​  
= logKC
= (+4,933) + (–5,070)
–(–71,137)
_______________
​        ​ = logKC = –137 kJ mol–1
2.303(8.31)(2.98)
∴ KC = 1012.47 2. �������
(a) (i)
= 2.95  1012 (3 s.f.) E
(ii) The magnitude of KC gives a measure
of the position of equilibrium. For this Ca2+(g) + 2F(g) + 2e–
case, the KC is very large (i.e. KC > 102),
2nd I.E.(Ca)
which means that the reaction is
Ca+(g) + 2F(g) + e– electron
almost complete (i.e. position of affinity of F
equilibrium is to the right) and the 1st I.E.(Ca) 2(–328)
composition of equilirbium system is Ca(g) + 2F(g)
mostly made up of products. B.E. (F–F) Ca2+(g) + 2F–(g)
(iii) SO3 has a much higher activation energy Ca(g) + F2(g) = +158
than SO2, when sulfur is burned with
Ca(s) + F2(g) ∆Hatm(Ca) = +178
oxygen. 0 L.E.
(iv) A catalyst is used to speed up the rate ∆Hf(CaF2) = –1220
of a reaction by lowering the activation CaF2(s)
energy, providing an alternative
By Hess' Law,
pathway for reaction to proceed, i.e. the
L.E. = ∆Hf(CaF2) – [∆Hatom(Ca) + B.E.(F–F) +
catalyst ensures that the reaction is fast
1st I.E.(Ca) + 2nd I.E.(Ca) + 2(–328)]
enough for a dynamic equilibrium to be
= –1220 – [178 + 158 + 590 + 1150 + 2(–328)]
set up within the very short time that the
= –2640 kJ mol–1
gases are actually in the reactor.
q .q
_______
(ii) Using L.E.  ​  + –  ​ 
(r+ + r–)
PAPER 3 • Magnitude of lattice energy of CaCl2
1. (a) ∆S for reaction I will be negative since there is smaller than that of CaF2 due to
is a decrease in the number of moles of the larger ionic size of Cl– while the
gaseous particles, making the system more charge remains the same.
orderly. • Magnitude of lattice energy of CaO
will be larger than that of CaF2
(b) ∆G = ∆H – T∆S
because the electrostatic forces of
∆H = ∆G + T∆S
attraction between Ca2+ and O2– ions
Since ∆G and ∆S are negative values,
is stronger than that between Ca2+ and
∴ ∆H has a negative value. F– ions.

Answers to A Level H2 Topical Chemistry A14


© Singapore Asia Publishers Pte Ltd Topic 5

H2 Topical Chemistry 2014.indb 14 3/27/2014 1:57:17 PM


(b) (i) Ksp = [Ca2+][F–]2 mol3 dm–9 Mr of C2H5OH = 46.0
CaF2(s)  Ca2+(aq) + 2F–(aq) No. of mole of C2H5OH
2.3 × 10–4 2.3 × 10–4 2(2.3 × 10–4) _____
Mass ____ 1.50
∴ Ksp = (2.3 × 10–4)(2 × 2.3 × 10–4)2 = ​  M  ​ 
 = ​ 46.0 ​ 
r
= 4.87 × 10–11 mol3 dm–9 = 0.0326 mol
(ii)
∆Hsoln __________
–25080 J
∴ ∆Hc = ​   
 ​ 
CaF2(s) + aq Ca2+(aq) + 2F–(aq) 0.0326 mol
∆Hf(Ca2+(aq)) = –769 kJ mol–1
∆Hf(CaF2(s)) 2∆Hf(F–(aq)) (ii) ∆Hc value calculated in (a)(i) is much
Ca(s) + F2(g) less than the true value of ∆Hc because
By Hess' Law, heat is lost to the surroundings and to
∆Hsoln = –543 + 2(–333) – (–1220) the copper can. Not all the heat from
= +11 kJ mol–1 combusting ethanol are used to heat up
(iii) CaF2 is more soluble in hot water than the water in the can.
in cold water. (b) (i)
∆Hsoln is positive (endothermic) and thus H H H O
the dissolution of CaF2 is favoured by __
1
H – C – C – O – H + ​ 2 ​  O = O H–C–C–H+O
high temperature.
H H H H H
3. ���������������������������������������������
(a) 2 factors affecting magnitude of lattice Bonds broken Energy Absorbed Bonds Formed Energy Evolved
energy are: 2 C–H 2(410) 1 C=O –740
1 Ionic Charge – Larger the ionic charge, 1 C–O 360 1 C–H –410
larger (more exothermic) is the lattice 1 O–H 460 2 O–H 2(–460)
energy __
1 __
1
​   ​  O=O ​   ​ (496)
2 2
2 Ionic Radius – Smaller the ionic radius, +1888 kJ –2070 kJ

larger (more exothermic) is the lattice
∴ ∆H = HB–B + HB–F
energy
= (+1888) + (–2070)
Tips = –182 kJ mol–1
__
5 ∆HC
q •q (ii) CH3CHO + ​ 2 ​ O2 + H2O 2CO2 + 3H2O
L.E.  _____
​ r + + r– ​   whereby: q+ = ionic charge of cation
+ –
q– = ionic charge of anion –182 –1367
r+ = ionic radius of cation CH3CH2OH + 3O2
r– = ionic radius of anion By Hess' Law,
∆HC = –(–182) + (–1367)
q •q
______
(b) Lattice Energy  ​ r + + r – ​  = –1185 kJ mol–1
+ –
As the size of the cation M2+, increases down Tips
Group II, the lattice energy of the oxides of Alternative Method:
Group II elements would decrease down the C2H5OH + 3O2 → 2CO2 + 3H2O –– ∆H1 = –1367 kJ mol–1
__
1
Group. C2H5OH + ​ 2 ​ O2 → CH3CHO + H2O –– ∆H2 = –182 kJ mol–1
__
5
q •q CH3CHO + ​ 2  ​O2 → 2CO2 + 2H2O –– ∆H3 = ?
______
(c) Lattice Energy  ​ r + + r – ​  Since Eq. = Eq. – Eq.;
+ –
Magnitude of the lattice energy of Mg3N2 ∴ ∆H3 = ∆H1 – ∆H2
should be larger than that of MgO as the ionic = (–1367) – (–182)
charge of N3– ions is higher than O2– ions, = –1185 kJ mol–1
as well as there are more ions per mole of __
5
Mg3N2 (as compared to MgO). (iii) CH3CHO + ​ 2 ​ O2 → 2CO2 + 2H2O

4. (a) (i) Heat evolved, q = mC∆T CH3CO2H + 2O2 → 2CO2 + 2H2O


= (200)(4.18)(30.0) ∆Hc for CH3CO2H is expected to be
= 25080 J less exothermic than that of CH3CHO
since more energy is needed to break

Answers to A Level H2 Topical Chemistry A15


© Singapore Asia Publishers Pte Ltd Topic 5

H2 Topical Chemistry 2014.indb 15 3/27/2014 1:57:18 PM


the strong C–O and O–H bonds in
Topic 6 Electrochemistry
CH3CO2H, while the same amount
of energy is evolved when bonds are PAPER 1
formed. Section A
5. ����������������������������������������
(a) The intermolecular hydrogen bonding 1. C
between the NH3 molecules are much Half equations involving electrolyte XCl are:
stronger than the weak intermolecular van 2X+(l) + 2e– → 2X(s)
der Waals' forces between CH4 molecules. 2Cl–(l) → Cl2(g) + 2e–
These requires much more energy to
overcome such strong forces and thus NH3 Half equations involving electrolyte YCl2 are:
has a higher BP than CH4. Y2+(l) + 2e– → Y(s)
(b) When methane is liquefied, the entropy of 2Cl–(l) → Cl2(g) + 2e–
the sample will decrease because the liquid Using the same current for the same time,
state has a more orderly arrangement of Mole of X deposited : Mole of Y deposited
particles as compared to the gaseous state. 2:1
(c) ∆Gθ = ∆Hθ – T∆Sθ __
1
∆H ________
θ
– ∆Gθ  ∴ Mole
������������������������
of Y deposited = ​ 2 ​ × mole of X deposited
∴ ∆Sθ = ​  T ​   
(+23.3 kJ mol–1) – (0.0 kJ mol–1)
___________________________ 2. B
= ​       ​  
(–33 + 273 K) _____
25
mol of Tl+ = 0.20 × ​ 1000  ​ 
= +97.1 J mol–1 K–1 (3 s.f.)
= 5.0 × 10–3
6. (a) Entropy is the measure of the disorder (or
randomness) of a system. _____
25
mol of MnO4– = 0.10 × ​ 1000  ​ 
(b) • Entropy increases as the system becomes
more disorderly due to the process of = 2.5 × 10–3
mixing which results in more ways
∴ 1 mol of MnO4– reacts with 2 mol of Tl+.
of arranging the particles and their
energies. Since 2Tl
�� + → 2Tl3+ + 4e–
• Entropy increases as the system becomes ∴ 1 mol of MnO4– gains 4 mol of e–.
more disorderly due to the increase
in temperature which results in the Oxidation number of Mn in MnO4– = +7
broadening of the Boltzmann energy ∴ Oxidation number of Mn in the reduced form
distribution, and more ways of arranging = +7 – 4
the particles and their energies. = +3
• Entropy decreases as the system becomes
more orderly due to the decrease in number 3. B
of gas molecules, which results in the loss I2 + 2e– l 2I– Eø = +0.54V [O]
of ways of distributing the particles and
their energies. H2O2 + 2H + 2e l 2H2O Eø = +1.77V [R]
+ –

• Entropy increases as the system becomes 2I– + H2O2 + 2H+ → I2 + 2H2O


more disorderly due to the increase in ​E​cell
ø
  ​ =
​  ​EøR​  ​​ – ​EøO​  ​​ 
number of gas particles which results in
more ways of arranging the particles and = (+1.77) – (+0.54)
their energies. = +1.23V
7. ��������������������������������������������
∆G should be negative since the reaction is Since E ​ øcell
​   ​ > 0, reaction is feasible.
spontaneous. KI(aq) is oxidised to I2(aq) which gives the
∆H should be negative since the reaction involves solution the brown colour.
bond-formation of various interactions (i.e. H- H2O2 is reduced to H2O.
bonding, ionic interactions, disulfide linkages and No effervescence since no gas is given off.
van der Waals’ forces between the R-groups of
the amino acids, which are exothermic. 4. C
∆S should be negative since the polypeptide chains Oxidation is involved in the formation of
coalesce into a hemoglobin structure, which is chromium(VI) compounds from chromium(III)
more ordered. bearing rocks.

Answers to A Level H2 Topical Chemistry A16


© Singapore Asia Publishers Pte Ltd Topic 5/Topic 6

H2 Topical Chemistry 2014.indb 16 3/27/2014 1:57:18 PM


From Al2O3 : � Al3+ + 3e– → Al � Eθ = –1.66
������ V
� 8. D
�������������
From CuO : �� Cu + 2e → Cu �
2+ –
E = +�����
θ
0.34 V
� Cu2+ + 2e– → Cu+ Eθ = +0.15V [R]
�������
From Fe2O3 : ��Fe3+ + e– → Fe2+ � Eθ = +0.77
������ V
� I2 + 2e– → 2I– Eθ = +0.54V [O]
�������������
From ZnO : �� Zn2+ + 2e– → Zn � Eθ = –0.76
������ V
� 2Cu2+ + 2I– → 2Cu+ + I2 ​E​θcell   ​ = –0.39V

From the list above, Fe2O3 is the best oxidising ​E​θcell


  ​ = ​E​   ​​ – ​E​   ​ = (+0.15) – (+0.54) = –0.39 V
θ
red
θ
oxd
agent since it has the most positive E�θ value. 9. B
5. A A stronger reducing agent has a more negative
Positively charged ions are discharged at the (i.e. less positive) Eθ value and is the most easily
cathode. Thus, the number of electrons as shown oxidised.
by the electronic configuration must be less than
the proton number. 10. D
Option A shows ion with proton number 11 which Overall reaction is:
is Na+ ion. 2MnO4– + 5SO2 + 2H2O → 2Mn2+ + 5SO42– + 4H+
Option B, C and D show ions formed are As the reaction proceeds, [H+] increases and the
negatively charged. solution becomes more acidic i.e. pH decreases
6. C with time.
Zn2+ + 2e–  Zn Eθ = –0.76 V 11. C
2H+ + 2e–  H2 Eθ = 0.00 V From the Data Booklet :
​E​cell
θ
  ​ ​  = ​E​  ​​ – ​E​  ​ ​
θ
R
θ
O Cl2 + 2e–  2Cl– Eθ = +1.36 V
= (0.00) – (–0.76) Br2 + 2e–  2Br– Eθ = +1.07 V
= +0.76 V I2 + 2e–  2I– Eθ = +0.54V
X : Increase in [Zn2+] shifts the position of Fe + e  Fe Eθ = +0.77 V
3+ – 2+
equilibrium to the RHS and thus ​Eθ​O ​ ​becomes
Fe2+ + 2e–  Fe Eθ = –0.44 V
less negative i.e. E ​ θO​ ​ ​> –0.76 V.
Eθ (Cl2/Cl–) and Eθ (Br2/Br–) are more positive
Hence, ​E​cell θ
  ​ < +0.76 V.
than Eθ (Fe2+/Fe) and Eθ (Fe3+/Fe2+) and so, Cl2
Y : Increase in [H+] shifts the position of
and Br2 would oxidise Fe to Fe3+, giving FeCl3
equilibrium to the RHS and thus ​EθR​ ​​ becomes
and FeBr3 respectively.
more positive i.e. E ​ θR​ ​​ > 0.
Hence, ​E​cell θ
  ​ > +0.76 V.
Eθ (I2/I–) is less positive than Eθ (Fe3+/Fe2+) and
so, I2 would not be able to oxidise Fe2+ to Fe3+.
7. C Eθ (I2/I–) is more positive than Eθ ( Fe2+/Fe) and
Orange Cr 2O 72– turns green suggests that so, would oxidise Fe to Fe2+, giving FeI2 as the
chromium is reduced to Cr3+.
product.
Thus, H2O2 is being oxidised to O2.
 O.N. of oxygen in O2 is 0. 12. C
Cr2O72– + 14H+ + 6e–  2Cr3+ + 7H2O Eθ = +1.33 V [R] Electrode : Carbon electrodes
O2 + 2H+ + 2e–  H2O2 Eθ = +0.68 V [O] Electrolyte : Concentrated Aqueous Sodium Chloride
Cr2O72– + 3H2O2 + 8H + → 2Cr3+ + 3O2+ 7H2O Species : Na+(aq), Cl–(aq), H2O(l)
​EθCell
​  ​ = ​EθR​ ​​ – ​EθO​ ​​ = (+1.33) – (+0.68) = +0.65 V At Cathode : 2H2O(l) + 2e– → H2(g) + 2OH–(aq)
At Anode : 2Cl–(aq) → Cl2(g) + 2e–
∴ feasible reaction since E ​ ​θCell
  ​ > 0

Colourless KI solution turns brownish suggests The Na+ ions that are not selectively discharged
that I– is being oxidised to I2. will form NaOH in solution with OH– ions.
Thus, H2O2 is being reduced to H2O. The chlorine produced will then disproportionates
 O.N. of oxygen in H2O is –2. in cold NaOH to produce NaClO.
I2 + 2e–  2I– Eθ = +0.54V [O] Cl2(g) + 2NaOH(aq) → NaCl(aq) + NaClO(aq)
H2O2 + 2H +2e  2H2O Eθ = +1.77 V [R]
+ –
+ H2O(l)
2I– + H2O2 + 2H+ → I2 + 2H2O
Cl2 is selectively liberated at the anode, instead
​E​θCell
  ​ = ​E​  ​​ – ​E​  ​​ = (+1.77) – (+0.54) = +1.23 V
θ
R
θ
O
of O2 due to the high concentration of NaCl.
∴ feasible reaction since E
​ ​θCell
  ​ > 0

Answers to A Level H2 Topical Chemistry A17


© Singapore Asia Publishers Pte Ltd Topic 6

H2 Topical Chemistry 2014.indb 17 3/27/2014 1:57:19 PM


13. A Therefore,
From the Data Booklet, Eθ(Cu2+/Cu) is the most Cr2+(aq) will be oxidised to Cr3+(aq).
positive among the four options, i.e. Cu has the Fe(OH)2 which is formed from the reaction
least tendency to be oxidised (metal dissolves to between FeSO4 and NaOH, will be oxidised to
form metal ions). Fe(OH)3.
14. D [Fe(CN) 6 ] 3– remains chemically stable
To measure the Eθ of Fe3+/Fe2+, platinum (Pt) (unchanged) because iron is already in its highest
metal is used as the electrode and the ions present oxidation state.
in the electrolyte solution must be Fe2+ and Fe3+
with concentration of 1 mol dm–3. Tips
3 – Fe2+(aq) + 2OH– → Fe(OH)2
15. C from FeSO4 from NaOH
H2O2 must undergo oxidation to give O2(g).
i.e. only occurs in acid oxidation. 19. B
Oxidation equation of H2O2: __
1
1 H+ + e–  ​ 2 ​ H2 Eθ = 0.00 V [O]
H2O2 → O2 + 2H+ + 2e–
Fe3+ + e–  Fe2+ Eθ = +0.77 V [R]
16. D ​E​θcell
  ​ ​  = ​E​  ​​ – ​E​  ​​ 
θ θ
R O
Electrolysis 1 = (+0.77) – (0.00)
At anode: 1 : 2 = +0.77 V
2Cl–(l ) → Cl2(g)  +  2e– 2 At the left hand electrode, oxidation of
​ ______
100
  ​ 
______
200
mol ​ 24,000
  ​ 
mol = I
H+ → H2 takes place and thus it is the
24,000 negative electrode.
Electrolysis 2 3 Under standard conditions, concentrations of
At anode: 1 : 2 : 4 Fe3+ and Fe2+ ions in the right hand solution
4OH–(aq) → O2(g)  +  2H2O(l )  +  4e– should be 1.0 mol dm–3.
​ ______
200
  ​ 
24,000
______
800
mol ​ 24,000 ​mol
    20. D
1 – As the [CH3CH2OH] increases at electrode
= 4I
1, equilibrium shifts to the RHS to remove
Tips the extra CH3CH2OH. Oxidation occurs and
Current ∝ no. of moles of electrons. more electrons are produced. Thus, electrode
potential becomes more negative.
Section B 2 – Oxidation number of hydrogen remains as
17. B +1. It is not reduced at electrode 2.
Fe3+ + e– → Fe2+ Eθ = +0.77 V 3 – Oxidation number of oxygen remains
Cl2 + 2e– → 2Cl– Eθ = +1.36 V unchanged at electrode 1 but changes from
0 (in O2) to –2 (in H2O) at electrode 2.
Br2 + 2e– → 2Br– Eθ = +1.07 V
I2 + 2e– → 2I– Eθ = +0.54 V 21. B
Oxidising agent must have Eθ value more positive 1 – Correct statement. The H2 gas used in S.H.E.
than +0.77 V, in order to oxidise Fe2+(aq) to is 1 atm.
Fe3+(aq). Thus, only Cl2 and Br2 can oxidise 2 – Correct statement. The electrodes are
connected through a high-resistrance
Fe2+(aq) to Fe3+(aq).
voltmeter so that the current in the external
18. D circuit is virtually zero and the cell registers
When left to stand in the atmosphere, only its maximum potential difference, which is
solutions with �
Eθ value less positive than +1.23 V called the electromotive force (e.m.f.) of the
will be oxidised. cell.
O2 + 4H+ + 4e– → 2H2O � Eθ = +
�������
1.23 V� 3 – pH 1.0 means [H+] = 0.10 mol dm–3.
Standard conc. of hydrogen ions, [H+] should
[Fe(CN)6] + e → [Fe(CN)6] E = + 0.36 V
3– – 4– θ
be 1.0 mol dm–3.
Fe(OH)3 + e– → Fe(OH)2 + OH– Eθ = +1.07 V
Cr3+ + e– → Cr2+ Eθ = – 0.41 V

Answers to A Level H2 Topical Chemistry A18


© Singapore Asia Publishers Pte Ltd Topic 6

H2 Topical Chemistry 2014.indb 18 3/27/2014 1:57:19 PM


22. D (b) 0.100 mol dm–3 solution of Na2C2O4 is
Based on Faraday's First Law of Electrolysis, considered too low since the titre volume
the mass of a substance liberated at an electrode at the end-point of tiltration would be very
during electrolysis is directly proportional to ___
32
large (i.e. about ​ 10 ​ =3.2 times the volume
the quantity of electricity passed through the of FA1 titrated).
electrolyte. For example, if we were to titrate 25.0 cm3
m α Q  or  m α I  t  since Q = I  t of FA1, it would require a titre volume of
where m = mass of substance liberated in about 80 cm3 . ���������������������������
This would require careful
grams, g refilling of the 50.0 cm3 burette since the
Q = quantity of electricity in coulombs, C titration would require more than one 50.0
cm3 burette full of Na2C2O4.
PAPER 2 (c) Volume of aliquot is approximately the same
1. ��������
(a) (i) as the volume of the titre.
If 25.0 cm3 of FA1 is used, the volume of FA2
Oxidation Oxidation
Species
number
Species
number
(Na2C2O4) required should be about 25.0 cm3.
reactant Cl2 0
Thus, FA2 must be made more concentrated.
i.e., 10 drops of FA1 should require 10 drops
products NaCl –1 NaClO +1
of FA2 for complete reaction.
(ii) Cl2 undergoes disproportionation i.e. it \ Concentration of Na2C2O4 to be increased
___
32
is simultaneously oxidised to NaClO by ​ 10 ​ = 3.2 times
and reduced to NaCl. [Na2C2O4]new = 0.100 × 3.2
(b) (i) H2O2 → O2 + 2H+ + 2e– = 0.320 mol dm–3
(ii) H2O2 + 2ClO3– + 2H+ → O2 + 2ClO2 + Mr of Na2C2O4 = (2)(23.0) + (2)(12.0)
2H2O + (4)(16.0)
(iii) H2O2 acts as the reducing agent. = 134.0
If 250 cm3 volumetric flask is used:
2. (a) ​E​θcell
  ​ = ​E​  ​​ – ​E​  ​​ 
θ
R
θ
O No. of moles of Na2C2O4
= +1.52 – (+0.70) = Molar Concentration × Volume
= +0.82 V _____
250
= 0.320 × ​ 1000  ​ 
(b) 5C6H6O2 + 2MnO​– 4 ​ + 6H+
= 0.08 mol
→ 5C6H4O2 + 2Mn2+ + 8H2O Mass of Na2C2O4 = Mole × Mr
(c) To reduce quinone to quinol, the reducing = 0.08 × 134.0
agent must have an Eθ value less positive = 10.72 g
than +0.70 V. Procedure:
SO2 gas can be used as the reducing agent. (1) Weigh a clean and dry weighing bottle.
SO​  2–
   ​(
  aq) + 4H+(aq) + 2e– Carefully place about 10.72 g of
4
anhydrous Na2C2O4 into the weighing
→ SO2(g) + 2H2O(l) Eθ = +0.17 V
bottle and reweigh.
∴ ​Eθcell
​  ​ = ​EθR​ ​​ – ​EθO​ ​​  (2) Transfer the weighed solid content into
= +0.70 – (+0.17) a 250 cm3 beaker. Rinse the weighing
= +0.53 V, feasible reaction. bottle with distilled water and transfer
all the rinsings into the beaker.
3. Planning (P)
(3) Add about 100 cm3 of distilled water
(a) 2FeO42– + 3C2O42– + 16H+ → 2Fe3+ + 6CO2
into the beaker and gently stir with a
+ 8H2O
glass rod to dissolve all the Na2C2O4
Tips completely.
8H+ + FeO42– + 3e– → Fe3+ + 4H2O –– (4) Transfer the solution of Na2C2O4 from
C2O42– → 2CO2 + 2e– –– the beaker into a 250 cm3 volumetric
( × 2) + ( × 3); flask. Wash the glass rod and beaker a
∴ 2FeO42– + 3C2O42– + 16H+ → 2Fe3+ + 6CO2 + 8H2O
few times and transfer all the washing
into the volumetric flask. This would

Answers to A Level H2 Topical Chemistry A19


© Singapore Asia Publishers Pte Ltd Topic 6

H2 Topical Chemistry 2014.indb 19 3/27/2014 1:57:20 PM


ensure the complete transfer of Na2C2O4 4. ��������
(a) (i)
solution. Element Cl H N Pb
(5) Mark up to the 250 cm3 mark by adding
Mass (g) 46.7 1.76 6.14 45.4
distilled water into the volumetric
Ar 35.5 1.0 14.0 207
flask.
Mole 1.315 1.76 0.4386 0.2193
(6) Stopper and shake the solution well to
obtain a homogenous solution of FA2. Molar Ratio 6 8 2 1
(7) Transfer the FA2 solution into a ∴ Empirical formula of Y is Cl6H8N2Pb.
burette.
(ii) Anion: [PbCl6]2– Cation: NH4+
(8) Pipette out 25.0 cm3 of FA1 into a
conical flask and add an equal volume Tips
of dilute H2SO4(aq) using a measuring Ratio of anions : cations
cylinder. 2:1
(9) Titrate the solution of FA1 against FA2 2 × NH4+ : 1 × [PbCl6]2–
from a burette until the dark red solution
in the conical flask first turns yellow. (iii) Let the oxidation state of Pb in Y to be x.
(10) Repeat the titration until 2 readings are ∴ x + (6)(–1) = –2
within ±0.10 cm3. ∴ x = –2 + 6 = +4
Tips (iv) Shape: Octahedral
• FA1 (K2FeO4 solution) is placed inside the conical flask.
• FeO42– ions have a dark red colour. When they oxidise Tips
another chemical in an acidic solution, the product is a 2–
solution of Fe3+ ions, which is yellow in colour. Cl
• This means that K2FeO4 solution can act as its own Cl Cl
indicator. Pb
• FA2 (Na2C2O4 solution) is placed inside the burette. Cl Cl
Cl
(d) From the balanced chemical equation in (a)
Mole of FeO42– (in FA1) __ (b) Pb4+ + 2e–  Pb2+ Eθ = +1.69 V [R]
____________________ 2
​     ​= ​   ​ 
Mole of C2O42– (in FA2) 3 VO + 2H + e  V + H2O Eθ = +0.34 V [O]
2+ + – 3+

∴ Mole of FeO42– (in FA1) ∴ ​Eθcell


​  ​ =
​  ​Eθred
​  ​​ – ​Eθoxd
​  ​ 
__
2 = (+1.69) – (+0.34)
= ​ 3 ​  × Mole of C2O42– (in FA2)
= +1.35 V
__
2 y
_____
= (​ 3 ​  × M × ​ 1000   ​ 
) mol Since E ​  ​ > 0; ∴ reaction is feasible.
​ θcell
Concentration of FeO42– (in FA1) Pb4+ + 2e–  Pb2+ Eθ = +1.69 V [R]
Mole of FeO42– VO2 + 2H + e  VO + H2O Eθ = +1.00 V [O]
+ + – 2+
_____________
= ​      ​  ∴ ​Eθcell
​  ​ =
​  ​Eθred
​  ​​ – ​Eθoxd
​  ​ 
Volume
__
2 y
_____ = (+1.69) – (+1.00)
(​ 3 ​  × M × ​ 1000
_________________    ​ 
) mol
= ​  _____
x       ​ = +0.69 V
(​ 1000
   ​ 
) dm3 Since E ​  ​ > 0; ∴ reaction is feasible.
​ θcell
2My
____ Thus, V3+ will be oxidised to VO2+ and
= ​  3x ​ 
 mol dm–3
finally to VO2+ while Pb4+ will be reduced
Hence, concentration of FA1, to Pb2+.
2My
____
K2FeO4 = ​  3x ​ 
 mol dm–3
(e) ��
Na2C2O4 is toxic to humans. It is harmful if PAPER 3
ingested and is also corrosive to the skin. 1. (a) Products at the cathode are H2(g) and
To minimise this risk, wear gloves and use NaOH(aq).
spatula when handling Na2C2O4(s), to avoid At Anode : 2R–CO2– → R–R + 2CO2 + 2e–
skin contact. At Cathode: 2H2O + 2e– → H2 + 2OH–
Overall equation :
2R–CO2– + 2H2O → R–R + 2CO2 + H2 + 2OH–

Answers to A Level H2 Topical Chemistry A20


© Singapore Asia Publishers Pte Ltd Topic 6

H2 Topical Chemistry 2014.indb 20 3/27/2014 1:57:20 PM


(b) Amount of charge, Q = I × t (c) (i) CH3CH2OH + 3O2 → 2CO2 + 3H2O
= 2 × (40 × 60) Bonds Broken HB–B Bonds Formed HB–F
= 4800 C 1 C–C (+350) 4 C=O (4)(–805)
To form 1 mol of C 2H 6, it requires 5 C–H (5)(+410) 6 O–H (6)(–460)
2 × (9.65 × 104 C mol–1). 1 C–O (+360) –5980
__________________
4800 C 1 O–H (+460)
No. of mol of C2H6 = ​       ​
2 × (9.65 × 104 C mol–1) 3 O=O (3)(+496)
= 0.0249 mol +4708

Mr of C2H6 = 30 ∆H = HB–B + HB–F


∴ Mass of C2H6
= (+4708) + (–5980)
= Mole of C2H6 × Mr of C2H6
= 0.0249 × 30 = –1272 kJ mol–1
= 0.747 g (ii) The calculated value in (c)(i) is smaller
(ie. less negative) than that given
2. ���������
(a) (i) O2 + 4H+ + 4e– → 2H2O in (b)(ii) because bond energies are
(ii) CH3OH + H2O → CO2 + 6H+ + 6e– – average values and do not relate to any
O2 + 4H+ + 4e– → 2H2O – specific compounds.
( × 2) + ( × 3); 3. ���������������������������������������������������
In the purification of copper by electrolysis, the
2CH3OH + 3O2 → 2CO2 + 4H2O impure copper is made the anode of the electrolytic
(iii) From the Data Booklet; cell. The cathode is pure copper and the electrolyte
O2 + 4H+ + 4e– → 2H2O ​E​θR ​​ = +1.23 V used is aqueous CuSO4.
Since E
​ Cell
​θ  ​ ​ = ​EθRed
​  ​ – ​EθOxd
​  ​  Form the Data Booklet:
Ag+ + e–  Ag Eθ = +0.80 V
=E
​ ​θRed
  ​ – ​E​  
θ
​ 
(CO /CH OH)
2 3
Cu2+ + 2e–  Cu Eθ = +0.34 V
∴ ​E​θ(CO
  ​ ​E​θRed
​ =   ​ – ​E​   ​ 
θ
/CH OH) 2 3
Cell 2H + 2e  H2
+ –
Eθ = 0.00 V
= (+1.23) – (1.18)
= +0.05 V Zn2+ + 2e–  Zn Eθ = –0.76 V
(iv) Methanol is a liquid at room temperature At the +ve anode, copper dissolves.
and pressure, and thus easier to store
Cu(s) → Cu2+(aq) + 2e– [oxidation]
and transport as compared to H2(g).
(b) (i) CH3CH2OH + 3O2 → 2CO2 + 3H2O Zinc impurity, which has less positive Eθ value
(ii) ∆Gθ = ∆Hθ – T∆Sθ than copper, also dissolves and stay as Zn2+ ions
in solution.
= –1367 – (298)(–140 × 10–3)
= –1325 kJ mol–1 Zn(s) → Zn2+(aq) + 2e– [oxidation]
Common Mistakes Silver impurity, which has a more positive Eθ
Remember to convert the units for ∆Hθ & ∆Sθ to be the value than copper, remains undissolved and drops
same. to the bottom of the cell as 'anode sludge'.
At the –ve cathode, copper(II) ions from the
(iii) From (a)(i): solution are discharged and pure copper are
3O2 + 12H+ + 12e– → 6H2O deposited on the cathode.
From (a)(ii):
Cu2+(aq) + 2e– → Cu(s) [Reduction]
CH3CH2OH + 3H2O → 2CO2 + 12H+
+ 12e– Since Eθ(Cu2+/Cu) > Eθ(Zn2+/Zn), Zn2+ ions
∴ z = 12 remains to be dissolved in solution. Only Cu2+
_____
–∆Gθ ions is reduced to copper and deposited on the
Hence, ​ECell
​θ  ​ = ​  zF ​ 
  cathode.
–(–1325 × 103)
_____________ Pure copper cathode will increase in size/mass.
= ​    
    ​
(12)(96500) The concentration of CuSO4(aq) electrolyte
= +1.14 V remains unchanged.
Common Mistakes
Remember to convert the units of ∆Gθ to joules per mole.

Answers to A Level H2 Topical Chemistry A21


© Singapore Asia Publishers Pte Ltd Topic 6

H2 Topical Chemistry 2014.indb 21 3/27/2014 1:57:21 PM


4. (a) Let the oxidation number of iodine be x. ___
1
∴ Mole of ClOx– = ​ 2x  ​ × Mole of e–
–1 = 4(–1) + x
∴ x = +3 _______________
0.150 ___
1
+ 16x ​ = ​ 2x  ​ (0.00565)
​ 39.1 + 35.5  
  
(b) (i) HClO + H+ + 2I– → Cl– + I2 + H2O
_______________
0.150 _______
0.00565
Tips + 16x ​ = ​  2x   
​ 39.1 + 35.5  
   ​ 

step : Balance each half equation: HClO → Cl– and 0.3x = 0.42149 + 0.0904x
2I– → I2, under acidic medium. 0.2096x = 0.42149
step : Combine the two half equations together.
x  2
Hence;
(ii) Mr of N-chlorosuccinimide
Fe2+ → Fe3+ + e– –
= [4(12.0) + 4(1.0) + 35.5 + 14.0 + 2(16.0)] ClO2– + 4H+ + 4e– → Cl– + 2H2O –
= 133.5
_____
6 +[×4];
Mole of Na2S2O3 = ​ 1000    ​ × 0.0050
4Fe2+ + ClO2– + 4H+ + 4e– →
= 3 × 10–5 mol 4Fe3+ + 4e– + Cl– + 2H2O
= Mole of S2O32– ∴ 4Fe2+ + ClO2– + 4H+ → 4Fe3+ + Cl + 2H2O
Mole of I2
____________ __
1
​        ​ = ​   ​  6. (a) (i) Electrochemical cell to measure
Mole of S2O32– 2
__
1 standard electrode potential of Cl2:
Mole of I2 = ​ 2 ​  × (3 × 10–5 mol)
high-resistant voltmeter
= 1.5 × 10–5 mol
Mole of I2 = Mole of HClO V
salt bridge
= 1.5 × 10–5 mol
H2(g) at 298 K
= Mole of N-chlorosuccinimide & 1 atm Cl2(g) at 298 K
Hence, mass of N-chlorosuccinimide & 1 atm
= Mole × Mr
= (1.5 ×10–5) × (133.5) 1.0 mol dm–3 Cl–(aq);
= 2.0025 × 10–3 H+(aq) 1 mol dm–3

= 0.00200 g
5. No. of moles of Fe2+ platinised platinum electrode
= Molar Concentration × volume Standard Hydrogen Cl2/Cl–
_____
11.3 Electrode
= 0.500 mol/dm3 × (​ 1000  ​ 
)dm3 Half-cell
= 0.00565 mol Electrochemical cell to measure

Fe2+ → Fe3+ + e– standard electrode potential of I2:
high-resistant voltmeter
∴ No. of moles of e– = No. of moles of Fe2+
V
KClOx → K+ + ClOx– salt bridge
∴ No. of moles of ClOx– H2(g) at 298 K
Pt(s)
& 1 atm
= No. of moles of KClOx
_____
Mass
= ​  M  ​ 
r
_______________
0.150 1.0 mol dm–3 I​ ​–3​(​  aq)/I–(aq)
+ 16x ​mol
= ​ 39.1 + 35.5  
  
H+(aq)
ClOx– + 2xH+ + 2xe– → Cl– + xH2O
Based on the above equation; platinised platinum electrode
Mole of ClOx– ___ Standard Hydrogen ​I–​3​(​  aq)/I–(aq)
____________ 1 Half-cell
​     = ​ 2x  ​ 
 ​  Electrode
Mole of e–

Answers to A Level H2 Topical Chemistry A22


© Singapore Asia Publishers Pte Ltd Topic 6

H2 Topical Chemistry 2014.indb 22 3/27/2014 1:57:21 PM


(ii) From the voltmeter, the e.m.f. of the
Topic 7.1 Chemical Equilibria
Cl2(g)/Cl–(aq) cell will be determined
to be 1.36 V which means the standard PAPER 1
electrode potential for Cl2/Cl– would be Section A
+1.36V. 1. A
From the voltmeter, the e.m.f. of the 2NO2(g)  2NO(g) + O2(g)
I3–(aq)/I–(aq) cell will be determined initial atm: P O O Total pressure = P atm
to be 0.54V which means the standard __1 ___
120
electrode potential for I2/I– would be Eqm atm : P – a a ​ 2  ​a Total pressure = ​ 100  ​ P atm
+0.54V. __
1 ____
120
Strong oxidising agents have high ∴ (P – a) + a + ​ 2 ​ a = ​ 100 ​ P
positive standard electrode potential ∴ a = 0.4P
values and are easily reduced by
gaining electrons. As such, chlorine is Therefore, at equilibrium:
more oxidising than iodine. 2NO2(g)  2NO(g) + O2(g)
(iii) Cl2(g) + 2I–(aq) → I2(s) + 2Cl–(aq)
__
1
Eqm atm: P – 0.4P
0.4P ​   ​  × 0.4P
2
Tips = 0.6P = 0.2P
Cl2(g) → 2Cl–(aq) + 2e– Eθ = +1.36V [R]
2I–(g) + 2e– → I2(s) Eθ = +0.54V [O] ∴ Mole fraction, x, of O2 in equilibrium mixture
________________
0.2P
Cl2(g) + 2I–(aq) → I2(s) + 2Cl–(aq) ​E​θcell
  ​ = +0.82V = ​ 0.6P + 0.4P +  
0.2P ​
= 0.17
(b) (i) 2Fe3+(aq) + 2I–(aq)  2Fe2+(aq) + I2(aq)
(ii) Eθ(Fe3+/Fe2+) = +0.77V [R] 2. C
Eθ(I2/I–) = +0.54V [O] If ∆H = 0, changing the temperature does not shift
∴ ​Eθcell
​  ​ =
​  ​EθR​ ​​ – ​EθO​ ​​  the equilibrium to the left or right, and equilibrium
= (+0.77) – (+0.54) concentration of product and reactants do not
= +0.23 V change. This makes the equilibrium constant to
Therefore, reaction is feasible and remain the same.
Fe3+ is reduced to form Fe2+ and I– is
3. B
oxidised to form I2 spontaneously.
X2(g)  2X(g) ∆H = +ve
Fe3+(aq) exists as the hydrated ions
[Fe(H2O)6]3+(aq). In the presence of As temperature increases, the position of the
CN–(aq) ions, there will be ligand equilibrium shifts to the RHS, which favours the
exchange reaction and H2O molecules forward reaction (endothermic) so as to remove
would be replaced by CN–(aq) ions. the extra heat.
[Fe(H2O)6]3+(aq) + 6CN–(aq) → Pressure increases since the number of mole of
​[Fe(CN)6]​3–  ​ + 6H O
(aq) 2 (l ) gases increases i.e. 1 mol of X2(g) gives 2 mol
This ligand exchange reaction would of X(g).
cause the [Fe3+(aq)] to decrease and
the system would want to compensate Tips
it by shifting the position of the Pressure  no. of moles of gases.
equilibrium towards the left (i.e. to
form more Fe3+).
4. C
N2O4  2NO2
Initial/mol : 1 0
Equilibrium/mol : 0.5 (2)0.5
Total no. of mole of gas = 0.5 + 1
= 1.5 mol at equilibrium

Answers to A Level H2 Topical Chemistry A23


© Singapore Asia Publishers Pte Ltd Topic 6/Topic 7.1

H2 Topical Chemistry 2014.indb 23 3/27/2014 1:57:22 PM


___
0.5
∴ ​P​N O​= ​ 1.5 ​ × 1 atm Tips
2

__
1 HCl is a strong acid and 1 mole of HCl gives 1 mole of H+.
= ​ 3 ​  atm
HCl → H+ + Cl–
___1
​P​NO ​= ​ 1.5   ​ × 1 atm Both H3PO3 and H3PO4 are assumed to be strong dibasic
2
acids. 1 mole of each acid will give 2 mole of H+.
__
2
= ​ 3 ​  atm H3PO3 → 2H+ + HPO32–
H3PO4 → 2H+ + HPO42–
​( ​P​NO ​ )2​ Thus;
______
Hence, Kp = ​   
 ​  PCl5 + 4H2O → 5HCl + H3PO4
2

​( ​P​N O ​ )2​
2 4
gives 5 gives 2

(  )
mole of H+ mole of H+
__
2 2
​​ ​ 3 ​   ​​ ​ PCl3 + 3H2O → 3HCl + H3PO3
____
= ​   ​ 
(  )
__
1
​ ​ 3 ​   ​


gives 3 gives 2
mole of H+ mole of H+

__
4
= ​ 3 ​  atm (b) (i) No. of moles of NaOH used
5. C _____
37.4
= 4 mol dm–3 × ​ 1000  ​ dm3
2 : 1 : 1 = 0.1496 mol
2H2(g) + CO(g)  CH3OH(g) = no. of moles of OH–
Initial /mol: 2.0 1.0 0
No. of moles of H+ in 25.0 cm3
__
1 __
1 = No. of moles of OH–
Change /mol: x ​ 2 ​  x ​ 2 ​  x

= 0.1496 mol
__
1 __
1
Equilibrium /mol: 2.0 – x 1.0 – ​ 2 ​  x ​ 2 ​  x
∴ No. of moles of H+ in 1.00 dm3

_____
1000
= 0.1496 × ​  25 ​  
PAPER 2 = 5.984 mol
[PCl3][Cl2]
_________  5.98 mol
1. Kc = ​   ​ 
 
mol dm–3
[PCl5] (ii) 7 – 2x = 5.984
2. ��������
(a) PCl5(g)  PCl3(g) + Cl2(g) ∴ x = 0.508
Initial/mol : 1.00 0 0 Amount of PCl5 = 1.00 – x
Change/mol : –x +x +x = 1.00 – 0.508
Equilibrium/mol : 1.00–x x x = 0.492 mol
For (1.00 – x) mol of PCl5, Amount of PCl3 = x = 0.508 mol
No. of moles of H+ formed = (7)(1.00 – x) Amount of Cl2 = x = 0.508 mol
= 7 – 7x _____
0.508 _____0.508
(​  5 ​   )(​  5 ​ 
[PCl3][Cl2] _____________
_________  )
For x mol of PCl3, (c) Kc = ​   ​ =
 
 ​ 
No. of moles of H+ formed = (5)(x) [PCl5] 0.492  
_____  ​ 
(​  5 ​   )
= 5x
= 0.105 mol dm–3
∴ Total number of moles of H+ formed
= (7 – 7x) + (5x)
PAPER 3
= 7 – 2x ​P​CH CH CH CHO​
__________________
1. (a) Kp = ​        ​atm–2
3 2 2

(​P​CH CH = CH ​)(PCO)(​P​H ​)
(b) (i) CH3CH = CH2 + CO + H2  CH3CH2CH2CHO
3 2 2

Initial/mol 1 1 1 0
___
​  120 ___
120 ___
120
Initial/atm 3   ​ = 40 ​  3   ​ = 40 ​  3   ​ = 40 0 PT = 120 atm
Change/atm –39.6 –39.6 –39.6 +39.6
Eqm/atm 0.4 0.4 0.4 39.6
∴ Partial pressure of CH3CH=CH2,
CO and H2 at equilibrium = 0.4 atm
_____
39.6
(ii) Kp = ​    ​ 
= 619 atm–2
(0.4)3

Answers to A Level H2 Topical Chemistry A24


© Singapore Asia Publishers Pte Ltd Topic 7.1

H2 Topical Chemistry 2014.indb 24 3/27/2014 1:57:22 PM


(iii) A higher pressure would favour the
Topic 7.2 Ionic Equilibria
formation of butanal. According to
Le Chatelier's Principle, an increase PAPER 1
in pressure will shift the position of Section A
equilibrium to the right (the side with 1. C
H2O(l)  H+(aq) + OH– (aq)
fewer gas molecules) so as to remove
some of the excess pressure. Kw = [H+][OH–]
[Hb(O2)4] ​[H+]​2​= Kw since [H+] = [OH–]
________
2. (a) (i) Kc = ​   ​ 
dm12 mol–4 __________
[Hb][O2]4 ∴ [H+] = √
​ 1.44 × 10–14 ​ 
__________
1 = 1.20 × 10–7 mol dm–3
(ii) Kc = ​     ​  since [Hb] = [Hb(O2)4]
(7.6 × 10–6)4
pH = –log10[H+]
= 3.00 × 1020 dm12 mol–4
= –log10(1.20 × 10–7)
(iii) For 99% of Hb to be converted to = 6.92 ∴ pH < 7
Hb(O2)4, it means 1% Hb remains.
2. B
[Hb(O2)4] ___
________ 99
∴ ​   ​  
 = ​  1 ​  For a weak acid HA,
[Hb] HA(aq)  H+(aq) + A–(aq)
______
99
Hence; Kc = 3.00 × 1020 = ​     ​   [H ][A–]
+
_______
1[O2]4 Ka = ​  [HA] ​ 
 
[O2]4 = 3.30 × 10–19
__________ pH = pKa when Ka = [H+] i.e. when [A–] = [HA]
4
∴ [O2] = ​√  3.30 × 10–19 ​  This occurs at the point of half-neutralisation
___
10
= 2.40 × 10–5 mol dm–3 when volume of NaOH = ​  2 ​ = 5 cm3
(b) Mb(aq) + O2(aq)  MbO2(aq) 3. B
[MbO2] The main buffer in blood plasma is the hydrogen
________ carbonate (���
HCO3–) and carbonic acid (� H2CO3)
Kc = ​   
 ​ 
[Mb][O2]
buffer system.
[MbO2]
______________ Contaminating H+(aq) ions are removed by
1 × 106 = ​    
   ​
[Mb](7.6 × 10–6) HCO3– ����������������������������
ions based on the equation:
[MbO2]
_______ ���HCO3– + H+ → H2CO3
∴ ​   ​ =
 
  7.6
[Mb]
4. D
 [MbO2] = 7.6[Mb] Ka, the acid dissociation constant, is only affected
Hence; % of MbO2 in Mb-MbO2 mixture by changes in temperature. It is not affected by
[MbO2] changes in volume, V.
_____________
= ​    
  ​× 100
[Mb] + [MbO2] 5. C
7.6[Mb]
______________ A buffer solution is a solution whose pH remains
= ​      ​× 100 almost unchanged (changes very slightly) on
[Mb] + 7.6 [Mb]
dilution or when small amounts of acid or base
= 88.4% are added to it.
Soft drinks are considered acidic buffers of weak
citric acid and sodium citrate (salt of the acid).
If a small amount of acid, H+ ions, is added to
the buffer solutions, the additional H+ ions are
removed by the large concentration of citrate ions
from the salt. Hence, pH should decrease very
slightly.
If a small amount of base, OH– ions, is added to
the buffer solutions, the additional OH– ions are
removed by the large concentration of citric acid
present in the buffer.
Hence, pH should increase very slightly.

Answers to A Level H2 Topical Chemistry A25


© Singapore Asia Publishers Pte Ltd Topic 7.1/Topic 7.2

H2 Topical Chemistry 2014.indb 25 3/27/2014 1:57:22 PM


6. A Results from student B:
For a weak acid (pyruvic acid) – strong base AgNO3(aq) + KBr(aq) → AgBr(s) + KNO3(aq)
(NaOH) titration, we would observe the following cream PPT
characteristics on the titration graph:
AgBr(s) + CH3COO Na (aq) → No Reaction
– +
① Titration graph begins at pH ≈ 3 (and not 1),
indicating presence of a weak acid. (i.e. no CH3COO–Ag+ formed)
② Titration graph ends at pH ≈ 13 – 14, indicating 1 – From the results, silver ethanoate
the presence of a strong base. (CH3CO2–Ag+) is an insoluble white PPT.
③ pH at the end-point (middle of the vertical 2 – From the results, AgBr is less soluble than
part of graph is greater than 7, due to the CH3COO–Ag+.
hydrolysis of the salt). 3 – Bromide is not oxidised by ethanoate.
7. D
Since stomach juices have a pH of 1.0, PAPER 2
∴ [H+] = 1  10–1 moldm–3 1. �����
(a) Ksp = [Ag+][Br–]
HA  H+ + A– (b) AgBr(s)  Ag+(aq) + Br–(aq)
[H+][A–]
_______ 7.1 × 10–7 7.1 × 10–7 7.1 × 10–7
Ka = ​  [HA] ​ 
  Ksp = [Ag+][Br–]
0.1[A ]

_______ = (7.1 × 10–7)2
10–4 = ​  [HA] ​  = 5.0 × 10–3 mol2 dm–6
[A–] = [HA]  10–3 2. (a) (i) Ksp = [Pb2+][Cl–]2 units
∴ [A ] < [HA]

= mol3dm–9
[H+] ≠ [A–] as H+ is mainly contributed by the (ii) PbCl2(s)  Pb2+(aq) + 2Cl–(aq)
strong acid in stomach. Solubility of PbCl2
4.7 g dm–3
__________
8. D = ​    ​ 
Mr of PbCl2
1 : 1 : 1 : 1 4.7 g dm
_________
–3

HCl(aq)  +  NaOH(aq) → NaCl(aq)  +  H2O(l ) = ​  278 ​   


0.0040 mol 0.0025 mol = 0.01691 mol dm–3
Excess Limiting In a saturated solution,
Reagent Reagent [Pb2+] = [PbCl2] = 0.01691 mol dm–3
NaCl (a neutral salt) and H2O have pH ≈ 7 and [Cl–] = 2(0.01691) = 0.03382 mol dm–3
thus the only substance to cause any pH changes ∴ Ksp = [Pb2+][Cl–]2 units = mol3dm–9
would be the unreacted excess HCl(aq). = (0.01691 mol dm–3)(0.03382 mol dm–3)2
Unreacted excess HCl = 0.0040 – 0.0025 = 1.93  10–5 mol3 dm–9
= 0.0015 mol ≈ 1.9  10–5 mol3 dm–9 (2 s.f.)
(b) (i) Immediately : white precipitate
[HCl] = 0.0015 mol/dm3
After one hour in the daylight : purple-
HCl(aq)  →  H+(aq) + Cl–(aq) grey solid
0.0015 mol/dm3 0.0015 mol/dm3
Tips
pH = –lg[H+]
Immediately:
= –lg(0.0015) HCl(aq) → H+(aq) + Cl–(aq)
≈ 2.82 AgNO3(aq) → Ag+(aq) + N​O​–3​(​  aq)
Ag+(aq) + Cl–(aq) → AgCl(s)
Section B After one hour in daylight:
9. B Colour changes which occur when AgCl are exposed to
Results from student A: sunlight, resulting from the superficial conversion of these
AgNO3(aq) + CH3COO–Na+(aq) → silver halides to silver and halogen.
__
1
AgCl(s) → Ag(s) + ​ 2 ​  Cl2(g)
CH3COO–Ag+(s) + NaNO3(aq)
white PPT
(ii) When aqueous ammonia is added to
CH3COO–Ag+(s) + KBr(aq) → AgCl, the precipitates will dissolve
AgBr(s) + CH3COO–K+(aq) readily to give a colourless solution,
cream PPT

Answers to A Level H2 Topical Chemistry A26


© Singapore Asia Publishers Pte Ltd Topic 7.2

H2 Topical Chemistry 2014.indb 26 3/27/2014 1:57:23 PM


due to the formation of soluble complex Based on Le Chatelier's Principle,
ion. the decrease in [H+] would shift the
AgCl(s) + 2NH3(aq) → [Ag(NH3)2]+(aq) + Cl–(aq) equilibrium position to the right. This
(iii) On adding concentrated HCl, the results in an increase in [S2–]. Hence,
solubility of PbCl2 increases due to the NiS will be precipitation in alkaline
formation of complex ion, PbC​l2– ​ ​  ​(aq). solution.
4
PbCl2(s) + 2Cl–(aq) → PbC​l2– ​4​  ​(aq) In the neutral or acidic solution,
[S2–] is not high enough for NiS to be
Common Mistakes precipitated.
On adding dilute HCl, [Cl–] is increased and this shifts i.e. Ionic Product (ZnS) < Ksp (Zns)
the position of equilibrium to the left. Hence, solubility of
PbCl2 decreases due to the common ion effect.
(iii) ZnS has Ksp value ( 10–24) that is
between that of CuS ( 10–36) and NiS
PbCl2(s)  Pb2+(aq) + 2Cl–(aq)
HCl(aq) → H+(aq) + Cl–(aq) ( 10–21).
common ion In a neutral solution, there will be
sufficient [S2–] for the lonic Product
(c) Copper(II) salts such as aqueous copper(II) (ZnS) > Ksp (ZnS) for precipitation to
chloride dissolve in water to give a blue occur.
colour solution, which is attributed to the With an alkaline solution, the OH– ions
[Cu(H2O)6]2+ ions. will react and reduce the [H+] in the
When excess Cl– (i.e. concentrated HCl) is solution.
added, the solution turns yellow due to the HS–(aq)  H+(aq) + S2–(aq)
formation of a tetrahedral complex through Based on Le Chatelier's Principle,
ligand exchange reaction. the decrease in [H+] would shift the
[Cu(H2O)6]2+ + 4Cl–  [CuCl4]2– + 6H2O equilibrium position to the right. This
yellow tetrahedral complex
results in an increase in [S2–]. Hence,
The reaction is reversible and the addition ZnS will be easily precipitated in
of water to the yellow solution would cause alkaline solution.
the equilibrium to shift towards the left. In the acidic solution, [S2–] is not high
The chloride ions are replaced as ligands enough for ZnS to be precipitated
by the water molecules to form the blue i.e. Ionic Product (ZnS) < Ksp (Zns)
[Cu(H2O)6]2+ ions.
(d) (i) In acidic solution, there's an increase PAPER 3
in [H+] and based on Le Chateliers 1. ������������
(a) HCN(aq)  H+(aq) + CN–(aq)
Principle, equilibrium of HS–  H+ + S2– [H+][CN–]
_________
will shift to the left and [S2–] decreases, Ka = ​  [HCN] ​   
CuS salt has a much lower Ksp ( 10–36)
[H+][CN–]
_________
than NiS salt ( 10–21) and ZnS ( (b) Ka = ​  [HCN] ​   
10–24) and is less soluble in water. This
means that CuS salt will be easier to be [H+]2
_____
4.9 × 10–10 = ​ 0.100  ​ 
precipitated, with a low [S2–].
[H+]2 = 49 × 10–12
Tips
[H+] = 7.00 × 10–6 mol dm–3
• Ksp applies only to sparingly soluble salt and is a measure
_________
7.00 × 10–6
of its solubility in water or aqueous solution. ∴ % of HCN ionised = ​  0.100 ​   
× 100%
• Salt with a low Ksp value is less soluble, while salt with
a high Ksp value is more soluble. = 0.007%
(c) HCN  H+ + CN– ...... (1)
(ii) NiS has a high Ksp ( 10–21) and would NaCN → Na+ + CN–
need [S2–] to be high enough so that common ion
Ionic Product > Ksp for precipitation The addition of NaCN will increase the
to occur. [CN–].
With an alkaline solution (presence of Presence of this common ion will shift the
OH– ions), it will react and reduce the position of equilibrium in (1) to the left
[H+] in the solution. i.e. less HCN molecules dissociate. Thus;
HS–(aq)  H+(aq) + S2–(aq) the % of HCN molecules that ionised is
decreased.
Answers to A Level H2 Topical Chemistry A27
© Singapore Asia Publishers Pte Ltd Topic 7.2

H2 Topical Chemistry 2014.indb 27 3/27/2014 1:57:23 PM


2. ������������������������������
(a) Let the lactic acid be HA.
4. �������
(a) HA  H+ + A– (where HA = Malonic acid)
HA  H+ + A–
From the above equation, [H+] = [A–] and
[H+][A–]
________
Ka = ​  [HA] ​    since the degree of dissociation is very small,
[HA]eqm = [HA]initial.
Since the degree of dissociation for weak
acid is very small, [HA]eqm = [HA]initial. From [H+][A–]
_______
∴ Ka = ​  [HA] ​  
  pKa = 2.85
the dissociation equation, [H+] = [A–].
∴ Ka = 10–2.85
[H+]2
_____ [H+]2
_____
∴ Ka = ​ [HA]   ​  Ka = ​ [HA]   ​ 
_______
[H+]2 = Ka[HA] = (1.38 × 10–4)(0.20) [H+] = √
​ Ka[HA] ​ 
___________
[H+] = 5.25 × 10–3 mol dm–3 =√
​ (10–2.85)(0.10) ​
  
∴ pH = –log10[H+] ___________
= –log10(5.25 × 10–3) =√
​ 1.4125 ×   
10–4 ​
= 2.28 ∴ pH = –log10[H+]
___________
(b) (i) A buffer solution is a solution that resists
= –log10(​√1.4125 ×   
10–4 ​)
any pH change when small amounts of
acid or base is added to it. = 1.925
(ii) Mixture is a solution of acidic buffer.  1.93 (to 3 s.f.)
[salt]
pH = pKa + log10_____
​    ​  (b) Key features of titration curve sketch:
[acid]
A Initial pH = 1.93 at 0 cm3 (as calculated
[salt]
= –log10Ka + log10​ _____   ​  in part (a)).
[acid]
= –log10(1.38 × 10–4) + log10​ ____
0.20 (  )
​ 0.30 ​  ​
B pH at MBC1 = pK1 = 2.85 at 5 cm3
__
1
(​V​MBC ​= ​ 2 ​V
  Equivalence)
= 4.04 1

[HO2CCH2CO2–] = [HO2CCH2CO2H]
(iii)
CH3CH(OH)COO– + H+ → CH3CH(OH)COOH C pH at MBC2 = pK2 = 5.70 at 15 cm3
(from acidic buffer) (from HCl) [HO2CCH2CO2–] = [–O2CCH2CO2–]
[gluconic acid]
_____________ D 1st equivalence point at 10 cm3
3. �
KC = ​  [GDL][H  
  
O] ​ 2 E 2nd equivalence point at 20 cm3
_____
Mass
No. of moles of GDL = ​  M  ​ 
  pH
r
14
1.00 g
______
= ​  178 ​ 
  12

= 5.62 × 10–3 mol 10

_____________
5.62 × 10 mol –3
[GDL] = ​    
    ​ 8


_____
50
​ ​ 1000
  ​  ​dm3 ) 6 MBC2

= 0.112 mol dm–3 4


5.70

2.85
GDL + H2O  gluconic acid
2 MBC1
Initial/mol dm –3 : 0.112 55.5 0 1.93
Equilibrium/mol dm–3 : 0.112 – 0.0670 55.5 0.0670 Volume of
0 10 20 30
= 0.0450 NaOH/cm3

[gluconic acid] ______


0.0080
_____________ 5. �����������
(a) (i) [Ca2+] = ​  40.1 ​ 
 
∴ Kc = ​  [GDL][H  
  
O] ​ 2 = 2.00 × 10–4 mol dm–3
(0.0670)
____________ ______
0.0049
= ​       ​ [Mg2+] = ​  24.3 ​ 
 
(0.0450)(55.5)
= 2.00 × 10–4 mol dm–3
= 0.0268 dm3 mol–1
Answers to A Level H2 Topical Chemistry A28
© Singapore Asia Publishers Pte Ltd Topic 7.2

H2 Topical Chemistry 2014.indb 28 3/27/2014 1:57:24 PM


______
0.0071 Thus, CaCO3 solid is precipitated out
[Cl–] = ​  33.5 ​ 
  first.
= 2.00 × 10–4 mol dm–3 (iii) Rocks of hills and mountains are
__________________
0.0366 probably made up of calcium carbonate
[HCO3–] = ​        ​
[1.0 + 12.0 + 3(16.0)] and magnesium carbonate.
= 6.00 × 10 mol dm–3
–4
Carbon dioxide from the air is dissolved
Formulae of the salts and their relative inside rainwater and the rainwater
amounts: comes into contact with CO32– ions:
CaCl2 : MgCl2 : Ca(HCO3)2 : Mg(HCO3)2 CO2(g) + H2O(l) + CO32– (aq)  2HCO3–(aq)
1 : 1 : 3 : 3
When rainwater percolates through the
Tips rock, carbon dioxide reacts with some of
Ionic Equilibrium–Revision–Extra Question: the dissolved CO32– ions from the partial
______
0.0080
[Ca2+] = ​  40.1 ​   
= 2.00 × 10–4 mol dm–3 dissolution of calcium carbonate and
magnesium carbonate to form HCO3– as
______
0.0049 present in the mineral water.
[Mg2+] = ​  24.3 ​   = 2.00 × 10–4 mol dm–3
______
0.0071 This will cause a decrease in [CO32–]
[Cl –] = ​  35.5 ​ = 2.00 × 10–4 mol dm–3
 
and thus shifts the position of the
__________________
0.0366 equilibrium below to the RHS:
[HCO3–] = ​        ​= 6.00 × 10–4 mol dm–3
[1.0 + 12.0 +3(16.0)]
\ Relative amounts of soluble salts is: XCO3(s)  X2+(aq) + CO32–(aq) (where X= Ca or Mg)
CaCl2 : MgCl2 : Ca(HCO3)2 : Mg(HCO3)2 This causes more metal carbonate to
1 : 1 : 3 : 3 dissolve which produces the metal
Solution: cations of Mg2+(aq) and Ca2+(aq) ions
CaCl2 : MgCl2 : Ca(HCO3)2 : Mg(HCO3)2 in the mineral water.
1 : 1 : 3 : 3 (b) (i)
1 Ca2+ 1 Mg2+ 3 Ca2+ 3 Mg2+
6Ca5(PO4)3F + SiO2 + H2O → SiF4 + 2HF + 3CaO + 9Ca3(PO4)2
2 Cl – 2 Cl – 6 HCO3– 6 HCO3–
∴ a = 1
\ total available mole of ions are: b = 1
4 Ca2+ 2 Ca2+ c = 1
4 Mg2+ 2 Mg2+ same as the molar
4 Cl
– ÷2
2 Cl– concentration of ions d = 2
we have calculated e = 3
12 HCO3– 6 HCO3–
f = 9
Comments:
– Both chlorides and hydrogencarbonates of magnesium (ii) Ksp = [Ca2+]3[PO43–]2 units: mol5 dm–15
and calcium had to be present. Besides the above 4 salts,
MgCl(HCO3) was also accepted as one of the salts. Tips
– Salts such as MgCO3 and CaCO3 should not be present. Ca3(PO4)2  3Ca2+ + 2PO43–
– Relative amounts of salts need to fit with the ionic ratios
calculated. (iii) Ca3(PO4)2  3Ca2+ + 2PO43–
x 3x 2x
(ii) Calcium carbonate.
Since CaCO3 is more insoluble than ∴ Ksp = [Ca ] [PO4 ]
2+ 3 3– 2

MgCO3, the Ksp value for CaCO3 is 1 × 10–26 = (3x)3(2x)2


smaller than that for MgCO3. 1 × 10–26 = 108x5
When the sample of mineral water ∴ x = 2.47 × 10–6
was partially evaporated, [HCO3–] Hence, [Ca2+] = 3x = 3(2.47 × 10–6)
increased. This caused the position = 7.42 × 10–6 mol dm–3
of the equilibrium CO2(g) + H2O(l) +
CO32–(aq)  2HCO3–(aq) to shift to the
LHS, forming more CO32–(aq).
The increased [CO32–] caused the ionic
product [Ca2+] [CO32–] to increase to a
value more than the Ksp(CaCO3) first,
with the ionic product [Mg2+] [CO32–]
remaining less than the Ksp(MgCO3).

Answers to A Level H2 Topical Chemistry A29


© Singapore Asia Publishers Pte Ltd Topic 7.2

H2 Topical Chemistry 2014.indb 29 3/27/2014 1:57:24 PM


Topic 8 Reaction Kinetics ____ k(0.006)(0.006)y
1.44 ______________
/; ​ 1.00 ​  = ​       
 ​
k(0.005)(0.005)y


PAPER 1
Section A 1.44 = 1.2​​ ​ 0.005 ​  ​​​
0.006 y
_____
)
1. D
The graph suggests an autocatalysed reaction. (  )
0.006 y
_____
1.2 = ​​ ​ 0.005 ​  ​​​

[MnO4–]
very slow (no catalyst) __
6
(  ) (  )
6 y
__
​ ​ 5 ​   ​ = ​​ ​ 5 ​   ​​​

fast (catalyst present) y = 1


∴ Reaction is 1st order with respect to [SO2].
slow (reactants are 6. B
used up) Since we measure the time taken for the same
0
0 time volume of nitrogen produced from a range of
diazonium cation concentrations, we will use
2. D ____
1
[C6H5N2+] against Rate (i.e. Rate = ​     ​ 
) to
8 days __1 8 days __1 8 days __1 1
___ time
1 ​  1  ​ ​  2  ​ ​  3  ​ ​  10  ​ 
determine the order of reaction with respect to
2 2 2 2
8 days 8(2) days 8(3) days 8(10) days diazonium cation.
7. C
Tips
Let the amount of uranium in the rock sample be
Since half-life, t1/2 of iodine–131 is 8 days, 80 days 
10t1/2 have lapsed.
x.
4.5  109 years __
1 4.5  10 years __
9
1
x ​ 2 ​  x ​ 4 ​  x
3. C __
1
The flattening of the curve shows that the rate Therefore, after 9.0  109 years, only ​ 4  ​ x of
of reaction is constant. Any increase in [ethanal] __
3
uranium is left. This means that ​ 4 ​  x has been
has no effect on the rate of reaction. decayed into lead. Hence, ratio of uramium : lead
At the point of flattening, all the active sites in
the enzyme is saturated with the ethanal substrate is 1 : 3 after 9.0  109 years.
molecules. 8. B
4. D Half-life is constant since it is a first order
A catalyst is a substance that increases the rate reaction. Hence, the % decay would be constant
of reaction by providing an alternative pathway at a certain period of time and independent of the
with lower activation energy, Ea. Hence, the rate original concentration of hydrogen peroxide.
constant increases. 9. A
Enthalpy change of the reaction, ∆H, remains The homogenous catalyst NO catalyses the
the same. reaction by providing an alternative pathway with
5. B a lower activation energy via the intermediate
When [SO2] are kept constant and [NO2] increases NO2. The catalysed reaction takes place in two
_____
0.008 steps and thus we will expect two “humps” in
by ​ 0.006 ​ = 1.33 times, the relative rate is also the reaction pathway diagram.
____
1.92
increased by ​ 1.44 ​ = 1.33 times
Common Mistakes
∴ Reaction is 1st Order with respect to [NO2]. Homogenous catalyst only speeds up the rate of reaction
by providing an alternative pathway with lower activation
Let rate = k[NO2][SO2]y energy. The energy level of the product(s) does not change.
From Expt. 1: 1.00 = k(0.005)(0.005)y – Thus, B is not the answer.
From Expt. 2: 1.44 = k(0.006)(0.006)y –

Answers to A Level H2 Topical Chemistry A30


© Singapore Asia Publishers Pte Ltd Topic 8

H2 Topical Chemistry 2014.indb 30 3/27/2014 1:57:25 PM


Section B 13. B
10. C 1 – The formation of HBr increases [HBr].
1– If x = 0 Rate = k [P] According to the rate equation for H2(g)/
__________
mol dm–3 s–1 –1 Br2(g) reaction, [HBr] is the denominator.
Unit of k = ​   ​ 
 
=s
mol dm–3 An increase in [HBr] will decrease the rate
∴ y = 0, z = 0
of the forward reaction.
2– If x = 1 Rate = k [P][Q] 2 – For the H2(g)/I2(g) reaction, the rate equation
Unit of k = __________
​ mol dm –3s 2 ​ 
–3 –1
suggests that one molecule of H2(g) and one
(mol dm ) molecule of I2(g) are involved in the slow
= mol–1 dm3 s–1 step. Since the slow step coincides with the
∴ y = –1, z = +3 overall stoichiometric balanced equation, the
3– If x = 2 Rate = k [P][Q]2 H2(g)/I2(g) reaction could be a single step
reaction.
Unit of k = __________
​ mol dm –3s 3 ​ 
–3 –1

(mol dm ) 3 – The mechanism, whether it involves free
= mol–2 dm6 s–1 radicals, cannot be determined from the rate
∴ y = –2, z = +6 equation.
11. B 14. B
Comparing Expt 1 and 2: 1 – A zero order kinetics proceeds at a constant
[I–] and [H+] are constant. rate, independent of [P] and hence does not
When [H2O2] triples, the initial rate is also vary with time.
tripled. 2&3 – Rate constant, k, is dependent on
∴ 1st order w.r.t. H2O2. temperature and presence of catalyst. It
Comparing Expt 2 and 3: does not vary with time.
[H2O2] and [H+] are constant.
15. A
When [I–] doubles, the initial rate is also
Comparing Expt. 1 and 2:
doubled.
[HS–] is kept constant:
∴ 1st order w.r.t. I–.
When [1–bromobutane] is doubled, initial rate
Comparing Expt 3 and 4:
is also doubled
[H2O2] and [I–] are constant.
∴ Reaction is 1st order w.r.t. 1–bromobutane.
When [H+] doubles, the initial rate remains
unchanged. Comparing Expt. 4 and 5:
[1–bromobutane] is kept constant:
∴ Zero order w.r.t. H+. ___
0.3
When [HS–] increases by ​ 0.2 ​ = 1.5 times, initial
Thus, rate equation, rate = k[H2O2][I–]
_________
rate ________
4.5  10–5
Rate constant, k = ​ [H O ][I
 –] ​  rate also increases by ​   
 ​= 1.5 times.
2 2 3.0  10–5
∴ Reaction is 1st order w.r.t. HS–.
Using information from Expt 1,
Both 1-bromobutane and HS– are involved in the
____________
2 × 10–6
∴ k = ​       ​ rate-determining step.
(0.010)(0.010)
= 0.02 dm3 mol–1 s–1 Rate = k[1-bromobutane][HS–]
Using data from Expt. 1:
12. B Rate constant, k
1 & 2 – Time taken for [•OH] to halved is ___________________
Rate
= ​        ​
2.0 × 10–4 s. Since it has a constant [1–bromobutane][HS–]
half-life, it is first order with respect to ________________________
1.5  10–5 mol dm–3s–1
[•OH]. = ​         ​
(0.1 mol dm–3)(0.1 mol dm–3)
2.0 × 10–4 s 2.0 × 10–4 s = 1.5  10–3 mol–1 dm3s–1
[•OH] = 10.0 [•OH] = 5.0 [•OH] = 2.5

3 – There is not enough information to deduce


the overall reaction is second order.

Answers to A Level H2 Topical Chemistry A31


© Singapore Asia Publishers Pte Ltd Topic 8

H2 Topical Chemistry 2014.indb 31 3/27/2014 1:57:25 PM


PAPER 2
1. ���������������������������������
(a) NO causes photochemical smog. Tips
(b) Plot graph of initial rate against (PNO)2. H2O2 + 2H+ + 2e–  2H2O Eθ = +1.77V [R]
I2 + 2e–  2I– Eθ = +0.54V [O]
200
H2O2 + 2H+ + 2I– → I2 + 2H2O
​Eθcell
​  ​ =
​  ​EθR​ ​​ – ​EθO​ ​​ 
160 = (+1.77) – (+0.54)
= +1.23V
Since ​E​θcell   ​ > 0,

120
∴ reaction is feasible.
initial rate / Nm–2 s–1

(b) The ethanoic acid/sodium ethanoate in the


reaction mixture is added to buffer the pH.
80
(c) Preparation of 1.5 mol dm–3 H2O2 stock
solution
40 ① Using 200 cm3 pipette, transfer 100 cm3
carefully into a 1000 cm3 volumetric
flask.
0 ② Top up the volumetric flask to the mark
0 0.2 0.4 0.6 0.8 1.0
(PNO)2 / N2 m–4 with deionised water.
③ Stopper, shake well and invert upside
Tips down a few times to obtain 1000 cm3
Graph of initial rate against PNO or (PNO)2 is acceptable. The of 1.5 mol dm–3 H2O2 stock solution.
graph must pass through the origin.
Tips
(c) Since the graph is a straight line, Dilution Formula: C1V1 = C2V2
rate  (PNO)2 whereby: C1 = Concentration of solution before dilution,
∴ Order of reaction w.r.t. PNO is Second Order. mol dm–3

C2 = Concentration of solution after dilution,
(d) When ​PO​ ​is halved, the gradient at each point mol dm–3
2

of the graph is also halved  indicates that V1 = Initial volume, cm3


rate is halved V2 = Final volume, cm3
∴ Order of reaction w.r.t. ​P​O ​is First Order. C1V1 = C2V2
2
(7.5 mol dm–3) V1 = (1.5 mol dm–3)(1000 cm3)
(e) (i) rate = k(PNO)2(​P​O ​)
2
∴ V1 = 200 cm3

Common Mistakes
For the rate equation, remember to represent them in terms Preparation of series of solution Bs with
of partial pressures rather than concentrations. different concentration of H2O2
① 3 different solutions (B1, B2 and B3) with
_____________________
units of rate different concentrations of H2O2 were
(ii) Units of k = ​        ​
(units of partial pressure)3
prepared, by keeping all other conditions
_______
Nm–2 s–1 constant.
= ​   ​ 
(Nm–2)3 ② The same proportions (i.e. 500 cm3)
= N–2 m4 s–1 of 1.0 mol dm–3 aqueous ethanoic acid
2. Planning (P) were added into their respective 1000
(a) Ionic Equation: cm3 volumetric flasks, labelled B1, B2
H2O2 + 2H+ + 2I– → 2H2O + I2 and B3.
③ Respective amounts of aqueous H2O2
were then carefully pipetted from the 1.5
mol dm–3 H2O2 stock solution prepared
earlier, into the respective B1, B2 and B3
flasks.

Answers to A Level H2 Topical Chemistry A32


© Singapore Asia Publishers Pte Ltd Topic 8

H2 Topical Chemistry 2014.indb 32 3/27/2014 1:57:26 PM


④ For flasks B1 and B2, deionised water PAPER 3
was added accordingly to the 1000 cm3 1. (a) Order of reaction is the power to which the
mark of the volumetric flask. concentration term of a reactant is raised to
⑤ All 3 flasks were stoppered, shaken in a rate equation.
well and inverted upside down to form Example:
homogenous working solutions of In the reaction, aA + bB → products
different H2O2 solutions of 1000 cm3 Rate equation is rate = k[A]m[B]n
total volume. where m = order of reaction with respect to A
Tips n = order of reaction with respect to B
Deionised water must be added accordingly to keep the total m + n = overall order of reaction
reaction volume constant. This is to ensure that the initial (b) Compare Expt 1 and 2:
concentration of each reagent after mixing is proportional [HCN] and [NaCN] are constant.
to the volume of the reagent used.
When [(CH 3 ) 2 CO] increases from
Experimental details: 0 . 4 m o l d m –3 t o 0 . 5 m o l d m –3
① 50 cm3 of solution A was carefully (1.25 times), initial rate of reaction also
transferred into a conical flask. increases by a factor of 1.25 times.
② 50 cm3 of solution B1 was carefully ∴ Reaction is 1st order w.r.t. (CH3)2CO.
transferred into the conical flask and
stopwatch was started immediately. Compare Expt 1 and 3:
③ Stop the stopwatch immediately upon [(CH3)2CO] and [HCN] constant.
the appearance of dark blue colouration When [NaCN] decreases from
and record the time taken for the dark
0.008 mol dm–3 to 0.006 mol dm–3 (0.75
blue to appear, in a table as shown
times), initial rate of reaction also decreases
below.
④ Repeat steps 1 – 3 for solutions B2 and B3, by a factor of 0.75 times.
and record the time taken accordingly. ∴ Reaction is 1st order w.r.t. NaCN.
Vol. of Conc.of Compare Expt 2 and 4:
Expt.
No.
CH3COOH/
Vol. of Vol. of
H2O2/cm3 H2O/cm3
H2O2/ Time/s (​  ____1
​ Time ​ )​/s–1
    
cm3 mol dm–3 Let the rate equation be rate =
B1 500 150 350 0.225 k[(CH3)2CO][NaCN][HCN]n
B2 500 300 200 0.450 1.25 = k(0.050)(0.008)(0.040)n – from Expt 2
B3 500 500 0 0.750 0.94 = k(0.050)(0.006)(0.050)n – from Expt 4
1.25 k(0.050)(0.008)(0.040)
n
Graphical Method: ____
​ 0.94 ​ = ____________________
​    n ​
____
1 k(0.050)(0.006)(0.050)
Plot [H2O2] against ​     ​ (which is the rate),
time
1.33 = 1.33 × (0.8)n
as initial rate of the reaction is inversely
(0.8)n = 1
proportional to the time taken for a fixed
∴n=0
amount of iodine to be formed.
____
1 ∴ Reaction is zero order w.r.t. HCN.
i.e., rate α ​     ​ α [H2O2]. Hence, Rate equation:
time
We should have a linear graph which Rate = k[(CH3)2CO][NaCN]
suggests that the rate of reaction would be (c) Mechanism: Nucleophilic addition with
1st order with respect to [H2O2]. CN– as nucleophile
[H2O2]/mol dm–3 HCN is a weak acid (dissociates partially)
and thus is a poor source of CN–. A small
amount of NaCN is added to accelerate the
reaction.

( Time )
1
______
​ ​     ​  ​/s–1

Answers to A Level H2 Topical Chemistry A33


© Singapore Asia Publishers Pte Ltd Topic 8

H2 Topical Chemistry 2014.indb 33 3/27/2014 1:57:26 PM


O–
CH3 δ+ δ– slow Tips
rate-determining
C=O CH3 – C – CH3
··C​​  N –
step
​  1   ​ (units: s–1)
Rate = ____
(from CH3
time
CN
NaCN)

fast + HCN Compare Expt. 2 and 4:


– [I–] and [H+] are kept constant.
OH – When [H2O2] doubles, the initial rate is
CH3 – C – CH3 + CN–
also doubled.
∴ Reaction is first order w.r.t. H2O2.
CN
Compare Expt. 2 and 3:
Rate equation shows that the slow step,
– [H2O2] and [H+] are kept constant.
which is the rate determing step, only 1
– When [I–] increases by 1.5 times, the initial
mole of (CH3)2CO and 1 mole of NaCN are
rate also increases by 1.5 times.
involved.
∴ Reaction is first order w.r.t. I– (or KI).
(d) O O
Let the rate equation be:
C C rate = k[H2O2][I–][H+]x.
H CH2CH3 CH3 CH3
Data from Expt. 1:
propanal propanone
0.0303 = k(15)(10)(5)x ––
There is only one bulky alkyl group
(CH3CH2–) bonded to the carbonyl carbon Data from Expt. 2:
in propanal, as the other group is a small 0.0100 = k(5)(10)(10)x ––
hydrogen atom. Thus, there is less steric ______ k(15)(10)(5)x
0.0303 ___________
hindrance for :CN– to attack the carbonyl / ​ 0.0100 ​  = ​       
k(5)(10)(10)x
carbon, as compared to two bulky alkyl  ​ ∴ x = 0
groups (CH3–) in propanone. ∴ Reaction is zero order w.r.t. H+ (or HCl).
(e) HCN is the reagent and NaCN is the (c) rate = k[H2O2][I–]
catalyst. units of rate constant,
_____________
units of rate
2. Reaction 1: k = ​       ​
(units of conc.)2
N2(g) + 3H2(g)  2NH3(g) __________
s–1
= ​     ​ 
In the Haber process, finely divided ion is used (mol dm–3)2
as a heterogenous catalyst. = dm mol–2 s–1
6

Reaction 2: Tips
2I–(aq) + S2O82–(aq) → I2(aq) + 2SO42–(aq) Alternative Answer:
In the oxidation of iodide ion, I– by thiosulfate, dm3 mol–1 s–1 is also accepted since it is the units for usual
S2O82–, aqueous iron(III) ions, Fe3+(aq) is used as overall second-order reaction.
the homogenous catalyst.
____
1
3. �����������������
(a) Initial Rate  ​     ​ 
time
The faster the reaction, the shorter is the
time taken for the blue colour to appear.
(b) Expt. No. Time taken / s Rate / s–1
1 33 0.0303
2 100 0.0100
3 67 0.0149
4 50 0.0200

Answers to A Level H2 Topical Chemistry A34


© Singapore Asia Publishers Pte Ltd Topic 8

H2 Topical Chemistry 2014.indb 34 3/27/2014 1:57:26 PM


5. B
Topic 9.1 The Periodic Table:
The water vapour produced when Al(OH)3
Chemical Periodicity decomposes at high temperature helps to delay
PAPER 1 the spread of flames in the event of a fire.
Section A 2Al(OH)3 → ∆
Al2O3 + 3H2O
1. C
6. B
0.3 mol of the chloride give 0.6 mol of HCl.
Across a period, atomic radius decreases as proton
∴ 1 mol of chloride will give 2 mol of HCl. number increases. This is due to the increase
�� PCl5 undergo complete hydrolysis in water to in nuclear charge and weak screening effect
give strongly acidic solution (pH  2). as successive electrons are added to the same
Stage 1 : PCl5 + H2O → POCl3 + 2HCl quantum shell.
Stage 2 : POCl3 + 3H2O → H3PO4 + 3HCl Down a group, atomic radius increases as each
Overall : PCl5 + 4H2O → H3PO4 + 5HCl successive member contains an extra shell
of electrons. The additional screening effect
(with excess water)
outweighs the effect of increased nuclear charge.
Thus, the outer electrons are lees strongly attracted
Tips
by the nucleus and are further away.
PCl5 readily hydrolyses in water and thus, the overall
equation should be used. In this question, option C is the 7. B
only possible answer though and students should learn
to choose the best answer in a MCQ question. In other
SiCl4 does not hydrolyse in water to form Si4+
context, students are encouraged to write the overall ions; therefore is eliminated.
equation instead. For the other three aqueous compounds, Al3+
cations in AlCl3(aq) has the highest charge-density
2. C and thus is most effective in precipitating finely
Across the third period, the melting points of the divided solid particles.
elements do not decrease steadily but varies.
In general; the melting points increases from Na 8. C
to Si and then decreases to Ar. In the third period of the Periodic Table,
A – Silicon has the highest melting point, not
3. B phosphorus
MgCl2 is an ionic chloride which dissolves in B – Phosphorus forms two acidic oxides ( P4O6
water with slight hydrolysis to give a slightly and P4O10).
acidic solution of pH  6.5. Sulfur also forms two acidic oxides (SO2
4. D and SO3)
No. of moles of AgNO3 C – Molecules present are P4, S8 and Cl2.
_____
100 D – Chlorides of period 3 elements from Al to
= ​ 1000  ​ dm3 × 0.30 mol dm–3
S react with water to give acidic solutions.
= 0.03 mol
= No. of moles of Ag+ 9. B
A – Na2O(s) + H2O(l) → 2NaOH(aq)
Since Ag+(aq) + Cl–(aq) → AgCl(s);
B – SiO2 does not react with water and thus
No. of moles of Cl– = 0.03 mol
H2SiO3 is not a product of the reaction
that has reacted with Ag+
between an oxide of a third period element
n (chloride of R) : n (chloride ion produced by R and water.
chloride solution) C – P4O10(s) + 6H2O(l) → 4H3PO4(aq)
0.010 mol : 0.030 mol
D – SO3(g) + H2O(l) → H2SO4(aq)
1 mol : 3 mol
Both option B(III) and option D(V) are possible 10. B
answers. A – Sulfur exists as a S8 molecule.
AlCl3 is a solid at rtp and thus element R cannot B – Na, Mg and Al are all good electrical
be Group III element. conductors. Al has the highest electrical
It should be a Group V element, Phosphorus, conductivity since the number of delocalised
since PCl3 is a liquid. electrons in the metallic structure is greatest.

Answers to A Level H2 Topical Chemistry A35


© Singapore Asia Publishers Pte Ltd Topic 9.1

H2 Topical Chemistry 2014.indb 35 3/27/2014 1:57:27 PM


C – Silicon has the highest melting point in the PAPER 2
third period. 1. ��������������������������������������������������
(a) (i) Conducts electricity in solid and molten
D – Chlorine has the smallest anion in the third state.
period. (ii) Change:
Increase in boiling point.
11. A
Explanation:
Silicon has a very high melting point due to its
Halogens have simple molecular
giant covalent structure. A large amount of
structures consisting of diatomic
energy is required to break the strong covalent
covalent molecules held together by
bonds in Si.
van der Waals’ forces.
Al has high melting point due to the metallic
As we go down the group from chlorine
bonds between the positive metal ions and the
to iodine, the molecules become larger
'sea of delocalised valence electrons'.
and the strength of intermolecular
S (exists as S8) has higher melting point than P
van der Waals’ forces increases as the
(exists as P4) due to its bigger size molecules
number of electrons in the molecules
(more electrons in molecule) which increases
increases. More energy is required to
the strength of weak van der Waals’ forces.
overcome the stronger forces and thus
boiling point increases.
Section B (b) (i) Giant covalent structure.
12. D (ii) From carbon to germanium, atomic
• Before forming cation, the electronic size increases and there will be lesser
configuration of Group II element is degree of overlap between the atomic
1s22s22p63s23p63d104s24p65s2 orbitals. This results in the increase in
(38 protons & 38 electrons) bond length and thus weaker covalent
After forming cation, the particle has 38 bond.
protons attracting 36 electrons. The melting point decreases since less
• Before forming anion, the electronic energy is required to break the weaker
configuration of Group VII element is covalent bonds.
1s22s22p63s23p63d104s24p5 (c) (i) SiCl4 + 4H2O → SiO2 • 2H2O + 4HCl
(35 protons & 35 electrons) (ii) From the Data Booklet,
After forming anion, the particle has 35 C–Cl: 340 kJ mol–1 Si–Cl:359 kJmol–1
protons attracting 36 electrons. (iii) Carbon is in Period 2 and cannot expand
∴ The smaller size of the cation is due to the its octet to accommodate lone pair of
greater nuclear charge which results in the same electrons from H2O.
number of electrons being attracted more strongly
by the positively charged nucleus. PAPER 3
1. ����
(a)
13. C Ar

1 – Chloride ions, being the conjugate base of


First Ionisation Energy

P Cl
a strong acid (weak conjugate base), does S
Mg Si
not react with water. Al
2 & 3 – AlCl3 dissolves in water with hydrolysis Na

to give an acidic solution (pH  3). proton number


11 12 13 14 15 16 17 18
AlCl3(s) + aq → [Al(H2O)6)] (aq) + 3Cl (aq)
3+ –

[Al(H2O)6]3+ → [Al(H2O)5(OH)]2+ + H+(aq) There is an increasing trend of first ionisation


energy from Na to Ar due to the increase in nuclear
The acidic nature of the solution is due to the charge and a decrease in atomic size, while the
relatively small but highly charged Al3+ ions shielding effect remains almost the same since the
which polarises the surrounding water molecules, electrons all go into the same electron shell.
weakens the O–H bond and causes it to break and
produce H+(aq) ions.

Answers to A Level H2 Topical Chemistry A36


© Singapore Asia Publishers Pte Ltd Topic 9.1

H2 Topical Chemistry 2014.indb 36 3/27/2014 1:57:27 PM


There are some exceptions in the increasing (b) • 2AlCl3(s) + 3H2O(l) → Al2O3(s) + 6HCl(g)
trend: With a few drops of water, steamy white
 First ionisation energy of Al is lower than fumes of HCl(g) are evolved and a white
Mg. solid Al2O3(s) remains.
Mg[Ne]3s2 Al[Ne]3s23p1 • AlCl3(s) + 6H2O(l) →
Less energy is required to remove a 3p [Al(H2O)6]3+(aq) + 3Cl–(aq)
electron in Al than a 3s electron in Mg, In large quantity of water, hydrated ions are
since the 3p electron is further away from formed.
the nucleus and experiences some slight [Al(H2O)6]3+(aq) + H2O(l) 
shielding from the 3s electrons. [Al(H2O)5OH]2+(aq) + H3O+(aq)
 First ionisation energy of s is lower than p.
Hydrated ions undergo hydrolysis in water
P [Ne] 3s2 3px1 3py1 3pz1
to produce a weakly acidic solution.
S [Ne] 3s2 3px2 3py1 3pz1
(c) When Na2O(s) is added to a solution of
Less energy is required to remove an electron
litmus in water, a blue colour solution is
from the paired 3px electrons in S due to
observed as Na2O(s) dissolves in water to
inter-electron repulsion.
(b) Add dilute hydrochloric acid to the sample give aqueous NaOH which is an alkaline
of white powder: solution (pH  13).
If there is no reaction, the sample will be Na2O(s) + H2O(l) → 2NaOH(aq)
SiO2. Litmus solution will turn to blue.
If the white powder reacts with HCl to give (d) (i) Ionic Bonding exists in Mg3N2.
a colourless solution, it is either MgO or 2+ 3–
Al2O3. 3 Mg 2 N
MgO(s) + 2HCl(aq) → MgCl2(aq) + H2O(l)
Al2O3(s) + 6HCl(aq) → 2AlCl3(aq) + 3H2O(l) (ii) Mg3N2(s) + 6H2O(l) →
To the colourless solution, dilute NaOH(aq) 2NH3(g) + 3Mg(OH)2(s)
alkaline gas white insoluble solid
is added dropwise until in excess.
If a white precipitate is formed and does not Mr of Mg3N2 = 100.9
dissolve in excess NaOH(aq); the sample Mr of Mg(OH)2 = 58.3
will be MgO. _____
Mass
Mole of Mg3N2 = ​  M  ​   
Mg2+(aq) + 2OH–(aq) → Mg(OH)2(s) r

If a white precipitate is formed and dissolves _____


2.0
= ​ 100.9  ​ 
mole
in excess NaOH(aq), the sample will be
From the balance chemical equation:
Al2O3.
1 mol of Mg3N2 produce 3 mol of
Al3+(aq)+ 3OH–(aq) → Al(OH)3(s)
Mg(OH)2.
Al(OH)3(s) + OH–(aq) → Al(OH)4–(aq)
_____
2.0 _____
2.0
(c) NaCl is an ionic chloride and dissolves in ∴ ​ 100.9  ​ 
mol of Mg3N2 produce 3 × ​ 100.9  ​ 
water without hydrolysis, giving a neutral mol of Mg(OH)2.
solution (pH = 7).
NaCl(s) + aq → Na+(aq) + Cl–(aq) Hence, Mass of Mg(OH)2 solid
PCl5 is an acidic chloride (simple covalent = Mole × Mr
molecule) and undergoes hydrolysis in water
to give a strongly acidic solution (pH = 1)

______
3 × 2.0
)
= ​ ​  100.9 ​  ​× 58.3
= 3.47 g
PCl5 + 4H2O → H3PO4 + 5HCl
3. �����������������������������������������������
(a) Magnesium burns in oxygen with a brilliant
2. �������������������������
(a) Molecular formula = Al2Cl6
white flame. A white solid residue is formed.
Cl Cl Cl
__
1
Mg(s) + ​ 2 ​ O2(g) → MgO(s)
Al Al Phosphorus burns in oxygen with a pale
Cl Cl Cl bluish-green flame. P4O6 or P4O10 (a white
Al2Cl6 dimer solid) is formed.

Answers to A Level H2 Topical Chemistry A37


© Singapore Asia Publishers Pte Ltd Topic 9.1

H2 Topical Chemistry 2014.indb 37 3/27/2014 1:57:27 PM


P4(s) + 3O2(g) → P4O6(s) P4O10 is an acidic oxide and it reacts readily with
P4(s) + 5O2(g) → P4O10(s) alkalis to give the salt and water. It does not react
with acids.
Sulfur burns with a blue flame to give SO2(g),
P4O10(s) + 12NaOH(aq) → 4Na3PO4(aq) + 6H2O(l )
which is further oxidised to SO3(g) in excess
Al2O3 is an amphoteric oxide and it reacts with
oxygen. both acids (like HCl) and alkalis (like NaOH).
S(s) + O2(g) → SO2(g) Al2O3(s) + 6HCl(aq) → 2AlCl3(aq) + 3H2O(l )
2SO2(g) + O2(g) → 2SO3(g) Al2O3(s) + 2NaOH(aq) + 3H2O(l ) →
(b) P4O6 and P4O10 dissolves readily in water to 2NaAl(OH)4(aq)
give acidic solutions, which turns universal Al2O3 is ionic with a marked degree of covalent
indicator red. character due to the oxide ions being polarised
P4O6(s) + 6H2O(l) → 4H3PO3(aq) Phosphorus Acid by the small but highly charged Al3+ ions. This
P4O10(s) + 6H2O(l) → 4H3PO4(aq) Phosphoric Acid
intermediate (ionic/covalent) bonding in Al2O3
causes its amphoteric character.
SO2 and SO3 dissolves readily in water to
give acidic solutions, which turns universal
indicator red.
SO2(g) + H2O(l) → H2SO3(aq) Sulfurous Acid
SO3(g) + H2O(l) → H2SO4(aq) Sulfuric Acid
4. ������
(a) PCl3 can be made by heating chlorine with
white phosphorus (P4), with continual
removal of PCl3 as it is being formed.
P4 + 6Cl2 → 4PCl3
PCl5 can be made by reacting PCl3 further
with excess chlorine, so that PCl3 is oxidised
to PCl5.
PCl3 + Cl2 → PCl5
(b) G: POCl3
Tetrahedral shape with respect to central P
atom.

Tips
Mr of compound G = 31.0 + 16.0 + 3(35.5)
= 153.5 (proven)

Tips
Clue: G is produced when PCl5 is reacted with a small
amount of H2O
i.e. PCl5 + H2O → POCl3 + 2HCl
Hence, G = POCl3

(c) H: SO2
PCl3 + SO3 → POCl3 + SO2
PCl5 + H2O → POCl3 + 2HCl
5. Across Period 3, the nature of the oxides changes
from basic (MgO) through amphoteric (Al2O3)
to acidic (P4O10).
MgO is a basic oxide and it reacts readily with
acids to give the salt and water. It does not react
with alkalis.
MgO(s) + 2HCl(aq) → MgCl2(aq) + H2O(l )

Answers to A Level H2 Topical Chemistry A38


© Singapore Asia Publishers Pte Ltd Topic 9.1

H2 Topical Chemistry 2014.indb 38 3/27/2014 1:57:28 PM


6. A
Topic 9.2 Group II
MgCO3 + 2HCl → MgCl2 + CO2 + H2O
PAPER 1 BaCO3 + 2HCl → BaCl2 + CO2 + H2O
Section A ���������������������������������������
1 & 2 – Total amount of carbonate (BaCO3
1. A plus MgCO3) in the mixture can be
Ba(OH)2 is an alkali and some of it would have determined.
reacted with acidic oxide CO2 in the air via the
3 – From the mass of BaSO4(s) precipitated,
reaction:
the amount of BaCO3 in the mixture can be
Ba(OH)2 + CO2 → BaCO3 + H2O determined and thus, the mole fraction of
As Ba(OH)2 has decreased, less HCl is required MgCO3 in the weighed samples.
to neutralise it during the titration.
7. C
2. C 1 – The low solubility of strontium sulfate makes
__
1
Ca(NO3)2(s) → CaO(s) + 2NO2(g) + ​ 2 ​ O2(g) it difficult for the salt to be absorbed and

incorporated into human metabolism.
8.2 g
_____
No. of mole of Ca(NO3)2 = ​  164 ​  2 – Half-life of 29 years suggests that 90Sr can
= 0.05 mol still cause damage to humans after many
1 mole of Ca(NO3)2 gives 0.5 mol of O2. years.
3 – Strontium being immediately below Ca
______
0.0500
∴ 0.05 mol of Ca(NO3)2 gives ​  2 ​  = 0.025 mol in Group II, readily replaces calcium
of O2. compounds in human metabolism.
Vol. of O2 = Mole of O2 × Molar Volume
= 0.025 mol × 24000 PAPER 3
= 600 cm3 1. �����������������������������������������
(a) • The very low melting point of Be(NO3)2
3. D suggests that it is a covalent compound
A – Ba reacts with water to form Ba(OH)2 and H2 with simple molecular structure.
B – CaO reacts with water to form Ca(OH)2
C – Mg reacts with steam to form MgO and H2 Tips
D – Mg does not react with water High charge density of Be2+ ion cause Be(NO3)2 to have
high covalent character.

Section B
4. D • The very high melting point of Ca(NO3)2
Atomic and ionic radius increases down the group suggests that it is an ionic compound with
(from Ca to Ba) due to successive increase in giant crystal lattice structure.
electron shell. (b) Mg(NO3)2(s) → ∆
__
1
MgO(s) + 2NO2(g) + ​ 2 ​ O2(g)
Tips
2 – Magnitude of the hydration energy of M2+ ion decreases Ca2+ ion is larger than Mg2+ ion and thus
from Ca to Ba. a smaller charge density; which results in
3 – The energy required for the process M(g) → M2+(g) + weaker polarising power towards the NO3–
2e– decreases from Ca to Ba due to increasing atomic
size and increasing shielding effect.
anion. Ca(NO3)2 is thus more stable than
Mg(NO3)2 and requires a higher temperature
5. D to decompose.
1 – Ionic radius of Ca2+ and Sr2+ are 0.099 nm
and 0.113 nm respectively. Sr2+ ions can Tips
replace Ca2+ ions because their ionic size is Thermal decomposition decreases down Group II due to
about the same . decreasing polarisation of the anion (NO3–) by the cation
of increasing size.
2 – Group II hydroxides becomes more soluble
down the group. Hence, Sr(OH)2 is more
soluble than Ca(OH)2 and so will precipitate 2. �����
Mg(NO3)2 • 6H2O(aq) ∆
less easily in the bone structure. __
1
MgO(s) + 2NO2(g) + ​ 2 ​ O2(g) + 6H2O(g)
3 – There are ionic and covalent bonding but
not metallic bonding in hydroxyapatite.

Answers to A Level H2 Topical Chemistry A39


© Singapore Asia Publishers Pte Ltd Topic 9.2

H2 Topical Chemistry 2014.indb 39 3/27/2014 1:57:28 PM


3. �����������������������������������������������
(a) Hold a piece of calcium metal using tongs,
Topic 9.3 Group VII
and heating it in a gas jar filled with oxygen
gas. PAPER 1
Calcium burns with a brick-red flame. Section A
(b) All Group II metal oxides dissolve in water 1. A
to give an alkaline solution (i.e. Group II Iodides react with conc. H2SO4 to produce HI,
hydroxides). Down the group, the solubility which is oxidised by conc. H2SO4 to I2.
of Group II hydroxides increases and more I– + H2SO4 → HI + HSO4–
OH– ions are present in the solution. As such,
pH values of solution increases. 8HI + H2SO4 → 4I2 + H2S + 4H2O
__
5 (black
(c) Ba(O3)2 + H2O → Ba(OH)2 + ​ 2 ​ O2 deposit)
__
1 Chlorides react with conc. H2SO4 to produce HCl,
4.
(a) Mg(NO3)2(s) → ∆ MgO(s) + 2NO2(g) + ​ 2 ​ O2(g) which forms a white precipitate (of AgCl) when
(b) Thermal stability of Group II nitrates bubbled into aqueous silver nitrate
increases down the group. This is because as Cl– + H2SO4 → HCl + HSO4–
the cationic size increases down the group, Ag+(aq) + Cl–(aq) → AgCl(s)
the charge density decreases and hence, the (white precipitate)
polarising power of the cation decreases.
2. C
This results in less polarisation of the
Boiling point of Group VII elements increases
N​O​3–​ ​ ion by the larger cations and the
from Cl2 to I2 due to stronger intermolecular van
decomposition thus becomes more
der Waals' forces between the diatomic molecules
difficult.
as molecular size increases (due to increase in
number of electrons).
Common Mistakes
Thermal decomposition decreases down Group II due to
Electron affinity becomes less negative from
decreasing polarisation of the anion (N​O​–3​)​  by the cation chlorine to iodine due to the increase in atomic
(NOT atoms) of increasing size. size and weaker attraction of the additional
valence electron by the positive nucleus.
3. C
As the size of the halogen increases down the
group, the H–X bond becomes longer and weaker
and so, is easily broken.
Hence, thermal stability of H–X decreases down
the group due to the decreases in strength of the
H–X bond as the size of the halogen increases.
4. C +5
3Cl2(g) + 6NaOH(aq) → 5 NaCl(aq) + NaClO3(aq)
hot aqueous sodium chlorate (v)
NaOH
+ 3H2O(l )

Section B
5. C
X2(aq) + 2e–  2X–(aq) +1.36 V
Y2(aq) + 2e–  2Y–(aq) +1.07 V
Z2(aq) + 2e–  2Z–(aq) +0.54 V
1 A less positive Eθ value suggests a decrease
in the oxidising power in the sequence X2 >
Y2 > Z2
2 ∴ Reducing power increase in the sequence:
X– < Y– < Z–

Answers to A Level H2 Topical Chemistry A40


© Singapore Asia Publishers Pte Ltd Topic 9.2/Topic 9.3

H2 Topical Chemistry 2014.indb 40 3/27/2014 1:57:29 PM


3 2Z–(aq) + Y2(aq) → Z2(aq) + 2Y–(aq) 8. B
Oxidation half-reaction: –1 0
2Z–(aq) → Z2(aq) + 2e– 1 – NaCl → Cl2  in O.S. = 1
Reduction half-reaction: 0 –1
2 – H2 + Cl2 → 2HCl  in O.S. = 1
Y2(aq) + 2e– → 2Y–(aq)
–1 –1
​E​θcell
  ​ =
​  ​EθR​ ​​ – ​EθO​ ​ ​  3 – NaCl + H2SO4 → HCl + NaHSO4
= (+1.07) – (+0.54)  in O.S. = 0
= +0.53V
Since E ​  ​ > 0, ∴ reaction is feasible.
​ θcell PAPER 2
1. (a) Cl2 + 2NaOH → NaCl + NaClO + H2O
6. D
NH3(aq) (b) 3Cl2 + 6NaOH → 5NaCl + NaClO3 + 3H2O
Ag+(aq) + Cl–(aq) → AgCl(s) [Ag(NH3)2]+ + Cl–
white ppt
from from
AgNO3(aq) MgCl2(aq) PAPER 3
Ag+(aq) + Fe2+(aq) → Ag(s) + Fe3+(aq) 1. ������������������������������������������������
(a) Bond energy is the energy required to break
from from
grey ppt one mole of covalent bond between two
AgNO3(aq) FeCl2(aq) atoms in the gaseous state.
1 – Fe2+(aq) is oxidised to Fe3+(aq) which (b) (i) Heating is required.
suggests that Ag+(aq) is acting as an oxidising (ii) The hydrogen halides, HX, is
agent. increasingly easy to decompose as we
2 – Ammonia, NH3(aq) forms a soluble complex go down Group VII.
with Ag+ ion, and not Ag(s) metal .
Tips
Common Mistakes As we go down Group VII, the H–X bond becomes longer
and weaker.
Students tend to make the common mistake that ammonia,
NH3(aq) forms a complex with Ag(s) metal when they look
at the formula of the soluble complex [Ag(NH3)2]+. (iii) 2HCl  H2 + Cl2
Note that Ag+(aq) ions are involved, and not Ag(s) metal. ∆H = (2 × 431) – 436 – 244
= +182 kJ mol–1 (more endothermic)
3 – Fe2+(aq) ions are reactants and not catalyst. 2HI  H2 + I2
7. B ∆H = (2 × 299) – 436 – 151
1 – Mg + Cl2 → MgCl2 = +11 kJ mol–1 (less endothermic)
1 mol of x 1 mol of Cl2 (iv) As we go down Group VII, less energy is
2Na + Cl2 → 2NaCl required for the decomposition reaction.
​  1__2 ​mol of Cl
1 mol of y 2
As the size of halogen increases down
2 – H2 + Cl2 → 2HCl the group, the H–X bond becomes
1 mol of x 1 mol of Cl2
longer and weaker.
2KBr + Cl2 → 2KCl + Br2
1 mol of y ​  1__2 ​mol of Cl 2. ��������������������������������������������
(a) Colour of halogens gets darker down the
group. Chlorine is yellowish-green, bromine
2

cold
3 – 2NaOH  +  Cl2  →  NaCl + NaClO + H2O is reddish brown and iodine is black.
1 mol of x​  2 ​mol of Cl
1 __
2 Volatility of halogens decreases down the
hot group. As the molecule size of halogens
6NaOH + 3Cl2 → 5NaCl + NaClO3 + 3H2O increases (due to increasing number of
1 mol of x​  2 ​mol of Cl2
1 __
electrons in the molecules) there is an
∴ In 1 & 2, the no. of moles of Cl2 that react increasing intermolecular van der Waals'
with 1 mol of x is twice the no. of moles of forces. Chlorine is a gas, bromine is a liquid
Cl2 that react with 1 mol of y. while iodine is a solid at r.t.p.
∴ In 3, the no. of moles of Cl2 that react with (b) H2SO4 + Cl– → HCl + HSO4–
1 mol of x is the same as the no. of moles of When Cl– ion reacts with concentrated
Cl2 that react with 1 mol of y. H2SO4, steamy white fumes of HCl(g) is
produced.
H2SO4 + Br– → HBr(g) + HSO4–

Answers to A Level H2 Topical Chemistry A41


© Singapore Asia Publishers Pte Ltd Topic 9.3

H2 Topical Chemistry 2014.indb 41 3/27/2014 1:57:29 PM


H2SO4 + 2HBr → Br2 + SO2 + 2H2O
Topic 9.4 An Introduction to the
When Br– ion reacts with concentrated
H2SO4, steamy white fumes of HBr(g), Chemistry of
together with some reddish-brown fumes Transition Element
of Br2(g) are produced. Some of the HBr PAPER 1
produced will be oxidised by concentrated Section A
H2SO4 to Br2(g). 1. B
H2SO4 + I– → HI(g) + HSO4– Iron, Fe has 26 electrons while Fe2+ has 24
H2SO4 + 8HI → 4I2 + H2S + 4H2O electrons.
When I– ion reacts with concentrated H2SO4, Electrons are removed from the 4s orbitals first
steamy white fumes of HI(g) is produced, before the 3d orbitals.
together with purple vapour of I2(g). Most Fe : 1s22s22p63s23p63d64s2
–2e–
of the HI produced will be oxidised by Fe2+ : 1s22s22p63s23p63d6
concentrated H2SO4 to I2(g).
2. C
3. (a) H2 reacts rapidly with Cl2 to form HCl. A transition element is a d-block element which
H2 + Cl2 → 2HCl (rapid reaction) forms compounds with its ions having partially
H2 reacts with Br2 only upon heating and in filled d orbitals.
the presence of Pt catalyst, to form HBr .
3. A
H2 + Br2→ 2HBr (slow reaction) Let the oxidation state of titanium ion be x.
(b) H2 reacts violently with F2 to form HF. ∴ x + 2(–2) = 0
H2 + F2 → 2HF (very rapid reaction) x = +4
∴ Charge of titanium ion is 4+.
Tips Electronic Configuration of Ti atom:
Reactivity decreases down Group VII as the H–X bond 1s22s22p63s23p63d24s2
becomes longer and weaker.
Electronic Configuration of Ti4+ ion:
1s22s22p63s23p6
4. 3Cl2(g) + 6NaOH(aq) → 5NaCl(aq) + NaClO3(aq)
Thus, 18 electrons are associated with each
+ 3H2O(l)
titanium ion (Ti4+) in the anatase lattice.
4. C
Transition metal complexes are often coloured
due to the presence of incompletely filled
d-orbitals in the metal ion. In the presence of
ligands, the 3 d-orbitals of the transition metal
become non-degenerative and split into two
groups with slightly different energy (d-d*
splitting). When a d-electron from the lower
energy group is promoted to the higher energy
group (d-d* electron transition), radiation in the
visible region of the electromagnetic spectrum
is absorbed. The light energy not absorbed will
be seen as the colour of the complex.
The colour of complexes depends on the energy
gap between the two groups of d orbitals which
depends on:
① Nature of the metal and its oxidation state.
② Nature of ligands.
[Cu(H2O)6]2+ + 4Cl–(aq)  [CuCl4]2– + 6H2O(l )
blue solution green solution
For both [Cu(H2O)6]2+ and [CuCl4]2– ions, the
central metal ion remains as Cu2+ ion and it would
have the same number of d-electrons around it.

Answers to A Level H2 Topical Chemistry A42


© Singapore Asia Publishers Pte Ltd Topic 9.3/Topic 9.4

H2 Topical Chemistry 2014.indb 42 3/27/2014 1:57:29 PM


ClO2 + 4H+ + 5e– → Cl– + 2H2O +1.50 V
Tips
Co (aq) → Co (aq) + e
2+ 3+ –
–1.82 V
Cu atom ClO2 + 4H+ + 5Co2+ → Cl– + 5Co3+ + 2H2O
3d 4s
​ ​θcell
E   ​  = +1.50 – 1.82 = –0.32 V
Cu ion
2+

3d 4s Since ​Eθcell
​  ​ < 0, reaction is not feasible.
[Cu(H2O)6]2+ : (iii) Cation P is [Co(NH3)6]2+
hybridised orbitals
(iv) [Co(NH3)6]3+ is formed.
3d 4s 4p 4d ClO2 + 4H+ + 5e– → Cl– + 2H2O +1.50 V
to accept 6 lone pairs of electrons
from 6 H2O molecules. [Co(NH3)6]2+ →[Co(NH3)6]3+ + e– –0.11 V
[Cu(Cl4)]2– : ClO2 + 4H+ + 5[Co(NH3)6]2+ → Cl– + 2H2O + 5[Co(NH3)6]3+
hybridised orbitals
​E​θcell
  ​  = +1.50 – 0.11 = +1.39 V

3d 4s 4p 4d Since ​Eθcell
​  ​  > 0, reaction is feasible.
to accept 4 lone pairs of electrons Therefore, ClO 2 has oxidised
from 4 Cl– ions. [Co(NH3)6]2+ to [Co(NH3)6]3+.
2. ������������
(a) Cu 1s22s22p63s23p63d104s1
5. D
Cu2+ 1s22s22p63s23p63d9
Transition elements have the following
characteristics: (b) (i) B is Ag(NH3)2Cl
① High density. Tips
② High melting point.
The white ppt is AgCl(s) which is insoluble in water. NH3
③ Successive ionisation energies in transition forms a complex ion with silver called the diamine silver
elements show a steady increase (and not ion, [Ag(NH3)2]+ which have most of its compounds being
sudden jump) becasue 3d and 4s orbitals are soluble.
of similar energy. AgCl(s) + 2NH3(aq) → [Ag(NH3)2]+(aq) + Cl–(aq)
Note: The question wanted the formula of the compound,
not the formula of complex cation.
Section B
6. B (ii) [Cu(H2O)6]2+
1 – Due to the close proximity in energy of the Tips
4s and 3d electrons, transition elements can
[Cu(H2O)6]2+ cation is commonly written as Cu2+(aq).
form ions of roughly the same stability by
losing different number of electrons. Cation in C is [Cu(NH3)4(H2O)2]2+.
2 – Ions of transition elements such as Mn​O–4​ ​​ 
(c) (i) D: CuCl2 + 2HCl → H2CuCl4
(as in KMnO4) and Cr2O72– (as in K2Cr2O7)
are common oxidising agents in aqueous Tips
solutions and widely used in the laboratory.
When D is formed, one mole of CuCl2 reacts with exactly
3 – Cations are smaller than their respective 2 moles of HCl.
metal atoms because cations have one shell
less than the neutral atoms. Hence, the outer E: CuCl2 + Cu + 2HCl → 2HCuCl2
electrons are more strongly attracted by the
nucleus. This phenomenon is observed for Tips
both the transition elements as well as by When E is formed, one mole of CuCl2 reacts with 1 mole
s-block elements such as calcium. of Cu and exactly 2 moles of HCl. No other compounds
is produced.
PAPER 2
(ii) D: [CuCl4]2–
1. ���������
(a) (i) 6
(ii) No significant colour change observed E: [CuCl2]–
since Co2+(aq) is not oxidised by (iii) Ligand exchange
ClO2.
Tips
H2O ligand is replaced by Cl– ligand.

Answers to A Level H2 Topical Chemistry A43


© Singapore Asia Publishers Pte Ltd Topic 9.4

H2 Topical Chemistry 2014.indb 43 3/27/2014 1:57:30 PM


(iv) Tetrahedral ​E​θCell
  ​ 
​ = ​E​θR ​​ – ​EθO​ ​​ 
(v) +1 = (0.00) – (+0.34)
(d) (i) Reduction = –0.34 V
∴ Since E ​ θCell
​  ​ < 0; reaction is not feasible.
Tips Hence, residue is Cu.
Oxidation state of copper decrease from +2 in D to +1
in F. Tips
For Zn, Al & Mg which has negative Eθ value of –0.76 V,
(ii) F is CuCl   ​ will
–1.66 V and –2.38 V respectively, the calculated ​E​θCell
(e) (i) Basis: 100 g be +ve and thus feasible reaction i.e. metal dissolves.
Element Cu F K
(c) (i) Sulfuric acid acts as an oxidising
Mass (g) 21.5 38.7 39.8
agent.
Ar 63.5 19.0 39.1
(ii) Zn → Zn2+ + 2e–
Mole 0.338 2.036 1.018
SO42– + 4H+ + 2e– → SO2 + 2H2O
Molar Ratio 1 6 3 ∴ Overall Equation:
∴ Empirical formula of G is CuF6K3. Zn + SO42– + 4H+ → Zn2+ + SO2 + 2H2O
(ii) +3 (d) (i) Al(OH)3.
(ii) Al3+(aq) + 3OH–(aq) → Al(OH)3(s)
Tips Al(OH)3(s) + OH–(aq) → Al(OH)4–(aq)
Let the oxidation state of copper in G to be x. aluminate ion
∴ 3(+3) + 6(–1) + x = 0 (e) Electronic configuration of Zn is [Ar] 3d104s2.
∴ x = +3 Just like Group II elements, Zn has 2 electrons
in the outermost shell and tends to form
(f) (i) In E and F, the copper ion present is compounds with zinc having +2 oxidation
Cu+. state.
__
1
Cu+ 1s22s22p63s23p63d10 (f) (i) Zn(NO3)2(s) →∆ ZnO(s) + 2NO2(g) + ​ 2 ​ O2(g)
Cu+ does not have partially filled d (ii) Zn2+ (0.074 nm) has a larger ionic radii
orbitals and thus no d → d* transition than Mg2+ (0.065 nm) and thus has
of electrons can occur. lesser polarising power towards the
(ii) In G, the copper ion present is Cu3+. NO3– polyatomic anion.
Hence, Zn(NO3)2 is more stable to heat
Cu3+ 1s22s22p63s23p63d8 than Mg(NO3)2 and decomposes at a
Cu3+ have partially filled d orbitals. An higher temperature.
electron can be excited from a lower (g) NH3(aq) + H2O(l)  NH4+(aq) + OH–(aq)
energy to a higher energy d orbital. Zn2+(aq) + 2OH–(aq) → Zn(OH)2(s)
Energy is absorbed from some part of the Zn(OH)2(s) + 4NH3(aq) → [Zn(NH3)4]2+(aq)
visible light spectrum that corresponds + 2OH–(aq)
to the energy difference between energy
levels of upper and lower d orbitals. The PAPER 3
wavelength of light not absorbed will 1. (a) Solution containing Cu2+(aq) ions is blue
be seen as the colour of the complex. due to [Cu(H2O)6]2+ ions.
When dilute aqueous ammonia is gradually
3. ����������������������������������������������
(a) Metals 3 and 4 are Cu and Zn respectively.
added, a blue precipitate of Cu(OH)2(s) is
Tips first formed.
Transition metals are generally hard and dense due to the NH3 + H2O  NH4+ + OH–
relatively small metallic radii (small size) and closely packed
structures in their metallic lattices.
[Cu(H2O)6]2+(aq) + 2OH–(aq) → Cu(OH)2(s) + 6H2O
blue PPT
When excess dilute aqueous ammonia is
(b) Cu + 2e  Cu
2+
E = +0.34 V [O]
– θ

2H+ + 2e–  H2 Eθ = 0.00 V [R] added, the precipitate dissolves to give a


For Cu + 2H+ → Cu2+ + H2 deep blue solution, due to the formation of

Answers to A Level H2 Topical Chemistry A44


© Singapore Asia Publishers Pte Ltd Topic 9.4

H2 Topical Chemistry 2014.indb 44 3/27/2014 1:57:30 PM


complex ion, [Cu(NH3)4(H2O)2]2+, as a result 2. ������
(a) Fe2+: [Ar]3d6
of ligand exchange. Fe2+ has an incompletely filled d-orbitals. In
the presence of ligands, the 3d orbitals of
Cu(OH)2 + 4NH3 + 2H2O  [Cu(NH3)4(H2O)2]2+ + 2OH–
deep blue complex the transition metal become degenerate and
(b) CuO + 2NH3 → Cu(NH2)2 + H2O split into two energy levels. When visible
white light shines on it, the electron in a d-
Solution is deep blue in colour.
orbital of the lower energy level will absorb
Tips a quantum of light and jump to one of the
A Brønsted-Lowry base is a proton acceptor. partially filled d-orbitals of higher energy
O2– will extract proton from the solvent, NH3 to give NH2– level. The energy gap, E, between the two
(i.e. similar to the extraction of proton from water to form levels corresponds to a range of wavelengths
OH–). NH3 molecules could act as a ligand to the Cu2+ ion in the visible spectrum. The light energy not
in this solvent, as easily as in the case of aqueous medium absorbed will be seen as the colour of the
in (a) and form a similar complex i.e. [Cu(NH3)4]2+ complex.
Note:
complex, which is red in this case.
The resultant complex ion does not have a mixture of both (b)
NH3 and H2O ligands as there is no water present.

(c) [Cu(H2O)6]2+ + 4Cl–  [CuCl4]2– + 6H2O


blue yellow 'high spin' state 'low spin' state
When conc.HCl is added to the blue Cu2+(aq)
(c) Electrons usually prefer to occupy orbitals
solution, ligand exchange occurs and the
singly, in order to minimise inter-electronic
solution changes to pale yellow green due repulsion.
to yellow [CuCl4]2– ions formed.
(d) Oxyhaemoglobin (i.e. 'low spin' state) will
The reaction is reversible and thus the pale contain the larger energy gap, E, between
yellow green colour observed is due to the its d-orbitals.
presence of a mixture of blue [Cu(H2O)6]2+ For the 'low spin' state, the lower energy d-
ions and yellow [CuCl4]2– ions. orbitals are filled up first, before the higher
energy d-orbitals are used. This suggests that
No such colour change occurs when conc.
the energy required to overcome the energy
H2SO4 is added to Cu2+(aq) solution because
gap, E, in adding subsequent electrons to
no complexes are formed.
the higher energy d-orbitals is more than
Upon dilution of the yellow-green solution, that required to overcome inter-electronic
the position of the equilibrium is shifted repulsion in adding electrons to the partially
to the LHS with more [Cu(H2O)6]2+ ions filled lower energy d-orbitials.
present, thus producing the original pale For the 'high spin' state, the electrons occupy
blue colour. all the d-orbitals singly, before pairing up in
(d) (i) 2Cu2+ + 4I– → 2CuI + I2 the lower energy d-orbitals. This suggests
(ii) From the Data Booklet: that the energy required to overcome the
Cu2+ + e–  Cu+ Eθ = +0.15 V energy gap, E, in adding subsequent electrons
I2 + 2e–  2I– Eθ = +0.54 V to the higher energy d-orbitals is less than
∴ ​E​θcell  ​ ​ = ​E​   ​ ​– ​E​   ​ 
θ
red
θ
oxd
that required to overcome inter-electronic
= (+0.15) – (+0.54) repulsion in adding electrons to the partially
= –0.39 V filled lower energy d-orbitals.
Since ​Eθcell ​  ​  < 0, ∴ the reaction is not
feasible.
(iii) The redox reaction does in fact occur
because the reaction takes place under
non-standard conditions i.e. solutions
are not of 1.0 mol dm–3 concentration.

Tips
Eθ values apply only to standard conditions.

Answers to A Level H2 Topical Chemistry A45


© Singapore Asia Publishers Pte Ltd Topic 9.4

H2 Topical Chemistry 2014.indb 45 3/27/2014 1:57:31 PM


NO2
Topic 10.1 Organic Chemistry: sp2 H
3 – + sp3
Introductory Topics + NO2+ →

PAPER 1
Section A Tips
1. B Benzene ring has a planar structure and the C atoms are
Compound X must have a chiral carbon sp2 hybridised. The intermediate has a tetrahedral structure
and the reactive C atom is sp3 hybridised.
(marked *) in order to be optically active.
*
CH3CH(OH)CO2H 5. A
Compound X Y Z
1 mole of X liberates 1 mole of H2 when reacted
with Na suggests that X must have 2-OH Molecular Formula C12H10O2N2 C12H10O2N2 C6H5ON

groups. Empirical Formula C6H5ON C6H5ON C6H5ON

CH3CHCOOH + 2Na → CH3CHCOO–Na+ + H2 1 – Correct statement.


2 – Correct statement. Isomers is defined as
OH O–Na+ compounds having the same molecular
Common Mistakes
formula but different arrangement of atoms
Compound A – Produces only half a mole of hydrogen gas in the molecules.
due to the presence of one–OH group. 3 – Correct statement. Mr of Y = 2 × Mr of Z.
Compound C & D – No chiral centre in the molecules &
thus cannot be optically active.

2. C
Bupropion is optically active since it has a chiral
carbon (marked *)
CH3
*
CH3CNHCH(CH3)CO
Cl
CH3
3. D
CH3 CH3
O C H * CH
C C HO
* CH CH2
H2C * CH2 H2C CH2
C *
C H C
CH H
H3C CH2
CH3 CH3
carvone Q
(Chiral centres are marked *)

Section B
4. A
CH3 CH3

1 – C = O + CN– → H3C – C – O–
CH3 sp2 sp3
CN
CH3 H sp2
CH3 H
2 – C=C + H+ → H – C – C – CH3

CH3
H H
sp2 sp3

Answers to A Level H2 Topical Chemistry A46


© Singapore Asia Publishers Pte Ltd Topic 10.1

H2 Topical Chemistry 2014.indb 46 3/27/2014 1:57:31 PM


6. A
Topic 10.2 Hydrocarbons
Alkenes undergo electrophilic addition reaction
PAPER 1 with Br2 across the C=C bond.
Section A
1. B 7. C
O Limonene undergoes electrophilic addition with
__
1 catalyst excess Br2 in the absence of UV light or heat.
CH3CH = CHCH3 + ​ 2 ​ O2 CH3C – CH2CH3
but-2-ene Br
Br2 Br
**
2. C *
Br
Propene (an alkene) undergoes electrophilic * Br
addition reaction. (chiral centres are marked *)
δ+ δ–
CH3CH = CH2 + I – Cl Tips
slow Free radical substitution takes place in the presence of
+ UV light or heat.
CH3CHCH2I + Cl–
fast
8. D
CH3CHCH2I
A – Free-radical substitution gives rise to a
Cl mixture of mono-substituted product.
Tips B – C–F bond is too strong to be broken.
Chlorine (Cl) is more electronegative than iodine (I). Furthermore, F cannot be substituted in
one step by nucleophilic substitution using
3. A NH3.
The covalent bond between C2 and C3 are formed C – Alkyl group (–CH 3 in this case) is
by the overlapping of sp2 hybrid orbitals. 2,4- directing and so NO2 cannot be obtained
at position 3.
4. B
D – Alkyl group (–CH2CH3 in this case ) is 2,4-
Option B will not give a good yield of
directing and so Br at position 2 is possible.
1,2-dibromocyclohexane because the 2nd step
This can be done using Br2 and AlBr3 catalyst
involves free-radical substitution which is not
via electrophilic substitution.
stereospecific, and thus a mixture of dibromo
9. B
products will be formed.
Br2(aq) Br Br HBr Br A – HI(g) instead by HI(aq).
A: + C – AlCl3 readily hydrolyses in water and the
Br OH heat Br
resulting Al3+ ions cannot act as the Lewis
HBr Br Br2, hv
B: acid catalyst.
Br Br Br (mixture of dibromo D – Cl2(g) or Cl2(l) instead of Cl2(aq).
+ + products)
Br Br 10. B
Br In general, for the combustion of hydrocarbons:
cold KMnO4 OH red P, Br2 Br y
__ y
__
C: H 3O +
CxHy + (x + ​ 4  ​)O2 → xCO2 + ​ 2  ​H2O
OH Br

D:
OH conc. H2SO4 Br2 Br (  ______
2n + 2
)
∴ CnH2n + 2 + ​ n +  ​  4 ​   ( 
______
2n + 2
)
 ​O2 → nCO2 + ​ ​  2 ​  
 ​H2O

5. C
warm in CCl4 Br
(  n+1
_____
)
CnH2n + 2 + ​ n + ​  2 ​   ​O2 → nCO2 + (n + 1)H2O
CnH2n + 2 + (1.5n + 0.5)O2 → nCO2 + (n + 1)H2O
From the Data Booklet, the relevant bond energies
are 11. C
C–Cl 340 kJ mol–1 In chloroethene, CH2 = CHCl, the delocalisation
C–H 410 kJ mol–1 of electrons from the nearby Cl atom into the �
C–Cl bond is weaker than C–H bond and thus bond results in partial double bond character,
homolytic fission of C–Cl bond occurs more hence rendering it much less reactive.
readily.

Answers to A Level H2 Topical Chemistry A47


© Singapore Asia Publishers Pte Ltd Topic 10.2

H2 Topical Chemistry 2014.indb 47 3/27/2014 1:57:32 PM


12. C PAPER 2
A – Correct. UV light is required to initiate free- 1. (a) C14H30  CnH2n+2  Alkanes
radical substitution mechanism. (b) 2C14H30 + 43O2 → 28CO2 + 30H2O
B – Correct. Concentrated H2SO4 is required
(c) (i) mass of kerosene burnt
to form the nitronium ion from HNO3, via
= (10.8 × 10 700) kg
electrophilic substitution mechanism.
____________
10.8 × 10 700
C – Incorrect. Cl2 should not be in the aqueous = ​  1000  
 ​  tonnes
(aq) form. Electrophilic substitution = 115.56 tonnes
of methylbenzene requires anhydrous  116 tonnes
condition i.e. Cl2(g). (ii) Mr of C14H30 = 198
D – Correct. With dilute HNO3, mono-nitration
Mr of CO2 = 44
occurs to give a mixture of 2-nitrophenol
_____
Mass ____ 116
(as shown) and 4-nitrophenol plus water. Mole of C14H30 = ​  M  ​ 
 = ​ 198  ​
r
13. C From the above equation; 1 mol of
C14H30 will produce 14 mol of CO2.
H
____
116
H C H ∴ ​ 198  ​ mol of C14H30 will produce​
3 different types of
H H H free radical X• can be ( 
____
116
)
​ 198  ​× 14  ​mol of CO2.
formed by loss of one
H C C C C H hydrogen atom at the ∴ Mass of CO2 = Mole × Mr
H H H 3 different locations
marked:
____
116

= ​ ​ 198  ​× 4  ​× 44)
H C H = 361 tonnes
= 361, 000 kg.
H
2. (a) (i) H H

Section B H–O–C–C–O–H
14. D H H
Catalytic converter converts air pollutants (ii) H H δ– δ+
(harmful gases) into harmless gases.
2CO + 2NO → 2CO2 + N2 H–O–C–C–O–H

harmful gases harmless gases H H


Hydrogen Bonding
NO2, CO and SO3 are considered air pollutants
H H
(harmful gases).
H–O–C–C–O–H
15. B δ+ δ–
H H
CH3CH2CH2CH2CH2CH2CH3  →   CH3  + 4H2 (b) (i) Ethene gas
(ii) Purple colour of KMnO4 is decolourised.

1 – CH3CH2 3. COOH
CH3
CHCH2CH2CH2CH3 CHCOOH and
CH3
CH3 COOH

CH3 CH3 + 4H2


2 – CH3 CH3
CHCH2CH2CH
CH3 CH3

Answers to A Level H2 Topical Chemistry A48


© Singapore Asia Publishers Pte Ltd Topic 10.2

H2 Topical Chemistry 2014.indb 48 3/27/2014 1:57:32 PM


(iii) Mole of H in alkane D
Tips = 2 × Mole of H2O
CH3 CH3 = 2 × 0.0861
CH – CH = CH2 CH – COOH = 0.172 mol
CH3 CH3
Mole of C in alkane D = Mole of CO2
CO2 = 0.0689 mol
CH2CH2OH COOH
∴ H : C ratio in alkane D is 0.172 : 0.0689
2.5 : 1
5 : 2
∴ Molecular formula of alkane D is C4H10.
CH3 COOH (d) Possible structures:
CH3CH2CH2CH2CH2CH3 (alkane E)
PAPER 3 CH3CH2CO2H (acid B)
1. In Free-Radical Substitution, any one of the H
Tips
atoms in butane may be replaced by Br atom.
With reference to the electrolysis equation given:
H H H H R'CO2– + RCO2–  R–R + R–R' + R'–R'
There are 6H atoms around
H – C – C – C – C1 – H
1 2 2 C–1 and 4H atoms around Alkane D is C4H10 or C2H5–C2H5
C–2. R–R
H H H H Alkane C is C5H12 or C2H5–C3H7
Hence, ratio of 1-bromobutane : 2-bromobutane R–R'
6:4 ∴ Acid B is C2H5COOH (since Acid A is C3H7COOH).
3:2 Hence; Alkane E is C6H14 or C3H7–C3H7
_____
11.4 R'–R'
2. ��������������������������
(a) Mole of NaOH used = ​ 1000  ​ × 0.1
= 1.14 × 10–3 mol (e) Structures of F, G and H:
Since A is a mono-carboxylic acid, F is CH3CH – CHCH3
Mole of A = Mole of NaOH CH3 CH3
= 1.14 × 10–3 mol
Br
_____
Mass
Mr of A = ​   ​ 
Mole G is CH3CH – CCH3
_________
0.100
= ​    ​  CH3 CH3
1.14 × 10–3
= 87.7 g *
H is CH3CH – CHCH Br
A possible structure of A is CH3CH2CH2COOH. 2

(b) Using the Ideal Gas equation, CH3 CH3


___
m
PV = nRT = ​ M   ​ RT Reason: Since alkane F only gives two
r
isomeric monobromo compounds
_____
mRT
∴ Mr of C = ​  PV ​  when reacted with Br2 under UV
___________________
0.20 × 8.31 × 380 light, it must be a symmetrical
= ​       ​ alkane. This ensures that H
(1.01 × 105)(87 × 10–6)
= 71.9 atoms are in only two chemical
∴ Molecular formula of C is C5H12. environments.
(c) (i) P2O5 + 3H2O → 2H3PO4 Note that compound H has a chiral atom
CO2 + NaOH → Na2CO3 + H2O which makes it chiral.
(ii) Mole of H2O produced 3. �������
(a) HNO3 + 2H2SO4  NO2+ + H3O+ + 2HSO4–
_________
Mass ____
1.55 electrophile
= ​    ​ 
= ​   ​ = 0.0861 mol
Mr of H2O 18.0 Concentrated H2SO4 is a stronger acid than
Mole of CO2 produced HNO3 and protonates HNO3 to produce
_________
Mass ____
3.03 strong electrophile, the nitronium ion,
= ​    ​ 
= ​   ​ = 0.0689 mol NO2+.
Mr of CO2 44.0

Answers to A Level H2 Topical Chemistry A49


© Singapore Asia Publishers Pte Ltd Topic 10.2

H2 Topical Chemistry 2014.indb 49 3/27/2014 1:57:33 PM


(b) Type of reaction : Nitration of benzene rings NaOH
Cl – – CH2Cl Cl – – CH2OH
Mechanism : Electrophilic substitution with heat

NO2+ as electrophile L P
Type of reaction is nucleophilic substitution.
Step 1: Production of electrophile, NO2+
HNO3 + 2H2SO4  NO2+ + 2HSO4– + H3O+ Cl – – CH2COOH + HOH2C – – Cl
electrophile
N P
Step 2: Unstable Carbocation Intermediate
heat conc. H2SO4
Formation
H NO2 O
NO2+ slow + Cl – – CH2C – O – CH2 – – Cl
carbocation
Q
intermediate
Type of reaction is esterification.
Step 3: Expulsion of H from the +

intermediate to form product with 5. (a) Presence of ultra-violet light.


delocalised � system restored (b) Mechanism: Free Radical Substitution
H NO2 NO2
  Initiation : Cl—Cl u.v.
→ Cl● + Cl●
HSO​4 ​: 

+ fast
+ H2SO4 Propagation : C2H6 + Cl● → C2H5● + HCl
4. ����������������������������������������������������
(a) Phenol reacts with nitric acid more easily than C2H5● + Cl2 → C2H5Cl + Cl●
methylbenzene. The lone pair of electrons on Termination: Cl● + Cl● → Cl2
the –OH group of phenol is delocalised into C2H5● + Cl● → C2H5Cl
the benzene ring thus making phenol more C2H5●+C2H5●→CH3(CH2)2CH3
reactive towards electrophilic substitution, as In the presence of UV light, Cl2 molecules
compared to methylbenzene. The conditions undergo homolytic fission to produce
used is milder (use of dilute HNO3 instead of chlorine free radicals, Cl●, which then
conc. H2SO4) and no catalyst of sulfuric acid attacks C2H6 molecules in propagating steps.
is needed (to create the strong electrophile A chain reaction occurs rapidly. The chain
of NO2+). reaction is finally terminated when two free
(b) CH3 CH3 CH3 radicals combine to form a molecule in the
Cl terminating steps.
AlCl3
+ 2Cl2 + + 2HCl (c) The first propagation step (i .e. C2H6 + I● →
C2H5● + HI) is endothermic.
Cl Energy released in forming H–I bond
K
(bond energy = 299 kJ mol–1) is not able to
Type of reaction is electrophilic substitution. compensate the energy required to break the
stronger C–H bond (bond energy = 410 kJ
Tips
mol–1) in ethane.
Take K as 4-chloromethylbenzene.
6. (a)
Cl2 O * Note
Cl – – CH3 Cl – – CH2Cl
light
C CH3 Both compounds A
K L and B are ketones
O and can react with
Type of reaction is free radical substitution.
A B 2,4-DNPH.
NaCN
Cl – – CH2Cl ethanol Cl – – CH2CN (C8H8O) (C9H8O)
L M
Type of reaction in nucleophilic substitution. Tips
Step 1: Production of electrophile from CH3COCl and
H2O/H2SO4
Cl – – CH2CN Cl – – CH2COOH AlCl3
reflux
O O
M N  –
H3C – C – Cl + AlCl3 H3C – C+ + AlC​l​4​ ​
Type of reaction is acid hydrolysis.

Answers to A Level H2 Topical Chemistry A50


© Singapore Asia Publishers Pte Ltd Topic 10.2

H2 Topical Chemistry 2014.indb 50 3/27/2014 1:57:33 PM


Step 2: Formation of the intermediate 4-valent cation 7. (a) (i)
O O O O O
O H C – CH3 M 2+
N +
N +
M O 2+ 2–
N +
N+
H3C – C+ slow + –
O O– –O O– O O

O–

Step 3: Expulsion of H+ from the intermediate to form


product
O O

O
O N+ N+
C – CH3 –
O O O–
H C – CH3
+  – fast
(ii) 2N2O5 →
 4NO2 + O2
+ AlC​l​4​ ​ + HCl + AlCl3
(iii) Step 1: Initial homolytic breaking of
A an N–O bond
(C8H8O)
O O O O
N N N + N
Tips
O O O O O O
Step 2: Dimerisation of the N​O.​3​ ​radical
Step 1: Production of electrophile
O O
Cl
+ to give N2O6
 –
+ AlCl3 �lC​l​4​ ​
+A O O O O
Step 2: Formation of the intermediate 4-valent cation N + N N N
O O O O O O O O O
+
slow + Step 3: Dissociation of dimer N2O6 via
H
O N–O bond cleavage
O O O O
Step 3: Expulsion of H+ from the intermediate to form
product N N N + N   +  O  =  O
O O O O O O
+ AlC​l​4​ ​ fast
 –
�lCl3
+ HCl + A
+
(iv) Step 1 would be the slowest (rate-
H O
O determining) step, since it is the only
B one involving just 1 reactant molecule
(C9H8O)
and hence responsible for the overall
1st order reaction.
(b) Tri-Iodomethane (Iodoform) Test:
Tips
Reagent: aqueous iodine + aqueous NaOH
The order of reaction could be predicted from the rate-
Condition: warm determining step of a given mechanism.
Compound A is a carbonyl compound with
CH3CO– group and will react to iodoform (b) (i) HNO3
test to give yellow crystals of CHI3 when (ii) Step 1: Production of electrophile,
warmed with alkaline aqueous iodine. NO2+
O O
N2O5(g)  [N​O​+2​ ​N​O​–3​]​  (as given in
warm
C – CH3 + 3I2 + NaOH CHI3 + C–Na+ + 3HI (b), N2O5 in
yellow
crystal solution exists
in ionic form)
A
Step 2: Formation of the intermediate
Compound B is a carbonyl compound 4-valent cation
but does not have the CH3CO– group.
slow H
It will not react with alkaline aqueous N​O​+2​ ​
+

iodine. NO2
carbocation intermediate
Answers to A Level H2 Topical Chemistry A51
© Singapore Asia Publishers Pte Ltd Topic 10.2

H2 Topical Chemistry 2014.indb 51 3/27/2014 1:57:34 PM


Step 3: Expulsion of H+ from the
Topic 10.3 Halogen Derivatives
intermediate to form the products
H
PAPER 1
– fast Section A
+   +  N​Ö​3​​  NO2  +  HNO3
NO2
nitrobenzene 1. D
CH3CH2CH2CH3 does not contain any halogen
(c) (i) N O (g) ∆​Hr​​ ​= +92.5 kJ mol NO (g) + NO (g)
θ –1
atoms and will not destroy ozone in the upper
2 5 2 3
atmosphere. It can be used safely as replacement
for CFCs.

N2(g) + O2(g)
2. A
Bromoethane undergoes nucleophilic substitution
∆​H​θr​  ​ = Σ∆​Hθf​​ ​(products) – Σ∆​Hθf​​ ​(reactants) reaction with ammonia to form ethylamine.
+92.5 = [∆​Hθf​​ ​(NO2) + ∆​Hθf​​ ​(NO3)] – ∆​Hθ​f​ ​(N2O5)
∴ ∆​Hθf​​ ​(NO3) = +64.3 kJ mol–1 ··N​​ H  + CH CH+ – Br– → CH CH NH + HBr
3 3 2 3 2 2
ammonia bromoethane ethylamine
(nucleophile)
(ii) 2NO3(g) → 2NO2(g) + O2(g)
∆​H​θr​  ​ = Σ∆​Hθf​​ ​(products) – Σ∆​Hθf​​ ​(reactants) 3. A
= [2∆​H​θf​ ​(NO2) + ∆​H​θf​ ​(O2)] – [2∆​Hθf​​ ​(NO3)] Compound X is a halogenoalkane.
= –62.2 kJ mol–1 BrCH2CH2CH2CH2Br
(d) Reaction 1: KCN
N2O5(g) → NO2(g) + NO3(g) ∆H = +92.5 kJ mol–1 in ethanol
1 mol 2 mol ∴ ∆S = +ve NCCH2CH2CH2CH2CN
Reaction 2: reduction
2NO3(g) → 2NO2(g) + O2(g) ∆H = –62.2 kJ mol–1
2 mol 3 mol ∴ ∆S = +ve H2N(CH2)6NH2
Based on ∆G = ∆H – T∆S
4. A
Reaction 1: ∆G = (+92.5) – T(+ve) = ?
Aryl Cl are resistant to hydrolysis. Thus no Cl
For reaction 1 to be spontaneous, temperature
atoms will be removed by hydrolysis.
of reaction must be high enough so that ∆G
= –ve 5. D
Reaction 2: ∆G = (–62.2) – T(+ve) = –ve The reaction involves nucleophilic substitution
This reaction is a spontaneous reaction �lCH2CO2H.
of a halogenoalkane, C
regardless of the temperature. Cl
For the same reasons above, we would expect acts as the nucleophile.
O–
reaction 2 to be more spontaneous due to a Cl
more –ve ∆G.
6. D
Any precipitate formed is due to formation of
AgCl(s).
Ag+(aq) + Cl–(aq) → AgCl(s)
white ppt
from ethanolic from hydrolysis of
silver nitrate organic compound

�����������������������������������������������������
Since little or no precipitate observed, it suggests
that compound S does not undergo hydrolysis .
Option D is the answer since in halogenoarene,
the C–Cl bond is strong and does not hydrolyse
to provide Cl–(aq) ions.
7. B
CH3COCl and –COCl hydrolyse readily and
gives immediate precipitate with AgNO3.

Answers to A Level H2 Topical Chemistry A52


© Singapore Asia Publishers Pte Ltd Topic 10.2/Topic 10.3

H2 Topical Chemistry 2014.indb 52 3/27/2014 1:57:34 PM


–Cl are relatively inert and difficult to 13. C
hydrolyse; even when boiled with NaOH. 1 – Benzene ring undergoes electrophilic
aromatic substitution during nitration with
CH3CH2CH2CH2Cl, an alkyl halide is X, since HNO3/H2SO4.
it hydrolyses readily when boiled with aqueous
2 – Halogenoalkanes undergoes nucleophilic
NaOH, but not in water at room temperature. With
substitution during alkaline hydrolysis with
NaOH, nucleophilic substitution occurs, releasing NaOH(aq). OH– ion act as nucleophile.
Cl– ions which reacts with Ag+ ions (from AgNO3)
3 – Halogenoalkanes undergoes nucleophilic
to form white precipitate of AgCl. substitution with NH3(g). NH3(g) acts as
8. D nucleophile.
NaCN 14. A
– CH2Cl – CH2CN
(Nucleophilic Substitution) P 1 – Electrophilic addition.

H2, Pt CH2 = CH2 + Br2 → CH2BrCH2 + Br–
– CH2CN – CH2CH2NH2 ↓
P (Nitrile Reduction) Q CH2BrCH2Br

9. B 2 – Nucleophilic substitution.
SN1 substitution (Unimolecular, Nucleophilic CH3CHBrCH3 + CH3O– → CH3CHCH3 + Br–
Substitution) involves the formation of a
carbocation intermediate via the heterolytic OCH3
fission of a C–X bond. 3 – Nucleophilic substitution.
CH3CH2Br + NH3 → CH3CH2NH3+ + Br–
Common Mistakes

C – It shows a carbocation intermediate for electrophilic
substitution mechanism. CH3CH2NH2 + HBr
D – It shows a transition state for SN2 substitution
(Bimolecular, Nucleophilic Substitution). 15. A
1 – Bromide is oxidised by conc. H2SO4 to
10. B bromine, Br2.
SN2 reaction is a continuous one-step process 2 – Conc. H2SO4 is reduced by bromide to sulfur
that involves the simultaneous loss of anionic dioxide.
leaving group (with δ–) and a gain of the anionic NaBr + H2SO4 → HBr + NaHSO4
nucleophile (with δ–). 2HBr + H2SO4 → Br2 + SO2 + 2H2O
3 – 1-bromopropane can undergo dehydration
11. D in the presence of conc. H2SO4 to produce
A, B and C are CFCS. Only D is a hydrocarbon. propene, which then reacts with steam (since
at high temperature) to produce propan-2-ol,
Section B based on Markovnikoff’s Rule.
12. A
1 Halogenoalkanes undergo nucleophilic 16. D
substitution whereby the OH– nucleophile Halogenoalkanes undergo hydrolysis in the
attack the carbon of the C–Hal bond. presence of aqueous NaOH to form alcohols.
2 Between X and Y, the C–Hal bond will be reflux
lengthening as C–OH bond is gradually CH3CH2CH2Br + NaOH CH3CH2CH2OH + NaBr
formed, as they form the transition state Z CH3CH2CH2OH product formed will provide
R  the alcoholic conditions for elimation reaction
to occur.
Hal C OH
reflux
H H CH3CH2CH2Br + OH– (alcohol) CH3CH = CH2
(–HBr)
3 Energy profile shows a one stage process by-product
involving the formation of transition state + Br– + H2O
Z, and the energy level from reactant to the
highest energy level represents the activation
energy, Ea.

Answers to A Level H2 Topical Chemistry A53


© Singapore Asia Publishers Pte Ltd Topic 10.3

H2 Topical Chemistry 2014.indb 53 3/27/2014 1:57:35 PM


17. B (f) 1-bromobutane will be in the lower layer.
1 – The electrons of the Br atom delocalise into Reason: 1-bromobutane has a higher density
the π bond of the C=C group and this makes than water.
the C–Br bond too strong to be broken.
Tips
2 – There is a region of high electron density on
Compare the densities (and not the Mr) to decide which
the π bond of the C=C group which repels the liquid would be at the upper or lower layer.
attacking nucleophile (electron proficient).
(g) (i) Concentrated hydrochloric acid
PAPER 2 reacts with butan-1-ol.
1. ����������������������
(a) stage I : NaBr + H2SO4 → NaHSO4 + HBr (ii) CH3CH2CH2CH2OH + HCl →
stage II: CH3CH2CH2CH2OH + HBr → CH3CH2CH2CH2OH2+Cl–
CH3CH2CH2CH2Br + H2O
Tips
(b) Mass of butan-1-ol in 25 cm3 = 25 × 0.81
The protonation of an aliphatic alcohol is an unusual case.
= 20.25 g Clue: Use of concentrated strong acid HCl.
No. of moles of butan-1-ol in 25 cm3
_____
Mass _____20.25 (iii) Product in (ii) is ionic and is able to form
= ​  M  ​ 
 = ​  74 ​ 
 = 0.274 mol ion-dipole interaction with water.
r
Butan-1-ol forms hydrogen bonding
No. of moles of NaBr used
35 g with water.
_____
Mass _____
= ​  M  ​ 
 = ​ 102.9  ​ 
= 0.340 mol Ion-dipole interaction is a stronger
r
interaction with water than hydrogen
Since mole of NaBr  mole of HBr  mole bonding and thus product in (ii) is more
of CH3CH2CH2CH2OH, soluble.
∴ NaBr is in excess. (h) Impurity: Concentrated HCl.
(c) (i) ���������������������������
Heat is evolved when conc.H2SO4 added Equation: 2HCl + Na2CO3 → 2NaCl + CO2
reacts with water in the flask. + H2O
(ii) Inorganic by-product: Br2 and SO2 (i) Water is removed.
Equation : 2HBr + H2SO4 →
Br2 + SO2 + 2H2O Tips
Anhydrous CaCl2 is a good drying agent.
Organic by-product: CH3CH2CH = CH2
Equation: CH3CH2CH2CH2OH → (j) From 100ºC to 104ºC.
CH3CH2CH = CH2 + H2O
PAPER 3
Tips
1. (a) The rate of reaction is slower for 2-
• Formation of inorganic by-product
→ due to reaction of halide ions with conc.H2SO4
bromobutane as compared to 2-iodobutane.
• Formation of organic by-product This is because the C-Br bond is shorter and
→ due to dehydration of alcohols to form alkenes at high thus, stronger than the C–I bond.
temperature (b)
NaCN LiAlH4
(d) Heat energy is required to break the many • CH3CH2CHCH3 CH3CH2CHCH3 CH3CH2CHCH3
in alcohol
strong covalent bonds before reaction can Br CN CH2NH2
occur. NaOH
CH3CH2CHCH3 CH3CH = CHCH3 and CH3CH2CH = CH2
(e) Compounds are unreacted butan-1-ol and •
cold KMnO4(aq)
the product water. Br
CH3CH – CHCH3 CH3CH2CH – CH2
Reason: Boiling point of water is lower
than that of 1-bromobutane while OH OH OH OH

boiling point of butan-1-ol is very


2. (a) Type of reaction: Electrophilic Addition
close to that of 1-bromobutane. As
such, they are distilled over together Mechanism: Electrophilic addition with the
with 1-bromobutane. positive end of the polarised
X–X as the electrophile.

Answers to A Level H2 Topical Chemistry A54


© Singapore Asia Publishers Pte Ltd Topic 10.3

H2 Topical Chemistry 2014.indb 54 3/27/2014 1:57:35 PM


H H 3. (a) Nucleophilic Substitution.
H H slow (b) J is
C=C H–C–C–H Br – CH2CH2CH2CH2CHCH2CH2CH3
H H 
δ+ δ– X
Br
X–X
·· 
​ –
X​ 1,5-dibromooctane.
fast Tips
Reaction involved is formation of amines from
H H halogenoalkane compound.
H–C–C–H
(c) The four isomeric alkenes are:
X X
(b) (i) Boiling point increases from C2H5Cl to H H CH2 = CHCH2 H
C2H5I due to stronger intermolecular C=C C=C
van der Waals' forces as the number
of electrons (and thus molecular size) CH2 = CHCH2 CH2CH2CH3 H CH2CH2CH3
increases from C2H5Cl to C2H5I. cis isomer trans isomer
(ii) Bond polarity decreases from C–Cl CH2 = CHCH2CH2 H
H H
to C–I due to a decrease in the
electronegativity from Cl to I. C=C C=C
(iii) Reactivity towards nucleophilic
reagent: CH2 = CHCH2CH2 CH2CH3 H CH2CH3
C2H5Cl < C2H5Br < C2H5I cis isomer trans isomer
Reactivity towards nucleophilic reagent
increase from C2H5Cl to C2H5I due to Tips
the decrease in bond strength from C–Cl
Reaction involved here is elimination reaction.
to C–I.
Halogenoalkane → Alkene
Common Mistakes
Question asked only about C2H5Cl, C2H5Br and C2H5I. 4. (a) A is C is
Thus, do not mention about C2H5F. Cl

(c) Structural formulae of compound P:


CH3Br H B is D is
Br
Cl
H3C – C – C *– C – CH3 Cl Br

H CH3 H (b) (i) CFCs are chemically inert, volatile,


(chiral carbon is marked *) non-flammable and have low toxicity.
The four alkenes are: (ii) CFCs are compounds that are liquefied
under pressure and become gaseous
H CH3 H CH(CH3)2
C=C C=C readily when the pressure is released.
CH3 CH(CH3)2 CH3 CH3 They do not react with air or water,
I II and rises high up into the stratosphere.
H CH2CH3 CH3 CH3 CFC molecules undergo photochemical
C=C C=C dissociation (in the presence of UV
H CH(CH3)2 CH3CH2 CH3
light) to produce reactive Cl● free
III IV radicals, which then initiate a chain
(I and II are cis-trans geometrical isomers) reaction , breaking O3 to form O2. This
will lead to the destruction of the ozone
Tips
layer.
• Q is a 3º alcohol since it is not oxidised by hot, acidified
(iii) Alkanes are flammable.
Na2Cr2O7(aq).
• P is an alkyl halide with the chiral carbon holding the 5. (a) The relative ease of hydrolysis of
halogen bromine. monohalogenoethanes (i.e. relative
• HBr can be eliminated from P in three possible ways to
give four different isomeric alkanes (of which two are
reactivities) can be determind by the bond
geometrical isomers of each other) energies of the C-Hal bond. The ease of

Answers to A Level H2 Topical Chemistry A55


© Singapore Asia Publishers Pte Ltd Topic 10.3

H2 Topical Chemistry 2014.indb 55 3/27/2014 1:57:36 PM


breaking C-I and C-Cl bonds are 240 kJ
Topic 10.4 Hydroxy Compounds
mol–1 and 340 kJ mol–1 respectively. Since
C-I bond is much lower, C2H5I tends to PAPER 1
hydrolyse almost immediately. However, for Section A
C2H5Cl, the C-Cl bond is very strong and so 1. C
does not break so easily. As such, C2H5Cl Solid Z is a salt of a strong base and a weak acid
will hydrolysed slower than C2H5I. since it dissolves readily in water to give a weakly
(b) (i) alkaline solution.
CH3 I CH3 OH
Z is C6H5O–Na+ which is formed from NaOH
OH–(aq) NH3 in (strong base) and C6H5OH (weak acid).
Cl (1 mol) ethanol
Cl 2. C
C
Tri-iodomethane (Iodoform) reaction – test for
CH3 OH
presence of CH3CH(OH)–group in alcohols.
Isomeric alcohols with molecular formula C5H12O
NH2
D are:
C7H15NO CH3CH2CH2CH2CH2OH CH3CH CHCH3
pentan-1-01
OH CH3
Tips CH3CH CH2CH2CH3
3-methylbutan-2-01
Formation of C:
OH CH3CH2CHCH2CH3
Note that only 1 mol of OH–(aq) is reacted in the hydrolysis
pentan-2-01
reaction. Since C-I bond is weaker than C-Cl bond, therefore OH
we expect I to be substituted by OH group. CH3CH2CHCH2OH
pentan-3-01
Formation of D: CH3 CH3
The remaining C-Cl group would react with NH3 (in ethanol) 2-methylbutan-1-01
to form amine by nucleophilic substitution. CH3CH2C CH3
CH3CHCH2CH2OH

(ii) H O H H OH
CH3
2-methylbutan-2-01
E: Cl – C – C – N – C – C – O – H 3-methylbutan-1-01
CH3
H H H H
O CH3C CH2OH
H C N
H CH3
F: C H C 2,2-dimethylpropan-1-01
H O C H
H H Only pentan-2-01 and 3-methylbutan-2-01 with
the CH3CH(OH)–group give yellow precipitate
(of CHI3) with alkaline aqueous iodine.
3. C
Tips
CH3(CH2)10O – (CH2CH2O)10H
H O H H H
C11 alcohol excess epoxyethane
Cl – C – C – Cl + H – N – C – C – O – H
H H H 4. C
H O H H Conc. H2SO4 (reagent Y) is used to dehydrate
Cl – C – C – N – C – C – O – H ethanol to ethene.
H H H H conc. H2SO4
C2H5OH CH2 = CH2 + H2O
Compound E
warm  +Na NaOH (Solution Z) is used to neutralise and
O remove any excess conc. H2SO4 from ethene.
H C N H O H H
C
H δ+ 5. A
H C Cl – C – C – N – C – C – O–
Structure of sugar, C6H12O6 is:
H O C H
H H H H
H H H H H H H O
Compound F H–C–C–C–C–C–C–H
C4H7NO2
OH OH OH OH OH

Answers to A Level H2 Topical Chemistry A56


© Singapore Asia Publishers Pte Ltd Topic 10.3/Topic 10.4

H2 Topical Chemistry 2014.indb 56 3/27/2014 1:57:36 PM


Each –OH group reacts with one mole of Na to
form half a mole of H2(g). Common Mistakes
Thus, five –OH groups on each sugar molecule The –OH group attached to the –CH2–­ group is not replaced
5 ​ moles by Cl atoms, as indicated in the question.
will react with 5 moles of Na(s) to form ​ __
2
of H2(g).���
9. B
6. D Cold, dilute acidified KMnO4 will:
A – The phenol group ( –OH) will react ① Cause the cyclohexene ring to become
with dilute nitric acid to form a substitution saturated, i.e. become a diol. The 2 carbon
product. atoms connectetd to the 2 –OH groups are
B – It will form only one ester group (–COO–) chiral carbons.
when the carboxylic acid group (–COOH) ② Secondary alcohols are oxidised to ketones.
reacts with ethanol under reflux in the
CH3 CH3
presence of concentrated sulfuric acid. (2 new chiral
cold, dilute
C – Ajulemic acid does not get oxidised. H acidified *H
centres are
* marked *)
D – Br2(aq) will react with both alkene and phenol HO KMnO4 O OH
to form the following product: OH
Br CO2H
OH Section B
Br 4Br atoms incorporated. 10. B
Br 1– Its aqueous solution is acidic due to presence
H3C (CH2)5CH3 of acidic phenol group
H3C O
Br CH3 CH3 2– A chiral carbon (marked*) is present in the
molecule.
7. A
Since the product at the end of oxidation is a Common Mistakes
diketone, compound P must be a 2º alcohol. 3 – An ester is formed when the phenol group reacts with
acyl chloride or carboxylic acid. The molecule does
H OH OH H H O OH H
not react with ethanol to form ester.
H – C –*C – *C – C – H oxidation
H – C – C –*C – C – H

H H H H H H H PAPER 2
P C4H10O2 Q C4H8O2 1. ���
(a)
A diol oxidation Compound Structural Formula
Reagent
A, B, C or D of the Organic Product

H O O H O
(Chiral centres are marked *)
O – C – CH3
H–C–C–C–C–H CH3COCl C OCH3

H H
CH2CH = CH2
R C4H6O2
OH
A diketone
O2N
Both P & Q have CH3CH(OH)– group while R Dilute HNO3 C
OCH3

has CH3CO– group. All give positive result with


CH2CH = CH2
iodoform test.
ONa
8. C Na C
OCH3

CH2OH CH2OH CH2CH = CH2


O O
H H OH H H Cl O
CH = CH – C – O–
Tollens' Reagent A

OH OH H H Cl Cl H H
H OH H Cl (b) (i) Aqueous bromine will react with
Molecular formula = C6H8Cl4O2 the phenol and alkenes functional
Empirical formula = C3H4Cl2O groups.

Answers to A Level H2 Topical Chemistry A57


© Singapore Asia Publishers Pte Ltd Topic 10.4

H2 Topical Chemistry 2014.indb 57 3/27/2014 1:57:37 PM


(b) (i) Reaction of phenol
OH OH
CH(OH)CH2NH2
Br OCH3 Br OCH3 Br Br
& Br OH
CH2CHCH2 CH2CHCH2 OH

Br Br OHBr (ii) Acid-Base reaction of phenol
CH(OH)CH2NH2
Tips
C=C bond on the side chain undergo Electrophilic
Addition whereas the phenol group undergo Electrophilic
O–Na+
Substitution O Na+

Common Mistakes
Alcohol do not react with NaOH.
Common Mistakes
The structure with side chain –CH2CH(OH)CH2Br is the
(iii) Oxidation of 2º Alcohol
major product, rather than –CH2CH(Br)CH2OH when
bromine attacks the double bond during Electrophilic O CH2NH3+
Addition, it is more favourable to form the carbocation C
intermediate with side chain –CH2CHCH2Br, rather than

–CH2CH(Br)CH2 since the 2º carbocation is more stable. OH
 OH

(ii) Oxidative cleavage of C=C bond Common Mistakes


K2Cr2O7/H+ does not oxidise the side chain of benzene.
COOH O
(iv) Acid-Base reaction of amine
+ CH3C – OH
CH(OH)CH NH​ + ​C  l–
2 3
OCH3
(c) (i) CH = CHCH2OH OH
OH
Common Mistakes
(ii) Stage I
The 2 C–OH group on benzene and the 1 C–OH group in
reagent(s) : LiAlH4 the side-chain does not react with HCl.
conditions : In dry ether
Tips 3. ��������
(a) (i) reaction with aqueous bromine
Reduction of Aldehydes to Primary Alcohol
C=C + Br2 + H2O C – C – + HBr
Stage II Br OH
reagent(s) : CH3CO2H ∴ Compound K has an alkene functional
conditions : Heat under reflux in the group.
presence of a small amount (ii) reaction with sodium metal
of conc. H2SO4 as catalyst. ___
2.4
No. of moles of H2 = ​ 24 ​ = 0.1 mol
Tips Mole ratio of K : H2
– Esterification reaction between carboxylic acids & 0.1 : 0.1
alcohols. 1 : 1
– Can also use CH2COCl at room temp. for esterification.
Mole ratio of K : Protons (Hydrogen)
1 : 2
2. (a)  Phenol ∴ K contains 2 hydroxy (–OH) groups
 Primary amine from 2 alcohols.
 Secondary alcohol
Common Mistakes
Not sufficient to leave answer as 2–OH groups because
–OH groups could come from alcohol or phenol.
Phenol is eliminated because there are less than 6 carbons.

Answers to A Level H2 Topical Chemistry A58


© Singapore Asia Publishers Pte Ltd Topic 10.4

H2 Topical Chemistry 2014.indb 58 3/27/2014 1:57:38 PM


(b) (b) C7H6O3 (Mr = 138)

Tips Tips
Stereoisomers refer to both the cis-trans isomers as well • Alkaline hydrolysis is whereby –Cl has been replaced
as the optical isomers. by –OH via nucleophilic substitution.
• The hydroxy (–OH) functional group is then oxidised to
carboxylic acids.
H H
H H O
H–O–C C–O–H
C=C H – C – Cl H–C–O–H C–O–H
H H
H H
cis-isomer O–H O–H O–H
H C7H7OCl C7H6O3

H–O–C H • Phenol ( –OH)group is not oxidised by KMnO4.


C=C H • MnO4– is being reduced to Mn2+ and thus decolorised.
H
H C–O–H
(c) (i) C7H4OClBr3
H
trans-isomer
Tips
Cis-trans (Geometric) isomers are common Halogenation (bromine water) of Phenol group:
in compounds with C=C bonds (where H H
rotation is restricted due to the presence of
H – C – Cl H – C – Cl
π bonds) and in certain ring systems. + 3Br2(aq) Br + 3HBr
Br
H
O–H O–H
H–O–C C7H7OCl Br
H White solid
H No cis-trans isomerism.
C=C (Mr = 379.2)
H H No optical isomerism.
Tri-bromination occurs.
H–O–C
H
(ii) Electrophilic Substitution
(Halogenation)
mirror plane
(iii) Benzene molecule is electron-rich and
H H involves the attack by electrophiles on
O O the � electrons of the aromatic benzene
H H H H ring.
C* C* Phenol will undergo electrophilic
H–O–C C=C C=C C–O–H
substitution with Br2(aq) to form 2, 4,
H H H H H H H H
6-tri-substituted compound containing
3 Br atoms.
Optical Isomers
(iv) Phenol
Optical isomerism arises because of the
(v) Reaction 3.
ability of compounds to rotate the plane of Side-chain oxidation of benzene ring
polarisation of plane-polarised light. will produce a benzoic acid.
4. �����������
(a) (i) AgCl (d) (i) H
(ii) Chloroalkane H – C – Cl
(iii) Alcohol and Phenol
(iv) Phenol. Alcohol does not react with O–H
alkali. (ii) The –OH functional group is electron
Phenol is acidic and so reacts with alkali donating and is a strong activating
group which cause 2, 4 directing when
to give salt and water.
Br2(aq) is added, the three Br atoms will
be attached to the 2, 4, 6 position with
respect to –OH functional group.

Answers to A Level H2 Topical Chemistry A59


© Singapore Asia Publishers Pte Ltd Topic 10.4

H2 Topical Chemistry 2014.indb 59 3/27/2014 1:57:38 PM


PAPER 3 Methanol to C:
1. • To convert butan-1-ol into 1-bromobutane: Reagent: Acidified KMnO4(aq)
– Butan-1-ol is refluxed with NaBr and conc. Condition: Heat under reflux
H2SO4. The resulting HBr produced will react
Tips
with butan-1-ol.
Methanol [O] Methanal [O] Methanoic Acid [O]
Tips CO2 + H2O  H2CO3
Alternatively, you can also react it with PBr3, PBr5 or SOBr2
at room temperature.
(b) (i)
D: CH3CH2CH2CH2OH E: CH3CH2CHCH3
• To convert butan-1-ol into 2-bromobutane (in
two steps): OH
CH3
– Heat with excess conc. H2SO4 at about 170ºC
F: CH3 – C – CH3 G: CH3CHCH2OH
to get but-1-ene, and then react with HBr.
OH CH3
2. ��������������������������������������������
(a) The experiment should be performed in a
fume cupboard since SO2 gas is poisonous (ii) D and G are primary alcohols.
and HCl gas is acidic. E is a secondary alcohol.
(b) SOCl2 + H2O → SO2 + 2HCl F is a tertiary alcohol.
(c) • A white precipitate is observed when
(iii) E is chiral
AgNO3(aq) is added.
*
Ag+(aq) + Cl–(aq) → AgCl(s) CH3CH2CHCH3 (chiral carbon marked *)
(from AgNO3) (from HCl) white ppt
OH
• Orange solution of K2Cr2O7 turns green
since SO2 is a reducing agent. (iv) E contains the CH3CH(OH)– group and
will react with alkaline aqueous iodine.
Cr2O7 + 2H++ 3SO2 → 2Cr3+ + 3SO42– + H2O
2–

orange colour green colour Products are CHI3 and CH3CH2CO2–.


(v) E, a secondary alcohol, is oxidised to
3. �������������������������������������������
(a) (i) Let the oxidation no. of carbon in
give a ketone, CH3CH2COCH3 which
CH3OH be x.
∴ x + 3(+1) + (–2) + 1 = 0 is a non-acidic organic product.
x = –2 4. �����
(a) Br
(ii) O O O Br
HO O
H–C–H H – C – OH HO – C – OH
A B C Br Br
OH
(O.N. = 0) (O.N. = +2) (O.N. = +4) O
(iii) Methanol to A: OH
Reagent: Acidified K2Cr2O7(aq) Br
Condition: Heat-gently and distill off OR Br
products. Br
HO O

Tips
Br Br
Methanol (an 1º alcohol) is oxidised to methanal (an Br
aldehyde). O
OH

Methanol to B: Br
Reagent: Excess acidified K2Cr2O7(aq) Tips
Condition: Heat under reflux. Br2(aq) reacts with both phenol & C=C functional
groups.
Tips
Methanol (an 1º alcohol) is oxidised to methanoic acid (a
carboxylic acid).

Answers to A Level H2 Topical Chemistry A60


© Singapore Asia Publishers Pte Ltd Topic 10.4

H2 Topical Chemistry 2014.indb 60 3/27/2014 1:57:39 PM


(b) F (C15H14O4) is: No. of moles of H2(g)
HO O

( _____
10.9
​ ​ 1000 
_______ )
 ​  ​
= ​  24.0 ​   
*
*
= 4.542 × 10–4 mol
OH
OH
No. of moles of CxHyOH (in 0.10 cm3) __
_______________________________ 2
G (C15H12O4) is: ​           ​= ​ 1 ​ 
No. of moles of H2(g)
HO O ∴ No. of moles of CxHyOH (in 0.10 cm3)
* = 2 × No. of moles of H2(g)
= (2)(4.542 × 10–4 mol)
O = 9.083 × 10–4 mol
OH
(chiral carbon atoms are marked *) Complete combustion of J gives
• When treated with Ni & H2: CO2(g) and H2O(l):
→ Addition of H2 across the C=C bond. y + 1 __
_____ 1
→ Reduction of ketone to 2º alcohol. CxHyOH(l) + (x + ​  4 ​  – ​ 2 ​ )O2(g) →
→ Obtain compound F. y + 1
_____
xCO2(g) + (​  2 ​ 
 )H2O(l)
• 1 mole of F reacts with 3 moles of
Na(s): Vol. of CO2(g) absorbed by NaOH
→ F has 3 –OH groups which reacts with = 109 cm3
Na(s) to form H2(g) No. of moles of CO2(g)
Na+ –O O
​ 3 ​  H2(g)
+ __
2
(  _____
109
_______ )
​ ​ 1000  ​  ​
= ​  24.0 ​    = 0.004542 mol

Na+ –O ∴ Ratio of CxHyOH : CO2


O– Na+
= 9.083 × 10–4 : 0.004542
• F dissolves in NaOH(aq): = 1 : 5
→ F contains acidic phenol groups which Hence, x = 5
can undergo acid-base reaction. Vol. of O2(g) used in combustion
O
= 109 + 54.4
Na+ –O
+ H2O
= 163.4 cm3

No. of moles of O2(g) used


OH
O Na – +


​ _____
_______ )
​ 163.4 ​  ​
= ​  1000 ​   
24.0
• F reacts with acidified K2Cr2O7 to give
= 0.006808 mol
compound G:
→ 2º alcohol group has been oxidised to ∴ Ratio of CxHyOH : O2
from G, a ketone. = 9.083 × 10–4 : 0.006808
= 1 : 7.5
• Compound G gives an orange precipitate
y + 1 __
_____ 1
with 2,4-DNPH: C5HyOH + (5 + ​  4 ​   – ​ 2 ​ )O2 →
→ confirms that compound G is a carbonyl y+1
_____
compound 5CO2 + (​  2 ​   )H2O
→ this is a condensation reaction and the y + 1 __
orange precipitate is 2,4-dinitrophenyl ∴ 5 + _____
​   ​  – ​ 1 ​  = 7.5
4 2
hydrazone. y = 12 – 1
5. ����������������������������������������������
(a) J reacts with excess sodium metal to give y = 11
H2(g): (b) Alcohol J is either a 1º or a 2º alcohol (and
not a 3º alcohol) since it undergoes oxidation
2CxHyOH(l) + 2Na(s) → 2CxHyO–Na+(aq) + H2(g) with K2Cr2O7/H+ to be dehydrated to an
0.10 cm3 10.9 cm3 alkene.

Answers to A Level H2 Topical Chemistry A61


© Singapore Asia Publishers Pte Ltd Topic 10.4

H2 Topical Chemistry 2014.indb 61 3/27/2014 1:57:39 PM


Since alkene K undergoes oxidative cleavage
Topic 10.5 Carbonyl Compounds
with hot conc. KMnO4/H+ to give equimolar
mixture of CH3CO2H and CH3COCH3, it PAPER 1
must have the following structure: Section A
CH3 1. C
H
C=C Tri-iodomethane (Iodoform) reaction.
CH3 CH
3
O NaOD O
Hence, alcohol J must be a 2º alcohol, with CH3C D O CH3C + CDI3
the following structure: CI3 2 O–Na+ yellow
CH3 H PPt

H3C – C – C – CH3 2. B
A homogerous catalyst is a catalyst that is in the
H OH
same phase as the reactant.
(c) Heat J with excess concentrated H2SO4(aq)
B : Catalyst used for the reaction of ketone
at 170ºC.
and HCN is NaCN(aq), which is in the
(d) K cannot show geometrical isomerism since
same phase as the reactants.
there are two identical groups (–CH3) on one
A and C: Catalyst used is Nickel metal, which is
of the carbon atoms of the C=C bond.
a solid.
6. �����������������������������������������������
(a) Phenol is more acidic than water, which in D : UV light is required to initiate the
turn is more acidic than ethanol. reaction by producing Cl•, which is
The electron-donating ethyl group in gaseous. Reactant is a liquid.
ethanol intensifies the negative charge 3. B
on the ethoxide ion, as compared to the When cortisone is first reacted with H2(g) / Pt,
hydroxide ion (from water). This destabilises the alkene (C=C) groups is hydrogenated to
the ethoxide ion, making it a stronger base form alkanes, while the ketone (C=O) groups
than the hydroxide ion. This makes ethanol are reduced to the corresponding 2º alcohols.
a weaker acid than water. When the product is oxidised by warm acidified
The p-orbital of oxygen overlaps with the KMnO4, both the 1º and 2º alcohols are oxidised
π-electron cloud of the benzene ring. The to carboxylic acids (–COOH) and ketones (C= O)
negative charge on oxygen is delocalised respectively.
into the benzene ring and this stabilises the The final product has 4 double bonds as
phenoxide ion, making phenol a stronger shown:
acid than ethanol and water. O O
CH3 OHC C OH
(b) Dilute nitric acid, room temperature. O
(c) 4-nitrophenol is likely to be more acidic CH3
than phenol due to the presence of the
electron-withdrawing –NO2 group on the O
para-position, which further disperses the
charge and stabilises the 4-nitrophenoxide 4. A
ion further. Aldehyde (–CHO) is reduced to 1º alcohol by
NaBH4. Alkene (C=C) functional group is not
(d) Chlorination: reduced.
Reagents: Cl2(aq) 1º alcohol is oxidised to carboxylic acid (–COOH)
Conditions: Room temperature by K2Cr2O7/H+ but not alkene (C = C) functional
group .
NaBH4
CH2=CHCH=O in CH3OH,
CH2=CH–CH2OH
then H2O P
K2Cr2O7/H+
heat

CH2=CHCO2H
Q

Answers to A Level H2 Topical Chemistry A62


© Singapore Asia Publishers Pte Ltd Topic 10.4/Topic 10.5

H2 Topical Chemistry 2014.indb 62 3/27/2014 1:57:40 PM


5. D 9. A
Following are compounds that can give tri- H3C H H H O CH3 H H H O
iodomethane on warming with aqueous alkaline Br2(aq)
C = C – C – C – C – CH3 H3C – C –*C – C – C – C – CH3
iodine:
H3C H H OH Br H H
Alcohols:
(chiral centre marked *)

Common Mistakes
B – Prolonged heating with acidified conc. KMnO4 produces
Carbonyl Compounds: (CH3)2C = O and HOOCCH2CH2COCH3
C – Reduction by NaBH4 on the carbonyl group produces
(CH3)2C = CHCH2CH2CH(OH)CH3 which is C8H16O.
D – Compound does react with alkaline aqueous I2 but it
D is an ester and so does not give CHI3 with produces CH3C​O​–2​​ salt instead of CH3CO2H.
alkaline aqueous iodine.
6. A Section B
����
NaBH4 reduces the aldehyde groups (in X and 10. B
Z) and the ketone group (in Y) to form the 1– Propanone will form orange precipitate with
corresponding 1º & 2° alcohol respectively. In 2,4-DNPH.
each carbonyl group being reduced, 2 hydrogen 2– Propanone will form yellow precipitate (of
atoms are incorporated. CHI3) with alkaline aqueous iodine, since it
NaBH4 has no effect on C=C bond (alkenes). O
Since ratio of X : Y : Z is 2 : 2 : 1; has the group CH3 C .
∴ Total no. of H atoms incorporated Common Mistakes
= (2 × 2) + (2 × 2) + (1 × 2) = 10 3 – only aldehydes react with Fehling's reagent.
∴ On average, no. of H atoms per molecule No reaction with both propanone (a ketone) and pentyl
________
10 ___
10 ethanoate (an ester).
= ​ 2 + 2  
+ 1 ​ 
= ​  5 ​ = 2.0

Common Mistakes 11. B


Students tend to think that NaBH4 reduces the C=C bond 1 Chiral centres are marked *.
and add 2 hydrogen atoms to each C=C bond. O CH2OH
CH3C OH
7. D HO *
CH3 H * *
2,4-DNPH is used to test for the presence of * *
carbonyl compounds. * H *H
Alcohols and carboxylic acids react with sodium O
to release hydrogen gas. 2 With excess hot conc. KMnO4, the 1º alcohol
X – Contains 1º alcohol and carboxylic acid group, 2º alcohol group and the C=C bond
groups that react with sodium. will all be oxidised to give the following:
Y – Contains carboxylic group that reacts
with sodium. Aldehyde group reacts with O CO2H
O CH3C OH
2,4–DNPH.
Z – Contains carboxylic groups that react with CH3 H
sodium. O
H H
HO2C O
8. D
Sodium borohydride NaBH4 is a reducing agent 3 With hot acidified K2Cr2O7, only the alcohol
and can efficiently (selectively) reduce ketones groups are oxidised.
into secondary alcohols, without affecting the
O CO2H
C=C bonds. C
O CH3 OH
Common Mistakes
CH3 H
H2/Ni can also reduce ketone into secondary alcohols, but
it would also hydrogenate C = C bonds and convert them H H
into saturated alkanes. O

Answers to A Level H2 Topical Chemistry A63


© Singapore Asia Publishers Pte Ltd Topic 10.5

H2 Topical Chemistry 2014.indb 63 3/27/2014 1:57:40 PM


12. C Cu2+ ions in Fehling’s solution is reduced
1 A ketone will produce this product. to Cu2O red ppt.
The ketone is =O 3 – Tollen’s reagent oxidises aldehyde to
2 Involves the reaction of aldehyde group with carboxylic acid (or carboxylate ion). The
CH3O– nucleophile (from CH3OH) [Ag(NH3)2)]+ in Tollen’s reagent is reduced
O H O H to Ag(s) 'silver mirror'.
CH3O–
C–C=C–C C – C = C – C – OCH3
HO O HO HO 16. A
1 – alpha 1 will undergo halogenation reaction
– H 2O
of alkanes via free – radical substitution with
H
chlorine gas in the presence of light.
O 2 – alpha 1 will undergo substitution reaction
O OCH3
3 Involves the reaction of aldehyde group with to form halogenoalkanes in the presence of
phenylhydrazine. SOCl2. The –OH groups will be replaced by
H Cl atoms.
H 3C 3 – alpha 1 will undergo reduction reaction with
C = O + H2 N – N –
H H2/Pt. The ketone group ( 0) will be reduced
– H2O to form secondary alcohol.
H
H 3C PAPER 2
C=N–N– 1. (a) LiAlH4 in ether ( or NaBH4)
H

13. B Tips
1 – Nitrile Reduction Reaction: A ketone is reduced to 2º alcohol.
CH3CN D /Pt CH3CD2ND2 2

2 – Reduction of Ketone to 2º Alcohol: (b) (i) reagent: 2,4-dinitrophenylhydrazine


O D OD observation: y e l l o w p r e c i p i t a t e
D2/Pt formed.
(ii) reagent: Sodium metal
14. A observation: Effervescence of H2 gas.
ketone
Give a 'POP' sound when
CH3 O
tested with a burning
wooden splinter.
Tips
Alternative answer:
reagent: neutral FeCl3 solution
HO phenol observation: violet/purple colouration

1 – Oestrone has a ketone functional group and (iii) reagent: Aqueous bromine solution
will react with 2–DNPH to give an orange observation: D e c o l o u r i s a t i o n o f
precipitate. reddish-brown aqueous
2 – The phenolic –OH functional group is acidic bromine solution.
and will react with Na metal to give H2(g). 2. ������������������������������
(a) (i) Nucleophilic addition.
3 – The phenolic –OH functional group also
(ii) Hydrolysis.
reacts with ethanoyl chloride (an acyl halide)
(b) OH
to form a phenate ester.
CH3 – C – H
15. A
1 – Baeyer’s reagent oxidises alkene to diol.
Thus itself being reduced from MnO4– to (c) Product in step I: Product in step II:
MnO2.
2 – Fehling’s solution oxidises aldehyde to O O
carboxylic acid (or carboxylate ion). The CH3CH2CH2C – OMgBr CH3CH2CH2C – OH

Answers to A Level H2 Topical Chemistry A64


© Singapore Asia Publishers Pte Ltd Topic 10.5

H2 Topical Chemistry 2014.indb 64 3/27/2014 1:57:41 PM


3. (a) (i) reagent: HCN + trace amount of NaOH (c) Molecule A:
(or NaCN) as catalyst The chirality of the two assymmetric carbon
conditions : cold, 10 – 20oC atoms (i.e. chiral carbons) does not cancel
type of reaction : nucleophilic addition and this molecule is a chiral molecule. It is
(ii) Step 1: Generation of nucleophile, optically active and it rotates the plane of
CN– ion polanised light to the right.
HCN being a weak acid is a poor source Molecule B:
of CN– ion. Hence, a trace amount
The chirality of the two assymmetric carbon
of Na+CN– is added to accelerate the
atoms (i.e. chiral carbons) does not cancel
reaction.
and this molecule is a chiral molecule. It is
Na+CN– → Na+ + CN–
optically active and it rotates the plane of
Step 2: Nucleophile CN– adds to the polarised light to the left.
electron-deficient C atom of the
carbonyl group (slow step) Molecule C:
H The chirality of the two assymmetric carbon
H H3C
δ+ δ– atoms (i.e. chiral carbons) cancels and this
slow C O–
C O molecule is not optically active. It has no
CH3 CN –
CN effect on plane polarised light since the
tetrahedral angle of rotation to the left by one of the
intermediate
Step 3: The intermediate will be chiral carbon is the same as the angle of
protonated by attacking an undissociated rotation to the right by the other carbon
HCN molecule, to form the cyanohydrin atom. There is an internal cancellation of
product (fast step) optical activity.
H CH3 Tips
H3C
fast
C O –
H CN H C OH +  CN –
CO2H CO2H CO2H
(regenerated)
CN CN H C* OH HO C* H H C OH
cyanohydrin
(b) (i) It is optically active. HO C* H H C* OH H C OH
(ii) CH3CHO molecule is planar and so the
nucleophile CN– has an equal chance CO2H CO2H CO2H
of attacking from above or below the molecules A and B molecule C
plane of the molecule. (chiral molecules) (achiral molecule)
CH3

CN– H C CN (+) isomer PAPER 3


H3C 1. • Reaction II is Oxidation Reaction:
OH
C=O mirror line – H e a t u n d e r r e f l u x w i t h a c i d i f i e d
OH K2Cr2O7(aq).
H
CN– C CN (–) isomer • Reaction III is Reduction Reaction:
H
– LiAlH4 in dry ether.
CH3
2. ����������������������������
(a) The three carbonyls are:
This results in the formation of an O O O
equimolar mixture of (+) and (–) H–C , CH3C & CH3–C–CH3
isomers known as a racemic mixture. H H
This is characteristic of the chemical Mr = 30 Mr = 44 Mr = 58
synthesis of chiral compounds because Ratio: 1 : 2 : 1
chemical reagents tend not to be
stereospecific. Hence, chiral compound Tips
shows optical activity if extracted 2(CH3COO)2Ca + 2(HCOO)2Ca
↓ heat
from natural sources but shows no
(CH3)2CO + HCHO + 2CH3CHO + 4CaCO3
optical activity if synthesised in the
laboratory.

Answers to A Level H2 Topical Chemistry A65


© Singapore Asia Publishers Pte Ltd Topic 10.5

H2 Topical Chemistry 2014.indb 65 3/27/2014 1:57:41 PM


(b) 1. Tollen's Reagent Test: 4. ��������������������������������������������
(a) Both aldehydes and carboxylic acids can
Add Tollen's Reagent to the three samples be prepared by the oxidation of 1º alcohols
of carbonyl compounds separately and with acidified K2Cr2O7.
warm. To ensure that only aldehyde is obtained:
HCHO and CH3CHO are aldehydes and An excess of 1º alcohol (over the oxidising
will show a silver mirror, but not (CH3)2CO agent) is used and the aldehyde formed is
which is a ketone. distilled off immediately as it is formed, to
2. Tri-iodomethane (Iodoform) Reaction prevent further oxidation.
Test:
Add aqueous iodine and aqueous NaOH, To ensure that only carboxylic acid is
each to the remaining two compounds obtained:
separately and warm the mixture. The 1º alcohol is heated under reflux with
CH3CHO will form a yellow precipitate the oxidising agent for some time to achieve
whereas HCHO will show no yellow complete oxidation to carboxylic acid.
precipitate. (b) Structure for compound L (C9H10) is:
CH = CH2
Tips
CH3
Only carbonyl compounds with CH3CO– group gives
yellow precipitate of CHI3 crystals when warmed with Structure for compound M (C9H10) is:
alkaline aqueous iodine.
CH2
CH2
3. �����������
• Reaction: CH2
OH
CH3 CH2 Structure for compound N (C9H12O) is:
C = O + HCN CH3CH2 – *C – H CH3
H
CN
CH(OH)CH3
Optical isomerism takes place since the
product has a chiral carbon (marked *) Tips
CN CN The benzene-1,2-dicarboxylic acid P is formed by the
* * oxidation of side chain of benzene ring. The structure of
C C P suggests that L, M and N have alkyl-type side chains on
H OH HO H
C-1 and C-2 of the benzene ring.
CH2CH3 CH2CH3
L reacts with Br2(aq) but not M:
Reaction: → suggests that L has C=C bond while M does not.
OH N reacts with alkaline I2(aq):
hot → suggests that N has CH3CH(OH)– group.
CH3CH2 – C – H Al2O3
CH3CH = CHCN

CN 5. ���������������������������������������������������
(a) A positive result of this test is indicated by
Dehydration of the alcohols produce isomers the formation of a red precipitate of copper(I)
F and G that shows cis-trans isomerism. oxide, Cu2O.
H H H CN
C=C C=C Tips
CH3 CN CH3 H This is a test for aldehydes functional group using Fehling's
cis-isomer trans-isomer Solution, which is an alkaline solution of complex copper(II)
ions, that is deep blue in colour. Only aldehydes give a red
• Each of the two optical isomers D and E produce precipitate. Benzaldehyde and ketones have no reaction
the same mixture of cyanoalkenes because with Fehling's Solution.
dehydration occurs for both isomers.
Both isomers can lose a H and an OH from (b) Aldehydes will give positive result:
adjacent C atoms in two different ways, thus H H H H O
giving the cis and trans isomeric compound.
H–C–C–C–C–C–H
H H H H

Answers to A Level H2 Topical Chemistry A66


© Singapore Asia Publishers Pte Ltd Topic 10.5

H2 Topical Chemistry 2014.indb 66 3/27/2014 1:57:42 PM


& (iii) Both isomers J and K are diprotic
H H H O
acids.
H–C–C–C–C–H Isomer K which has the larger difference
H H
in pKa1 and pKa2 values will produce the
H–C–H more stable mono-anion on treatment
with 1 mol of NaOH.
H
Ketones will give negative result: After dissociation of a proton (pKa1),
it is more difficult for the mono-anion
H H O H H
to dissociate the 2nd proton. Thus the
H–C–C–C–C–C–H large difference in pKa1 and pKa2.
H H H H (iv) Hydrogen Bonding
(Intramolecular)

or H H H O H OH O
H–C–C–C–C–C–H O=C C=O
H H H H C=C
H H
Common Mistakes
cis-isomer (K)
Remember to show the Displayed Formulae, by showing
both the relative placing of atoms and the number of bonds When 1 proton is removed from
between them.
the cis-isomer (isomer K), a strong
intramolecular hydrogen bonding is
6. (a) formed with the nearby remaining
O
carboxyl group, which stabilises the
Br bromoethanal
Reagent: Ethanolic KCN
mono-anion. This opposes the removal
step 1 of 2nd proton from the species.
Condition: Heat with reflux
O For the trans-isomer (isomer J), the
NC G two carboxyl groups are far from each
Reagent: HCN with trace KCN (or NaOH) as catalyst other and thus, such intramolecular
step 2
Condition: In the cold, 10 – 20oC hydrogen bonding does not occur.
CN (c) �����������������������������������������
• L does not undergo acid-metal reaction
NC H with Na metal
OH → L does not contain alcohol / carboxylic
Reagent: H2SO4(aq) group.
step 3
Condition: Reflux • L does not undergo condensation reaction
O OH with 2, 4-DNPH
OH → L does not contain carbonyl group
malic acid
HO • L undergoes nucleophilic substitution to
O form M with NH3.
Tips → L contains anhydride which reacts
•  Step 1 – a step-up reaction for halogenoalkanes with NH3 in a condensation ���������
reaction�
•  Step 2 – a step-up reaction for carbonyl compounds to form amide in M.
•  Step 3 – acid hydrolysis to convert –CN group to –COOH H H H H
group
C=C C=C
+ NH3
(b) (i) H H H COOH
O=C C=O O=C C=O
C=C C=C
O OH NH2
HOOC COOH HOOC H L M
cis – butenedioic acid trans – butenedioic acid
Type of isomerism: cis/trans isomerism • M undergoes neutralisation with NaOH
to form a salt
(ii) Reagents: excess concentrated H2SO4 → M contains an acidic group, probably
Conditions: heat at about 170oC –COOH.

Answers to A Level H2 Topical Chemistry A67


© Singapore Asia Publishers Pte Ltd Topic 10.5

H2 Topical Chemistry 2014.indb 67 3/27/2014 1:57:42 PM


H H H H
Tips
C=C C=C Mechanisms:
+ NaOH
O=C C=O O=C C=O δ+
H H H H
OH NH2 O Na
– +
NH2 O Oδ– –
O O+
M O=C C=O O=C C=O
• M does not neutralise HCl to form a C=C C=C
salt
H H H H
→ M does not contain a basic group i.e. J
–NH2 amine group. O H
H H
→ ∴ It could be an amide group. O=C δ+
C=O O +
N
• M undergoes electrophilic addition with H δ- O=C C
O–
C=C N–
Br2(aq) H C=C
H H H
→ M contains an alkene group.
L H H
H H Br OH
H OH NH2
H
C=C H–C–C–H
+ Br2(aq) O+ O=C C=O
N–H
O=C C=O O=C C=O O=C C
O– C=C
OH NH2 OH NH2 C=C
H H
or H H M
OH Br

H–C–C–H (d) O O O
OH OH
O=C C=O O=C=O
HO
OH NH2 O O
Compound N Compound P Gas Q
H H H H
Tips
C=C C=C O OH
Hence; O=C C=O O=C C=O OH
HO
O OH NH2 O
L M malic acid
[Oxidation] 2 alcohol converted
o

Compound L must have been formed from • acidified K2Cr2O7 to ketone


isomer K as only the cis alkene can form a stable • heat
O O • reacts with 2,4–DNPH due to
5-membered ring structure. C = O group
OH
HO
O
Compound N
Heat several hours
with inert solvent
O • reacts with 2,4–DNPH due to
OH C = O group
• reacts with aqueous alkaline
I2 due to CH3CO– group
O
Compound P
+
O=C=O
Gas Q

Answers to A Level H2 Topical Chemistry A68


© Singapore Asia Publishers Pte Ltd Topic 10.5

H2 Topical Chemistry 2014.indb 68 3/27/2014 1:57:44 PM


Alcohols are not acidic enough to react with
Topic 10.6 Carboxylic Acid &
NaOH.
Derivatives
PAPER 1 5. C
Section A The ester 3-methyl butyl ethanoate can be made
1. C from the corresponding acid chloride and alcohol
CH3COCl reacts with the phenol functional group shown below:
to form an ester.
OH OCOCH3
OCH3 OCH3
+ CH3COCl

CH2 – CH = CH2 CH2 – CH = CH2

2. C
Acid hydrolysis of ester X gives a carboxylic
acid and an alcohol.
The yellow precipitate obtained by reacting
with alkaline aqueous iodine (Iodoform Test)
suggests that the alcohol contains a CH3CH(OH)– 6. C
group. Formula of nitrile is C2H5CN.
Hence, X is (CH3)2CHCO2CH2CH3. Acid hydrolysis of a 3C atoms nitrile will
O form a carboxylic acid with 3C atoms i.e.
(CH3)2CHC CH3CH2CO2H.
OCH2CH3
Acid Hydrolysis 7. B
H+/H2O
of Ester The reactions involve hydration of alkene,
O followed by acid hydrolysis of ester.
(CH3)2CHC + CH3CH2OH OH
OH O O
hydration
CH3CH2OH + 4I2 + NaOH O O x
↓ or
Iodoform Test
CHI3 + HCO2Na + 5HI O OH
yellow ppt
O y
3. C
Most Acidic Least Acidic O OH O
acid
+ HO OH
Chloroethanoic > bromoethanoic > propanoic > phenol O hydrolysis OH
acid acid acid y
CH2ClCOOH > CH2BrCOOH > CH3CH2COOH > C6H5OH
(2) (1) (4) (3)
8. B
Halogen increases the acidity of an acid by
Acid chloride (W) is the most susceptible to
its electron – withdrawing ability. Chlorine
hydrolysis due to its highly electron deficient
is more electronegative than bromine and
thus chloroethanoic acid is more acidic than carbonyl carbon, that is bonded to two
bromoethanoic. electronegative O and Cl atoms.
Propanoic acid is the least acidic among the Chlorobenzene (X) is the least susceptible to
three carboxylic acids due to the presence of an hydrolysis due to partial double bond character
electron – donating ethyl (–CH2CH3) group. of C–Cl, due to overlapping of p-orbital of Cl
Phenol is a very weak acid as compared to with the � orbitals of the benzene ring.
carboxylic acid. Both iodoalkanes & bromoalkanes are normal
4. B halogenoalkanes but iodoalkane (Z) is more
Only the acidic groups –COOH and phenolic –OH susceptible to hydrolysis than bromoalkane (Y)
will react with NaOH to give the corresponding due to a weaker C–I bond (as compared to stronger
sodium salt. C–Br bond).

Answers to A Level H2 Topical Chemistry A69


© Singapore Asia Publishers Pte Ltd Topic 10.6

H2 Topical Chemistry 2014.indb 69 3/27/2014 1:57:44 PM


9. A 1 –
O N COCl + HOCH2CH3
CH3
C – Cl CO
NH
x: CH2Cl y: CH2 N COOCH2CH3 + HCl
CH3 (as by-product)

Tips
Common Mistakes
• Alcohols react with PCl 5 to give chloroalkanes 2 – Esterification occurs but product (ester) is not
(–CH2Cl) pethidine.
• Carboxylic acids react with PCl5 to give acid chlorides​

(  )
O 3 – No esterification reaction.
– C – Cl   ​
• Chloroalkanes react with NH3 to form amines (–NH). PAPER 2
• Acid chlorides react with NH 3 to form amides 1. ����������������������������
(a) (i) Stage I : Hydrolysis
(–CONH).
Stage II: Elimination Reaction
(ii) Mechanism: Nucleophilic Substitution
10. C
OH– acts as the nucleophile.
Phenol groups ( OH ) are slightly acidic and

will react with the alkali, NaOH(aq). – + O O
Carboxylic acids (–COOH) will also react with
alkali, NaOH(aq)
Cl – C
Cl O​
·· 
​ H–
slow
Cl C OH
O Cl
Esters (– C – O –) are readily hydrolysed under
reflux with alkali, NaOH(aq), to form an alcohol fast
and a sodium salt of the carboxylic acid. O
C – OH + Cl–
Section B Cl
11. C (b) O
2 – Salicylic acid is obtained by acid hydrolysis CH3CH2O – C – OCH2CH3
of both ester groups (c) (i) Ammonia
3 – Salicylic acid is obtained by acid hydrolysis (ii) Lone pair of electrons on each of the N
of both nitrile (–CN) group and ester atoms can accept H+ (from the strong
group. acid).
(d) (i) 4CO(NH2)2 + 6NO2 → 4CO2 + 8H2O + 7N2
Tips (ii) NO2 in the atmosphere will dissolve in
1 – The aryl chloride group cannot be hydrolysed because water vapour in the air to form nitric
the aryl–Cl bond is strengthened by the overlapping of acid, which contributes to acid rain.
p-orbital of Cl with the � orbitals of the benzene ring.
Final product of acid hydrolysis is: COOH 2. ������������������������������������������������
(a) Benzoic acid and ethanoic acid are stronger
than carbonic acid.
Cl
Tips
A stronger acid has a larger Ka value.
12. D
The reactants to produce pethidine can be (b) C6H5OH(aq)  C6H5O–(aq) + H+(aq)
determined from the products of hydrolysis of
[C6H5O–][H+]
___________
the ester compound: Ka = ​  [C H OH]
    ​  
6 5
N COOCH2CH3 Since phenol is a weak acid, the degree of
CH3
dissociation is very small;
∴ [C6H5OH]eqm  [C6H5OH]initial
[H+]2
_________
from a carboxylic acid from an alcohol ∴ Ka = ​ [C H OH]  ​ 
or an acyl chloride 6 5
_______________
[H+] = √
​ (1.3 × 10–10)(0.10) ​
  
= 3.605 × 10–6 mol dm–3

Answers to A Level H2 Topical Chemistry A70


© Singapore Asia Publishers Pte Ltd Topic 10.6

H2 Topical Chemistry 2014.indb 70 3/27/2014 1:57:45 PM


∴ pH = –log10[H+] (iii) A and D.
= –log10(3.605 × 10–6)
Tips
= 5.44
Change of molecular formula from C9H8O3 to C9H9O3Br
(c) Phenol is a stronger acid than methanol and
suggests that HBr is added to an unsaturated compound
thus a larger Ka value. to form an addition product. Therefore, C=C bond is
This is because the phenoxide ion (C6H5O–) present.
produced is stabilised by the delocalisation
of the negative charge into the benzene (iv) A and D.
ring, while the methoxide ion (CH3O–) is
not stabilised. Tips
(d) Observation: Reddish-brown Br 2 (aq) The replacement of two H atoms with two Na atoms in the
decolourised, steamy white molecular formula suggests that two functional groups has
fumes of HBr evolved and a reacted with Na. A & D have alcohol and carboxylic acid
groups both present which can react with Na.
white precipitate is formed.
Structural formula of product: (v) D.
OH
Br Br Tips
(white ppt) D has both the carboxylic acid and phenol functional groups
to react and neutralise the NaOH added, as shown in the
Br molecular formula of product with 2 Na atoms.
(e) (i) Heat benzoic acid with methanol
under reflux in the presence of a little Common Mistakes
amount of concentrated sulfuric acid Many students tend to include A as the answer which is
as catalyst. incorrect.
A will only react with 1 mole of NaOH with its carboxylic
Tips acid functional group.
conc.H2SO4 Its alcohol functional group in the side chain (–CH=CHOH)
C6H5CO2H + CH3OH C6H5CO2CH3 + H2O
heat under is not acidic enough to react with NaOH.
reflux methyl benzoate
(an ester)
(b) (i) Reaction (a)(v).
(ii) Ethanoic acid is first converted to (ii) O–Na+
ethanoyl chloride by adding SOCl2 (or
PCl5).
Ethanoyl chloride is then reacted with
phenol to give phenyl ethanoate. CH = CHCO​ –2​N
   a+
(c) CO​ –2​  
Tips
CH3CO2H + SOCl2 → CH3COCl + SO2 + HCl
CH3COCl + C6H5OH → CH3CO2C6H5 + HCl
CO​ –2​  

Tips
Common Mistakes
Students tend to make the mistake of reacting phenol with Product formed is an ionic salt of benzene-1, 4-dicarboxylic
ethanoic acid directly. acid (and not the benzene-1, 4-dicarboxylic acid itself) since
Phenate esters cannot be prepared by esterification of a oxidation reaction is carried out in an alkaline medium.
carboxylic acid with a phenol. Ethanoyl chloride is needed.
(d)
3. ��������������������
(a) (i) A, C and D. A produces D produces
CO​ –2​N
   a+ OH
Tips
No silver mirror with Tollen's reagent suggests aldehyde
group is not present.

(ii) A and D. CH = CHOH CH = CHCO2–Na+

Tips Tips
No ppt with 2,4-DNPH suggests that carbonyl group is Phenol has no reaction with NaHCO3.
not present.

Answers to A Level H2 Topical Chemistry A71


© Singapore Asia Publishers Pte Ltd Topic 10.6

H2 Topical Chemistry 2014.indb 71 3/27/2014 1:57:45 PM


4. ��������������������
(a) (i) Basis: 100 g
Element C H O CO2 dissolves in water to form carbonic
Mass (g) 70.6 5.9 23.5
acid, H2CO3(aq), which is a stronger acid
than phenol but a weaker acid than both
Ar 12 1 16
ethanoic and benzoic acids.
Molar 4 4 1 Hence, only sodium phenoxide reacts with
Ratio
H2CO3 to give phenol.
∴ Empirical formula of E is C4H4O. Sodium ethanoate and sodium benzoate have
(ii) Using the Ideal Gas Equation, no reaction with H2CO3.
__
m
PV = nRT = ​ M   ​RT 5. (a) (i) Other functional groups present in
r
chlorogenic acid are:
_____
mRT
∴ Mr of E = ​  PV ​  •  ester linkage
(0.34)(8.31)(21 + 273) •  carboxylic acid group
_____________________
= ​    
   ​ •  phenol group
(101.7 × 103)(60.4 × 10–6)
•  alcohol group
= 136.8 (ii) A chiral carbon atoms (marked *)
Mass of empirical formula O
= (12)(4) + 4 + 16 = 68 HO
∴ 68n = 136.8 * OH
O
n  2 * *
HO * O
 ∴ Molecular formula is C8H8O2.
OH
OH
Tips
OH
Temp. must be in K. Volume must be in m3.
(b) O O
HO
(b) (i) OH HO
O O
OH OH
HO
O – C – CH3 C – O – CH3 OH
OH
non-aromatic aromatic compound R
compound
Structure E1 Structure E2
(ii)
E1 produces E2 produces
O O

Na+O– – C – CH3 C – O–Na+


+
+
O–Na+ CH3OH

Tips
In Alkaline Hydrolysis, the ester will hydrolysed to form
an alcohol and a sodium salt of the carboxylic acid, when
it is refluxed with aqueous NaOH.
CH3OH does not react with aqueous NaOH.
–OH is acidic enough to react with aqueous NaOH to
form –O–Na+.

(c)
Mixture from E1 Mixture from E2
produces produces

OH No Reaction

Answers to A Level H2 Topical Chemistry A72


© Singapore Asia Publishers Pte Ltd Topic 10.6

H2 Topical Chemistry 2014.indb 72 3/27/2014 1:57:46 PM


Compound S
Tips
Tips
Alkaline Hydrolysis of Esters:
Reagent: NaOH(aq)
HO
Condition: Reflux
O excess
OH    +
HO NaHCO3(aq)
OH chlorogenic acid OH
O
Compound R
HO O
OH O
OH
OH Na+O–
(Alkaline NaOH(aq)
Hydrolysis) Reflux OH
O O OH
HO Compound S
O–Na+ HO (Molecular Formula is C9H7O4Na)
+
HO OH O–Na+
OH O–Na+ Common Mistakes
H+(aq) (acidification) H+(aq) Note that OH groups are not acidic enough to react
with NaHCO3(aq).
O O
HO
OH
+ HO (ii) There is only one –COOH group in
compound R to react with NaHCO3.
OH
HO OH
OH
(i.e. only 1 H atom from the –COOH
OH group can be replaced by Na atom to
non-aromatic compound aromatic compound R
form the salt).
(c)
Electrophilic Addition Reaction PAPER 3
O (C = C group) 1. ��������
(a) (i) Reaction I:
HO Reagent : HCN with trace amount
of NaOH (or NaCN) as
OH Electrophilic Substitution catalyst
OH Reaction ( OH group)
Conditions : In the cold (10 – 20ºC)
Excess Br2 (organic solvent) (ii) Reaction II:
Reagent : HCl(aq) or H2SO4(aq)
O Br Br Conditions : Heat
Br (b) (i) Ester functional group
HO
Br
(ii) Hydrolysis
Br OH
Tips
OH
• Hydrolysis is the reverse of esterification.
Tips • Esters are readily hydrolysed by heating under reflux
Note that excess Br2(organic solvent) is used. As such, Br with acid or base.
atom will be substituted at both positions 2 as well as 4 of
the phenol groups. (c) (i) Reaction III:
–OH group is electron-donating (i.e. 2–, 4–directing)
Reagents : NaOH(aq)
and activates the aromatic ring towards electrophilic
substitution. Conditions : H e a t , f o l l o w e d b y
acidification with
(d) (i) O H2SO4(aq).
Na+O–

OH
OH

Answers to A Level H2 Topical Chemistry A73


© Singapore Asia Publishers Pte Ltd Topic 10.6

H2 Topical Chemistry 2014.indb 73 3/27/2014 1:57:46 PM


(ii) Step VI
O O Reagent: KMnO4(aq)/H2SO4(aq)
CH3CHC – N – C2H5 Condition: heat under reflux
CH3CHC – N – C2H5
Step VII
Cl H NH2 H Reagent: PCl5 (or SOCl2)
neutral product K L Condition: room temperature
Step VIII
In the dichloride J, the chlorine atom
Reagent: NH3(g)
attached to C=O (i.e. acyl chloride) is Condition: room temperature
much more reactive than the chlorine
atom in C–Cl halogenoalkane bond, Compound C: Compound D:
due to the electron-withdrawing effect COOH O
of C=O group. C – NH2
(d) Test 1:
Br
– Add 2, 4-dinitrophenylhydrazine to each
compound separately and warm. Br
– Compound M forms an orange
3. ���������������������������������������������
(a) Malonic acid has an electron-withdrawing
precipitate group –CH2CO2H (as compared to the
– No such precipitate is observed with electron-donating group –CH3 in ethanoic
compound N. acid).
Test 2: The electron-withdrawing group increases
– Add sodium metal. the polarisation of the O–H bond in
– Compound N gives effervescence of carboxylic acid functional group, causing
H2(g). it to weaken and hence dissociates readily.
– Compound M does not give any In addition, the electron-withdrawing group
effervescence. stabilises the carboxylate ion by reducing
the negative charge on the O atom.
Tips
– Compound M has aldehyde and ketone functional Tips
groups.
– Compound N has carboxylic acid and alcohol functional Smaller the pKa value, the more acidic is the carboxylic
groups. acid.
• Tollen's Reagent, Fehling's solution and Na2CO3 tests
can also differentiate the two compounds. (b) –O2CCH2CO​ –2​  ion is much more negatively
charged (2–) as compared to CH3CO​– 2​  
2. (a) (i) R–CONH2 + Br2 → R–CONHBr + HBr ethanote ion (1–), which makes it more
(ii) +1 likely to accept a proton back to form
HO2CCH2CO​– 2 ​. 
Tips
Br has replaced H, and since N is more electronegative Tips
than Br, oxidation state of Br = +1 CH3CO2H is a stronger carboxylic acid than HO2CCH2CO2–.
Hence; pK2 value of malonic acid is higher than the pK1
value of ethanoic acid.
(iii) 6 electrons

Tips (c) (i) O=C=C=C=O C3O2


H and Br will each take away one electron from their Linear shape
bond pairs to form HBr, leaving behind 8 – 2 = 6 valence (ii) Reaction with NH3(g):
electrons around N atom.
H2NOCCH2CONH2
(iv) The other product is CO32–. Reaction with HCl(g):
R–N=C=O + 2OH– → R–NH2 + CO32– ClOCCH2COCl
(b) Step V
Reagent: FeBr3, Br2
Condition: in the dark

Answers to A Level H2 Topical Chemistry A74


© Singapore Asia Publishers Pte Ltd Topic 10.6

H2 Topical Chemistry 2014.indb 74 3/27/2014 1:57:47 PM


(d) (i) To form HO2C–CO2H:
Topic 10.7 Nitrogen Compounds
BrCH2CH2Br
PAPER 1
• NaOH(aq) Section A
• Heat 1. C
HOCH2CH2OH N-methyl-D-aspartic acid has no hydroxy (–OH)
group and cannot react with ethanoyl chloride to
• Acidified K2Cr2O7(aq) form an ester.
• Heat
Tips
HO2C–CO2H
A – It is optically active due to the presence of a chiral
To form HO2C–CH2CH2–CO2H: carbon.
B – The carboxyl (–CO2H) group reacts with ethanol to
BrCH2CH2Br form an ester.
D – It reacts with PCl5 to from an acyl chloride.
• Ethanolic KCN
CO2H COCl
• Heat + PCl5
CH2 CH2
NCCH2CH2CN
CH3NHCHCO2H CH3NHCHCOCl
• H2SO4(aq)
• Heat
2. B
HO2C–CH2CH2–CO2H
Phenylamine, C6H5NH2, is a weak base.
Its conjugate acid, C6H5NH3+Cl– will be a stronger
acid.
Base strength: C2H5NH2 > NH3 > C6H5NH2
Conjugate Acid Strength:
C2H5NH3+Cl– < NH4+Cl– < C6H5NH3+Cl–

Tips
K+Cl– is a neutral salt made from a strong acid (HCl) and
a strong base (KOH).

3. D
At pH 10 (alkaline condition), the acidic –CO2H
groups will ionise to produce H+ ions to neutralise
the OH– ions present, to give the corresponding
carboxylate anion.
4. C
At pH = 2 (acidic), the basic –NH2 group would
react with the H+ ions:
NH2 NH3+
CH2 – CH CH2 – CH
CO2H CO2H
+ H +
(from acid)
NH2 NH3+

5. D
Proteins in the body are built from 2-aminocarboxylic
acid (or -aminocarboxylic acid).
D is a 3-aminocarboxylic acid.
12 3
HO2CCH2CH(CH3)NH2

Answers to A Level H2 Topical Chemistry A75


© Singapore Asia Publishers Pte Ltd Topic 10.6/Topic 10.7

H2 Topical Chemistry 2014.indb 75 3/27/2014 1:57:47 PM


6. C Protonation of amine.
Amines are bases and thus forms an ionic salt Acidic hydrolysis of amide and ester groups
with dilute aqueous acid, which will then form to form protonated amine and carboxylic acid
an immiscible mixture with benzene. respectively.
The organic and inorganic mixture can then be 12. C
separated using a separating funnel. The α-helix of proteins is stabilised by hydrogen
7. C bonds between the N-H group of one amino acid
Insulin will undergoes acid hydrolysis when it is unit and the C = O group of another along the
heated with HCl for prolonged period. main chain.
The peptide (–CONH–) linkage will break up 13. B
to give corresponding carboxylic acids and After refluxing nitrobenzene with tin and
amines. OH concentrated HCl (in excess), the product
appears as a salt, C6H5NH3+Cl–. Excess aqueous
NaOH is added to liberate the free phenylamine,
which is separated from the mixture by steam
CH2OH CH2 distillation.
– CONHCHCONHCHCONHCHCONHCHCONH – 14. B
The resonance structure in amides involves the
CH CH2
movement of a pair of electrons from the nitrogen
CH3 CH3 CH2CONH2 atom to the oxygen atom on the carbonyl group.
∴ The products of acid hydrolysis are: This reduces the electron density on the nitrogen
CH2OH +
H3NCHCO2H OH +
H3NCHCO2H atom, making it less effective as a proton acceptor.
In fact, amides are not bases. Solutions of amides
H3NCHCO2H
+
CH CH2 are neutral.
CH3 CH3 δ–
CH2CO2H O
CH2 δ +
– C
+
H3NCHCO2H N H
H
Common Mistakes
There are 6 breakage of peptide links. Common Mistakes
However, many students fail to realise there is one at the
In the resonace structure, it is true that a pair of electrons
side-chain. Thus, they will only find 5 breakage and hence,
on the nitrogen atom moves onto the C-N bond. However,
miss out on the answer Option C.
basicity has nothing to do with breaking of double bond
character. As such, option C is incorrect.
8. B
Hydrolysis of small peptide X gives 5 amino acids
15. A
suggests that X has 4 peptide linkage (–CONH–)
Aryl amines undergo electrophilic substitution
i.e. 4 mol of H2O are lost in the forming of peptide with aqueous bromine at room temperature and
X. tri-bromination occurs at the position 2, 4 and 6
∴ Mr of X = 2(75) + 89 + 2(105) – 4(18) of the aromatic ring (provided the positions are
= 377 not taken up by other groups). The amine group
9. D does not react with aqueous bromine.
Trypsin catalyses the hydrolysis of protein and NH2 NH2 NH2
converts it into peptides and amino acids. CH3 Br + CH3 Br 1 CH3
6 2
5 3
10. B 4

At pH 7, major species of glutamic acid should 2-methylphenylamine H Br Br


contain two basic groups and one acidic group. Z

11. B Note:
Br group is not shown at position 2 of the final
OR
CONH product since it is already occupied by –CH3
H3C group. As such, only dibromination occurs for
H 2N CO2CH2CH3
2-methylphenylamine.

Answers to A Level H2 Topical Chemistry A76


© Singapore Asia Publishers Pte Ltd Topic 10.7

H2 Topical Chemistry 2014.indb 76 3/27/2014 1:57:48 PM


Section B 2 – At pH = pK1, which is the equivalent point of
16. B half-neutralisation in an acid-base titration,
There are 3 functional groups in the capsaicin the [acid] = [salt] i.e. [H3N+CHRCO2H ] =
molecule: [H3N+CHRCO2–].
1. Phenol 1 – H3N+CHRCO2– is indeed the most common
2. Amide species present at the isoelectric point, but
3. Alkene need not be always of pH7. Amino acids
Addition of Br2 in organic solvent will react with with one –CO2H group and one –NH2 group
both phenol and alkene groups. have isoelectric points in the pH range of
Phenol incorporates 1 atom of Br through 5.5 – 6.5, depending on the nature of the
electrophilic substitution. amino acids.
Alkene incorporates 2 atoms of Br through 19. C
electrophilic addition. 1 – It does not contain CH3CO – or CH3CH(OH)–
CH3O CH2NHCO(CH2)4CH – CHCH(CH3)2
groups and thus does not react with alkaline
HO Br Br aqueous iodine.
Br 2 – The amide linkage is hydrolysed to give an
Addition of NaOH will react with both phenol amine and a carboxylate ion.
and amide groups. H
1 mole of NaOH is required for neutralisation
O2N – – CH – CH – N – C – CHCl3
with phenol.
1 mole of NaOH is required for hydrolysis of OH CH2OH O
amide linkage.
CH3O CH2NH2 OH– reflux
+ Na+–OOC(CH2)4CH = CHCH(CH3)2 O
Na+–O 
On heating under reflux with acidified KMnO4, O2N – – CH – CH – NH2 + O – C –CHCl2
C=C bond is cleaved and (CH3)2CHCHO is OH CH2OH
formed initially. It is then further oxidised to
amine carboxylate ion
produce (CH3)2CHCO2H.
3 – Compound is directly oxidised by KMnO4
17. A
to form 4-nitrobenzoic acid, due to benzylic
In all the three amino acids, the presence of N–H
H atom on the carbon that is attached to the
group forms hydrogen bonding and hence, the
benzene ring.
cross-link to stabilise the tertiary structure of a benzylic H atom
protein.
O2N – – CH – CH – NHCOCHCl2
18. C
K1 OH CH2OH
H3N+CHRCO2H + OH–  H3N+CHRCO2– + H2O
K2 [O]
H3N+CHRCO2– + OH–  H2NCHRCO2– + H2O
O2N – – COOH

Common Mistakes Purple KMnO4 is decolorised.


Note that the question ask for 'Which statements are
ALWAYS correct for ANY amino acid?' and not just based
20. D
ester
on the titration curve information provided.
O OCH2CH3

The pH at which an amino acid has a net charge of


(CH3CH2)2CHO * ** NH2
zero is called the isoelectric point. The isoelectric
point of any amino acids can be determined from H C NH
3
Amide
its titration curve. O
3 – The point in the titration curve when the slope (chiral centres are marked *)
is at its maximum in the centre corresponds 21. B
to the isoelectric point. Thus, the species 1 – hydrophobic van der Waals’ attractions occur
present is H3N+CHRCO2– which has no net between the non-polar groups in alanine and
charge. glycine.

Answers to A Level H2 Topical Chemistry A77


© Singapore Asia Publishers Pte Ltd Topic 10.7

H2 Topical Chemistry 2014.indb 77 3/27/2014 1:57:48 PM


2 – hydrogen bonding occurs between the polar PAPER 3
hydroxy groups in serine. 1. ����
(a) O
3 – no ionic bonding in the tertiary structure of
spider silk since all the 3 amino acids does C – OH
not have any charged groups. A=
C – OH
PAPER 2 O
1. �����������������������������������������������
(a) (i) Denaturation is the loss of biological O
function of protein. The secondary
and tertiary structures of proteins are C – Cl
disrupted i.e. R group interactions are B=
C – Cl
broken/destroyed.
(ii) Thermal denaturation. Collagen can be O
denatured to produce gelatin by heating
to above 50 – 60 ºC. Tips
(b) 6 different amino acid residues. A → B: Formation of acyl chlorides from carboxylic acids.
B →C: Formation of amide from acyl chlorides.
Common Mistakes
Must take into account that both the amino acids (–NH2) and (b) Reaction I : Acid-base reaction
imino acids (>NH) are both considered as 'amino acids'. Reaction II : Nucleophilic substitution
Reaction III : Hydrolysis reaction
(c) (i) H O
(c) Heat under reflux with dilute H2SO4(aq)
H–N
H N–C–C–O–H
Tips
H C – OH
H Acid Hydrolysis
O
OR
(ii) OH
Heat under reflux with dilute NaOH(aq)
H–N *
*
C – OH Tips
O Alkaline Hydrolysis
(chiral atoms are marked *)
(d) (i) Proteins can be hydrolysed in the 2. (a) It is the order or sequence of amino acid
laboratory by prolong heating with residues in a polypeptide chain or protein.
concentrated HCl(aq) to obtain the (b) H H O H H O H H O
individual amino acids.
–N–C–C–N–C–C–N–C–C–
(ii) HOOC – – COOH
& H–C–H H–C–H H–C–H
H2N – – NH2
O–H S–H
(iii) In proteins, the monomers are amino
acids (with both –NH2 and –COOH Part of protein chain of HSA using serine,
groups) whereas in polyamides, cysteine and phenylalanine amino acids.
the monomers are diamines and (c) A polypeptide chain is held in the shape of
dicarboxylic acids. an -helix by hydrogen bond form between
(e) the N–H group of each amino acid and the
fourth group of the amino acid along the
chain.
(d) –CH2SH + –CH2SH →
–CH2–S–S–CH2– + 2[H]
Tips
(f) (i) Hydrolysis
The thiol group in cysteine can be oxidised to disulfide
(ii) The products are hydroxyacids which bridges in the presence of oxidising agents such as O2 or
can form hydrogen bonding with water H2O2.
molecules, hence soluble in water.

Answers to A Level H2 Topical Chemistry A78


© Singapore Asia Publishers Pte Ltd Topic 10.7

H2 Topical Chemistry 2014.indb 78 3/27/2014 1:57:49 PM


___
36 2. O
​  2 ​  = 18 disulfide bridges could be made CH2
within each HSA molecule. Na+–O2C CH C O–Na+
(e) Two other types of side-chain interaction: NH2
 Hydrogen Bonding
3.
– CH(OH)CH3 threonine

Hydrogen Bonding CH2


O – CH2 – CH
C O Na
– +
serine H2N
H
O
 van der Waals' Forces
phenylalanine (iii) • With cold HCl(aq):
– CH(CH3 )2 C6H5CH2_ Amine is basic and will react with cold
valine HCl(aq) to form ionic salt.
van der Waals' Forces
or Ionic Bonding between charged groups:
between RCH2CH2CO2– (glutamic acid) CH2
and RCH2CH2CH2CH2NH3+(lysine) O
CH OCH3
C N C
(f) Valine & phenylalanine (or leucine) HO2C CH2 CH
H O
Tips NH3+Cl–
Interact by van der Waals' Forces since they are all
hydrophobic.
• With CH3COCl:
CH3COCl reacts with amine to form an
(g) Glutamic acid, lysine and threonine. amide.

Tips
Interact strongly with water molecules by forming hydrogen CH2
O
bonds with water molecules. CH
C N C OCH3
HO2C CH2 CH
3. (a) H O
R OCH2CH2CH3 N–H

conc. HNO3 C
O CH3
S O2N OCH2CH2CH3 4. (a) The haemoglobin (Hb) molecule is a
NO2
conjugated protein that exists as a tetramer
(i) Sn + small amount of HCl + Heat of four separate polypeptide chains:
(ii) Dilute NaOH(aq) 2 -chains and two -chains, each with its
own iron-containing haem residue.
T O2N OCH2CH2CH3
Primary structure refers to the linear sequence
NH2
or order of amino acids that are covalently
(b) (i) The four functional groups in aspartame, bonded together in each polypeptide
other than the phenyl group are: chain.
1. Carboxylic acid
2. Amine Secondary structure refers to the detailed
3. Amide configurations of the polypeptide chains.
It shows the folding and coiling of the
4. Ester
polypeptide chains and how they give
(ii) Alkaline hydrolysis of amide and ester
-helix or -pleated sheets. The structures
functional groups occur when aspartame
are stabilised by hydrogen bonds between
is treated with hot NaOH(aq).
the N–H group of one amino acid unit and
Three other compounds are:
the C=O group of another amino acid unit
1. CH3OH
along the main chain.

Answers to A Level H2 Topical Chemistry A79


© Singapore Asia Publishers Pte Ltd Topic 10.7

H2 Topical Chemistry 2014.indb 79 3/27/2014 1:57:49 PM


Tertiary structure refers to the overall 3-
dimensional shape of the protein involving Tips
folding or coiling of the chains. It shows how Larger the Kb value, stronger the base.
protein molecules are arranged in relation
to each other. The four types of interactions (b) C2H5NH2 + H2O  C2H5NH3+ + OH–
involved are: [C2H5NH3+][OH–]
_______________
 Hydrogen bonds between polar R- ∴ Kb = ​  [C H NH  
  ] ​
groups 2 5 2

 Ionic bonds between charged R-groups Since the degree of dissociation is very
 Van der Waals' forces between non-polar small,
R-groups [C2H5NH2]eqm  [C2H5NH2]initial
 Disulfide linkages (covalent bonds) [OH–]2
______
∴ Kb = ​ 0.100 ​  
= 6.4 × 10–4
Quarternary structure refers to the way
in which more than one polypeptide ∴ [OH–]2 = (0.100)(6.4 × 10–4)
chains in protein group together to form [OH–] = 0.00800 mol dm–3
a stable protein molecule. Haemoglobin's pOH = –log10[OH–]
quarternary structure is made up of 4 = –log10(0.00800)
polypeptide chains bonded by attraction of = 2.10
side chains of the polypeptides. The type of
∴ pH = 14 – pOH
bonding between the chains are the same as
= 14 – 2.10
in tertiary structure.
= 11.9
(b) (i) H O
(c) • C2H5NH2 + (CH3)2CHCOCl
– N – CH – C –
Products:
CH2 O
CH2 (CH3)2CHC – N – C2H5 + HCl
C=O H
O – • C6H5NH2 + Br2(aq)
(ii) Weak van der Waals' forces between Products:
the non-polar side chain of valine NH2
residue causes the aggregation of Hb–S Br Br
molecules. + 3HBr
(iii) Molecules of normal haemoglobin do
not attract each other due to the likely Br
repulsion between the negatively
charged glutamic acid residues. 6. �����������������������������������������������
(a) (i) A weak acid , HA, dissociates as shown
below:
5. ���������������������
(a) Strength of base: HA  H+ + A–
C2H5NH2 > NH3 > C6H5NH2 Acid dissociation constant, Ka, is shown
Ethylamine, C2H5NH2 is a stronger base than below:
NH3 because the electron-donating ethyl [H+][A–]
_______
group increases the electron density on the Ka = ​  [HA] ​  
and pKa = –log10Ka.
N atom, making the lone pair of electrons The smaller the pKa value, the stronger
on the N atom to be more available to accept is the acid.
a H+. (ii) • pH 1  + NH​3 ​ 
Phenylamine, C6H5NH2 is a weaker base than
NH3 because the lone pair of electrons on HO2C CO2H
the N atom is delocalised into the benzene
ring, and this makes it less available to accept
H+.

Answers to A Level H2 Topical Chemistry A80


© Singapore Asia Publishers Pte Ltd Topic 10.7

H2 Topical Chemistry 2014.indb 80 3/27/2014 1:57:49 PM


• pH 3 The quaternary structure of proteins consists
NH​ +3 ​ 
of more than one polypeptide chain coming
– together to form the complete protein
HO2C CO2
maintained by the same forces which are
• pH 7 responsible for tertiary structure.
NH​ +3 ​  (b) (i) Heat under reflux with aqueous NaOH
or dilute H2SO4.
CO2 ​  ​ 


O2C
(ii) met-asp-gly-ser-ala-gly-glu-ser-lys-
• pH 11 tyr
NH2
8. ����
(a) Step I:

O2C CO2

Reagents: Sn, excess conc. HCl(aq)
Conditions: Reflux, follow by addition of
(b) (i) T h e t e r t i a r y a n d q u a r t e r n a r y excess NaOH(aq).
structures of proteins are altered Step II:
during coagulation as the side-chain Reagents: Excess NaOH(aq)
R group interactions are broken. The Conditions: Room temperature
hydrophobic proteins aggregate and
Step IV:
form bonds between them.
Reagents: CH3COCl
(ii) Ca2+(aq) ions will form ionic interaction Conditions: Room temperature
with the negatively charged R group (b) Step I : Reduction
of the side chains, and so breaks up Step II : Neutralisation / Deprotonation /
the ionic interaction between the side Acid-Base Rxn
chains. Step III : N u c l e o p h i l i c s u b s t i t u t i o n
e.g. 2 –CO2– + Ca2+ → –( CO2–)2Ca2+ (Bimolecular)
Step IV : Nucleophilic substitution /
(iii) Weak acids will dissociate partially to
Condensation
form H+ ions which will protonate the (c) To make a stronger nucleophile.
ionic R groups and so disrupts the polar
ionic bonds between the side chains. 9. (a) Amides are slowly hydrolysed in the
laboratory by heating under reflux with
e.g. –CO2– + H+ → –CO2H
aqueous alkali or acid.
7. ��������������������������������������������������
(a) The secondary structure of proteins refers to Acid Hydrolysis:
the detailed arrangement of the polypeptide Condition: reflux with dilute H2SO4 (or dilute
chains. It shows how the chains may be HCl)
coiled or folded to give the α-helix or β- Alkaline Hydrolysis:
pleated sheets. The structures are stabilised Condition: reflux with aqueous NaOH
by hydrogen bonds between the N–H group (b) Using dilute HCl in Acid Hydrolysis:
of one amino acid unit and the C=O group
Products of hydrolysis of phenybutazone:
of another along the main chain. OH OH
The tertiary structure refers to the overall O O &
3-D shape of the protein involving folding –
Cl+H2N — N​H​+2​ ​Cl–
or coiling of the chains. It shows how protein
molecules are arranged in relation to each
other. The four types of R-group interactions Products of hydrolysis of phenobarbital:
O
that hold the tertiary structure in its necessary
OH
shape are:  Hydrogen-bonding between & NH3, CO2 & H2O
polar R-groups,  Ionic bonds between OH
charged R-groups,  Hydrophobic van der O
Waals’ forces between non-polar R-groups
and  Disulfide linkages.

Answers to A Level H2 Topical Chemistry A81


© Singapore Asia Publishers Pte Ltd Topic 10.7

H2 Topical Chemistry 2014.indb 81 3/27/2014 1:57:50 PM


10. (a) •  D–: –7.5o (ii) Melting involves the breaking up of the
•  T+: +5.0o lattice structure of a solid. Hence, the
•  He2+: +7.5o melting point of an element indicates the
strength of forces holding the particles
Tips
(i.e. atoms, ions or molecules) together
Charge
______ of in the crystal lattice.
Particle Charge Mass ​  Mass ​ 
Deflection Na is a metal and its melting point is
__
1
H+ 1+ 1 ​ 1 ​  +15o determined by the amount of energy
__
1 required to overcome the metallic
D– 1– 2 ​ 2 ​  –7.5o
bonding between the ‘sea of delocalised
__
1
T+ 1+ 3 ​ 3 ​  +5o valence electrons’ and the positive Na+
__
2 __ 1 ions.
He 2+
2+ 4 ​ 4 ​  = ​ 2 ​  +7.5o
Si has a very high melting point because
it exists as a giant covalent structure. A
(b) (i) Overall charge of particle R = 4+ large amount of energy is required to
break the extensive network of strong
Tips
covalent bonds in Si.
Charge
______ of P and Cl have lower melting points
Particle Charge Mass ​  Mass ​ 
Deflection since they are simple covalent molecules
__
1
H+ 1+ 1 ​ 1 ​  15o that are held together by weak van der
R 4+ 12
___4 __ 1
​ 12  ​ = ​ 3 ​  5o
Waals’ intermolecular forces. Cl would
have a lower melting point than P since
Cl2 is much smaller than P4. Strength
(ii) No. of neutrons = 6 of van der Waals' forces depends on
No. of electrons = 2 the number of electrons in the molecule
Tips (which increases with increasing
Mass = Nucleon No. molecular mass).
= No. of Protons + No. of Neutrons (d) (i)
12 = 6 + No. of Neutrons O O O O
∴ No. of Neutrons = 12 – 6 H2N – CH2 – C – NH – CH – C – NH – CH – C – NH – CH2 – C – OH
=6 (CH2)3 CH2
Since particle R has charge of 4+, it must have given out 4
electrons. Therefore 6 – 4 = 2 inside particle R. NH C=O
C = NH  O
(c) (i) Na, Si, P and Cl are elements in the  NH3 Ionic
Bonds
@ pH = 7
Period 3 of The Periodic Table.  NH3
Anions (P3– & Cl–) are bigger than  O C = NH
cations (Na+ & Si4+) because anions have
C=O NH
one more shell of electrons than cations
and so outer electrons are less strongly O O CH2 O (CH2)3 O
attracted by the positive nucleus. HO – C – CH2 – NH – C – CH – NH – C – CH – NH – C – CH2 – NH2
Between the two cations, Si4+ has a Ionic Bonding (electrostatic attractions
smaller ionic radius than Na+. This between charged –NH3 groups and
is because Si4+ has more protons in –COO groups) between aspartic acid
the nucleus and the same number of residues on one chain and arginine
electrons are attracted more strongly residues on another chain.
by the increasing nuclear charge. O O O O
Similarly, Cl­– has a smaller ionic radius  H3N – CH2 – C – NH – CH – C – NH – CH – C – NH – CH2 – C – O 
than P3– for the same reason. O O O O
Ionic
Bond

Tips  O – C – CH2 – NH – C – CH – NH – C – (CH2)3 – NH – C – CH2 – NH3 


Na to Si4+ and P3– to Cl– are isoelectronic.
+
CH2 NH
C=O C = NH
@ pH = 7
OH NH3

Answers to A Level H2 Topical Chemistry A82


© Singapore Asia Publishers Pte Ltd Topic 10.7

H2 Topical Chemistry 2014.indb 82 3/27/2014 1:57:50 PM


Ionic Bonding (electrostatic attractions
between charged –NH3 group and
–COO group) between the C-terminal
end of one chain and the N-terminal end
of another chain.
(ii) ① Hydrophobic (or van der Waals'
attractions between non-polar
groups) eg. alkyl or aryl groups
② Disulfide linkages e.g. –SH or
–CH2–S–S–CH2–
O

(e) (i) Ester –C–O– linkages
(ii) H O H O
H 2N – C – C – O – C – C – O – H
H3C – C – H H – C – CH3
CH3 CH3

H O H O
H 2N – C – C – O – C – C – O – H
H3C – C – CH3 CH3
H
Tips
Note that enzyme peptidase are used and they are specific
in hydrolysing the amide linkages (i.e. peptide bonds).
Ester linkages are not hydrolysed by peptidase.

Answers to A Level H2 Topical Chemistry A83


© Singapore Asia Publishers Pte Ltd Topic 10.7

H2 Topical Chemistry 2014.indb 83 3/27/2014 1:57:50 PM


@ High temperatures:
H2 CHEMISTRY EXAMINATION – there are negligible intermolecular attractions
October 2013 since the gas particles have sufficient kinetic
energy to overcome them.
PAPER 1
Section A 7. A [Chemical Energetics]
1. B [Atoms, Molecules and Stoichiometry] ∆Hrxn = ∆Hf(products) – ∆Hf(reactants) as per
2H3PO4 → H4P2O7 + H2O equation.
3H3PO4 → H5P3O10 + 2H2O 8. B [Chemical Energetics]
2. B [Atomic Structure] Since reaction is vigorous, the reaction is
spontaneous.
Particle Protons Electrons Neutrons
∴ ∆G < O.
D3O+ 8 + 3 = 11 7 + 3 = 10 3 + 8 = 11 Temperature decreases from 25ºC to 0ºC.
H3O+ 8 + 3 = 11 7 + 3 = 10 8 ∴ ∆H > O
∆G = ∆H – T∆S
N​H​ ​​ 

2
7+2=9 8 + 2 = 10 7 –ve +ve (+ve)
OD– 8+1=9 8 + 2 = 10 8+1=9 ∴ ∆S must be +ve in order for ∆G to be –ve.
9. D [Electrochemistry]
Tips
Au3+ + 3e– → Au
​ ​H
1
​  has no neutron. ____
6
1
Mole of Au = ​ 197   ​ mol
____
6
3. D [Atomic Structure] Mole of electron, ne = (3 × ​ 197   ​ 
) mol
C atom : 1s22s22p2 Q = it = ne × F
C+ ion : 1s22s22p1
C+ ion (on gaining 1e–) : 1s22s22p2 ne × F
_____
∴ t = ​   ​   
N atom : 1s22s22p3 i
N atom (on gaining 1 e–) : 1s22s22p4 ____
6
(3 × ​ 197   ​ 
)(9.65 × 104)
__________________
Si atom : 1s22s22p63s23p2 = ​  0.10 ​    
Si– ion : 1s22s22p63s23p3
Si­– ion (on gaining l e–) : 1s22s22p63s23p4 = 8.8 × 10 s 4

P atom : 1s22s22p63s23p3 10. C [Ionic Equilibria]


P+ ion : 1s22s22p63s23p2 Ionic Product = (1 × 10–9)(0.1)
P+ ion (on gaining l e–) : 1s22s22p63s23p3
= 1 × 10–10 mol2 dm–6
4. C [Atomic Structure] For CaCO3 : Ionic Product < Ksp ∴ No PPT
There is a huge jump of ionisation energies For FeCO3 : Ionic Product > Ksp ∴ PPT occurs
from 3313 to 7863 kJmol–1. Therefore, M lies in For MnCO3 : Ionic Product > Ksp ∴ PPT occurs
Group IV and the chloride formed would have
11. D [Ionic Equilibria]
the formula of MCl4.
A : Incorrect as [H+] = [OH–] at all temperatures.
5. A [Chemical Bonding] B : Incorrect as temperature increases, the forward
Statements (B), (C) and (D) are all correct. reaction is favoured. Thus at 0ºC, equilibrium
For statement (A), giant covalent structure lies furthest to the left.
does not conduct electricity in any state as its C : Incorrect as temperature increases, the
electrons are all used up for bonding (except equilibrium is observed to shift to the right
graphite) i.e. diamond. (i.e. Kw value increases). Therefore, forward
reaction must be endothermic since increase in
6. C [The Gaseous State]
temperature tends to favour the endothermic
@ Low pressure:
reaction.
– gas molecules are widely spaced and so, have
negligible size (since volume occupied by the D: Temp./ºC Kw pH
gas is very large compared to the gas molecules 0 1.15 × 10–15 7.47
themselves). 25 1.00 × 10–14 7
– forces of attraction between gas molecules are
50 5.50 × 10–14
6.63
also virtually zero.

Answers to A Level H2 Topical Chemistry A84


© Singapore Asia Publishers Pte Ltd Year 2013

H2 Topical Chemistry 2014.indb 84 3/27/2014 1:57:51 PM


14. D [An Introduction to the Chemistry of Transition
Tips Element]
e.g. Kw = 1.15 × 10–15 Mg < Al : Al has more valence electrons per
[H+]2 = 1.15 × 10–15
__________ metal atom.
[H+] = √
​ 1.15 × 10–15 ​ 
Ca < Cu : Cu is a transition element and has
∴ pH = –1g[H+] = 7.47 electrons in its d-orbitals that can conduct
electricity.
12. B [Reaction Kinetics] 15. B [Electrochemistry]
A : CO2 does not cause the formation of acid rain Initially, there is only Fe2+ ions in the conical
since it dissolves in rain water to form carbonic flask, which has a pale green colour.
acid, H2CO3, which is a weak acid. Upon addition of KMnO4, the Mn​O​–4​​  ions are
B: The main cause of acid rain is sulfur dioxide, being reduced to Mn2+, which has a pale pink
SO2. It is oxidised to give sulfur trioxide, SO3, colour.
which reacts with rainwater to form dilute When KMnO4 is added in excess, the excess
sulfuric acid, H2SO4, an acid rain. Mn​O​–4​​  ions present will cause the solution to
SO3(g) + H2O(l) → H2SO4(aq) have a purple colour.

Nitrogen dioxide acts as a catalyst in the 16. D [Halogen Derivatives]


conversion of sulfur dioxide to sulfur trioxide. When CH2ClCHICO2H is heated with sodium:
The nitrogen dioxide first oxidises sulfur – NaI and NaCl are produced.
dioxide to sulfur trioxide. In the process, When NaI and NaCl are dissolved in HNO3 and
the nitrogen dioxide is reduced to nitrogen AgNO3 is added:
monoxide. – White PPT of AgCl and yellow PPT of AgI are
produced.
SO2(g) + NO2(g) → SO3(g) + NO(g)
When concentrated NH3 is added in excess:
The nitrogen monoxide is then converted back – White Agcl PPT readily dissolves in NH3(aq)
to nitrogen dioxide again by reacting with to give a colorless solution.
oxygen. AgCl(s) + 2NH3(aq) → [Ag(NH3)2]+(aq) + Cl­­–(aq)
__
1
NO(g) + ​ 2 ​  O2(g) → NO2(g) – Yellow AgI PPT is insoluble in NH3(aq).
So the nitrogen dioxide is regenerated at the ∴ The precipitate will appear more yellow.
end of the reaction, and can go on to do the 17. D [Chemical Bonding]
same thing again and again. S
C : CFCs causes the thinning of ozone layer, not A : H H all electrons are paired.
+ –
NO2. B : Cl – Cl →  Cl +   Cl  all electrons are paired.
D : CO2 and CH4 are the main greenhouse gases, H
+

not NO2. –

C : H N H Cl all electrons are paired.


13. D [The Periodic Table: Chemical Periodicity]
A : Electrical conductivity increases from Na to Al H
as the number of valence electrons contributed D : Copper ion in CuO is Cu2+.
to the delocalised electron cloud increases. It Electronic configuration of Cu2+ : [Ar]3d9
then decreases to Si which is a semi-conductor. ∴ contains a single unpaired electron.
P, S and Cl are non-conductors as there are
18. A [The Periodic Table: Chemical Periodicity]
no mobile electrons in the structures of P4, S8
Element X is Phosphorus.
and Cl2.
B : P4 + 6Cl2 → 4PCl3
B : In general, the 1st ionisation energy increases
PCl3 + Cl2 → PCl5
across the period due to increasing nuclear
C : Phosphorus, P4, is a solid at room temperature.
charge and decreasing atomic radius.
D : P4O6 and P4O10 dissolve readily in water to
C : Melting point increases from Na to Si and then
give acidic solutions.
decreases from Si to Cl.
P4O6(s) + 6H2O(l ) → 4H3PO3(aq)
D : Atomic radius decreases across the period.
phosphorous acid
This is due to increase in the effective nuclear
P4O10(s) + 6H2O(l ) → 4H3PO4(aq)
charge across the period.
phosphoric acid
Answers to A Level H2 Topical Chemistry A85
© Singapore Asia Publishers Pte Ltd Year 2013

H2 Topical Chemistry 2014.indb 85 3/27/2014 1:57:51 PM


A : PCl3 + 3H2O → H3PO3 + 3HCl 22. D [Hydroxy Compounds]
Addition of NaOH(aq) to solution of H3PO3 O O
and HCl does not give a white PPT. H– O – C – C = C – C – O – H fumaric acid
19. C [Organic Chemistry: Introductory Topics] H H
Step 1 H2O(g), Hydration
H2SO4(aq) reaction
O OH H O
H– O – C – C – C – C – O – H intermediate T
* H H
Step 2 hot acidified Oxidation of 2º
KMnO4(aq) alcohol to ketone
*
O O H O
H– O – C – C – C – C – O – H oxaloacetic acid
H
*
* *
23. D [Carboxylic Acid and Derivatives]
C : Aryl halides are relatively inert and difficult
to hydrolyse. Under normal conditions, aryl
halides are not hydrolysed.
* chiral carbons A and D are acyl chlorides and they are more
ester linkage reactive towards hydrolysis as compared to B
No. of chiral centres = 5 which is an alkyl halide.
____
1
No. of ester linkages = 2 Mole of Cl – from hydrolysis of A = ​ 78.5   ​ 
= 0.0127 mol
20. C [Organic Chemistry: Introductory Topics]
– ____
1
H H H H Mole of Cl  from hydrolysis of D = ​ 155   ​ × 2
w = 0.0129 mol
x C y C z C C H
24. B [Halogen Derivatives]
all carbon atoms
in benzene ring
all the 4 carbon atoms Order of increasing Mr :
are sp2. product Q < product P < product R
are sp2.
Therefore, w, x, y and z bonds are all made up H Br H H OH H
NaOH(aq)
of an sp2 – sp2 overlap. H– C – C – C – H H– C – C – C – H
nucleophilic
H H H substitution reaction H H H
21. C [Hydrocarbons] product P
NaCN(alc.) (Mr = 60)
H H H H H H NaOH(alc.)
δ+ δ– nucleophilic
H C C C H+H I H C C C H
Markovnikov's elimination
Rule substitution reaction
H H I H applies reaction
H CN H H H H
H H H H H H H– C – C – C – H H– C – C C–H
δ+ δ–
H C C C H + I Cl H C C C H H H H H
product R product Q
H H Cl H (Mr = 69) (Mr = 42)

Tips 25. B [Hydroxy Compounds]


Since alcohol X produces a pale yellow PPT
Cl is more electronegative than I.
with alkaline aqueous iodine, it must have a
CH3
– C – OH structure.
H
Only 1º and 2º alcohols can be oxidised by
acidified K2Cr2O7(aq).

Answers to A Level H2 Topical Chemistry A86


© Singapore Asia Publishers Pte Ltd Year 2013

H2 Topical Chemistry 2014.indb 86 3/27/2014 1:57:52 PM


∴ Alcohol X is butan–2–ol with the structure: C
α-helix
OH H H Hydrogen
O
H
CH3 – C – C – C – H bonding N
H H H
Proline either breaks or kinks a helix, both
26. B [Hydroxy Compounds]
because it cannot donate an amide hydrogen
A : Incorrect. There is no amide linkage.
bond (having no amide hydrogen), and also
B : Correct. The 2 phenol groups will react with because the side chain interferes sterically with
NaOH(aq) to form C12H17NO3Na2. the backbone of the preceding turn, inside a
C : Incorrect. The molecular formula of terbutaline helix, which forces a bend of about 30º in the
should be C12H19NO3. helix axis.
D : Incorrect. The molecule contains 7 lone pairs
31. A [Hydrocarbons]
of electrons.
1 : Each molecule of diallyl sulfide contains 6
27. C [Carboxylic Acid and Derivatives] carbon atoms. Upon complete combustion,
A : Incorrect. Both the –OH and the –CO2H we would expect 1 mole of diallyl sulfide to
groups react with PCl5(s). Therefore, it produce 6 moles of CO2.
should be 4 mol of PCl5(s). ____
0.10
Mole of diallyl sulfide = ​ 114 ​ 
B : Incorrect. Only the –OH group can react with
= 8.7719 × 10–4 mol
HCl(g). –CO2H groups do not react with
____
0.23
HCl(g). Mole of CO2 = ​  44 ​ 
C : Correct. Both –OH and the –CO2H groups are = 5.2273 × 10–3 mol
acidic enough to react with Na(s).
Mole of CO2
___________________ ___________
5.2273 × 10–3
D : Incorrect. Only the –CO2H groups are acidic ​        ​ = ​     ​
Mole of diallyl sulfide 8.7719 × 10–4
enough to react with NaOH(aq). –OH group __
6
does not react with NaOH(aq). ≈ ​ 1 ​ 
2 : Each molecule of diallyl sulfide contains 1
28. C [Nitrogen Compounds]
sulfur atom. Upon complete combustion,
2 R–S–H → R–S–S–R + 2[H]
we would expect 1 mole of diallyl sulfide to
Since hydrogen is given out, it is an oxidation produce 1 mole of SO2.
reaction. _____
21
Mole of SO2 = ​ 1000   ​ ÷ 24
29. A [Carboxylic Acid and Derivatives] = 8.75 × 10–4 mol
Acyl chlorides react with amines to form amide
Mole of SO2
___________________ ___________
8.75 × 10–4 __
1
linkages through condensation reaction. ​        ​= ​      
 ​≈ ​   ​ 
Mole of diallyl sulfide 8.7719 × 10–4 1
N –  H  +  Cl OC(CH2)9CH3 3 : Each molecule of diallyl sulfide contains
2 × C = C bonds. Upon addition of excess
bromine, 1 mole of diallyl sulfide will react
O with 2 moles of Br2 to form a product with
N – C – (CH2)9CH3 + HCl Mr = 114 + 4(79.9)
= 433.6
30. D [Nitrogen Compounds] Mass of product = Mole × Mr
The α-helix is a common secondary structure = (8.7719 × 10–4)(433.6)
of proteins and is formed and stabilised by the = 0.380 g
interactions of amino acids that are fairly close 32. D [Reaction Kinetics]
to one another on the polypeptide chain, through Only 1 is correct. Homogenous catalysis refers
hydrogen bonds between the C = O group of one to catalyst and reactants in the same phase or
peptide and N-H group of another peptide. physical state.

Answers to A Level H2 Topical Chemistry A87


© Singapore Asia Publishers Pte Ltd Year 2013

H2 Topical Chemistry 2014.indb 87 3/27/2014 1:57:53 PM


33. A [Electrochemistry] 3 : Indium is not a transition metal. Therefore,
2 : Standard electrode potentials are measured its ionic salts are not expected to be highly
under standard conditions of 298k, 1 atm coloured.
pressure and 1 moldm–3 concentation
37. A [Hydroxy Compounds]
1 and 3 : Standard electrode potentials are
1 : As temperature increases and with concentrated
measured by comparing with the
H 2 SO 4 (aq), elimination of H 2 O from
standard hydrogen electrode which
CH 3 CH 2 CH 2 OH to form CH 3 CH=CH 2
consists of a platinised platinum
occurs.
electrode in a solution of 1.00 mol dm–3
2 : HBr reactant would react with CH3CH=CH2 to
H+ ions and H2(g) at 1 atm is bubbled
form CH3CH(Br)CH3, based on Markovnikov's
over the Pt electrode.
Rule in Electrophilic Addition.
34. B [Group II] 3 : When NaBr is heated with con��·H2SO4(aq), the
O·S· : +2 – 1 +2 – 2 0 following inorganic reactions occur:�
1 : 1
MO2  →  MO  +  __ ​   ​ O2
2
NaBr + H2SO4 → HBr + NaHSO4
Disproportionation of oxygen occurs. The 2HBr + H2SO4 → Br2 + SO2 + 2H2O
oxidation state of oxygen increases from –1
in MO2 to 0 in O2, and decreases from –1 in Common Mistakes
MO2 to –2 in MO. Redox reaction occurs. Students tend to focus only on Organic Reactions, and forget
2 : The anion in MO2, Group II metal peroxides, is about the Inorganic Reactions they have learned. They need
to be aware that NaBr and H2SO4 are inorganic reagents
the peroxide ion ​O​2–2
​  ​. No of electrons = (2)(8) and thus inorganic reactions are possible.
+ 2 = 18.
|
q+ • q –
______
3 : Lattice Energy α ​ r+ + r– ​  | 38. B [Hydrocarbons]
Since O2– oxide ion is smaller than ​O2– ​ ​  ​peroxide 1 and 2 : Forms only one monochloro-derivative
2
ion, the lattice energy of the oxide, MO is since all the hydrogen atoms on the
greater than that of the corresponding peroxide, carbon atoms are similar.
MO2. H H H
Tips H– C – C – H HH – C – H H
Oxidation state of Group II metal does not change. H– C – C – H H – C —– C —– C – H
35. D [Group II] H H H H
cyclobutane 2, 2-dimethylpropane
1 : As number of electronic shells increases, the
atomic and ionic radius increases down the 3 : 2-methylpropane can form 2 monochloro-
group. derivatives since it has 2 different types of
2 : Magnitude of hydration energy of M2+ ion hydrogen atoms.
decreases down the group as the charge density H  –  C  –  H
decreases, reducing the attraction between the H H
ions and the water molecules. H  —–  C  —–  C  —–  C  —–  H
3 : As the number of electronic shells increases,
the atomic radius increases down the group, the H H H
valence electrons are further from the nucleus 39. C [Organic Chemistry: Introductory Topics]
and hence less tightly held to the nucleus, and 1 : Not possible. The C ≡ N group would cause
less energy is required to remove the valence a loss of hydrogen atoms that could be
electrons. attached.
36. B [The Periodic Table: Chemical Periodicity] 2 : Possible Structure :
O H H H
1: Indium is in Group III of the Periodic Table, H
just like aluminium. Indium chloride will HO – C – C – C – C – N
H
exists as the dimer. In2Cl6, just like Al2Cl6, in H H H
the vapour phase. 3 : Possible Structure :
2 : Since Al2O3 is amphoteric, we would expect H O H H
H
In2O3 to be also amphoteric, which dissolves H–C–C–O–C–C–N
in aqueous acid. H
H H H

Answers to A Level H2 Topical Chemistry A88


© Singapore Asia Publishers Pte Ltd Year 2013

H2 Topical Chemistry 2014.indb 88 3/27/2014 1:57:53 PM


40. A [Carboxylic Acid and Derivatives] Table 1
1 : The reaction is known as condensation Mass of weighing bottle
reaction since a small molecule such as H2O CuSO4·5H2O/g
is released as a by-product. Mass of empty weighing
In terms of mechanism, this is a nucleophilic bottle/g
addition-elimation reaction. The NHNH2 Mass of CuSO4·5H2O/g
first adds across the carbon-oxygen bond
② Carefully transfer the weighed solid into
(the addition stage) to give an intermediate
a clean and dry 100 ml beaker.
compound which then loses a molecule of
③ Rinse the weighing bottle with deionised
H2O (the elimation stage).
coater and pour the washings into the
2 : This is a nucleophilic addition reaction. The beaker. Rinse a second time.
carbon-oxygen double bond is highly polar, and ④ Add some more deionised water to the
the slightly positive carbon atom is attacked beaker and stir well to dissolve all the
by the cyanide ion acting as a nucleophile. solids, using a glass rod.
δ+ δ– ⑤ Transfer the solution into a 250 ml
C
O
standard volumetric flask by pouring it
CN–
down a glass rod.
3 : Nucleophilic addition-elimination reaction ⑥ Rinse the beaker with deionised water
occurs between acyl chlorides and ammonia. and transfer the washings to the standard
Tips flask. Rinse a second time.
Mechanism of the nucleophilic addition-elimation reaction ⑦ Finally, rinse the glass rod and add all
between acyl chlorides and ammonia is not required in the washing into the standard flask.
cambridge A-Level H2 Chemistry Syllabus. ⑧ Add deionised water from wash bottle
until near the 250 ml mark.
PAPER 2 ⑨ Make up to the mark with deionised
1. [Planning/ Atoms, Molecules & Stoichiometry/ water using a dropper.
An Introduction to the Chemistry of Transition ⑩ Stopper the standard flask securely and
Element] shake well to obtain a homogenous
(a) [Cu(H2O)6]2+ + 4NH3  [Cu(NH3)4(H2O)2]2+ solution.
+ 4H2O Preparation of a series of diluted range of
(b) Colour : Orange standard solutions:
Explanation : Copper(II) sulfate solution ① 5 different solutions (No.1 to 5) with
is deep blue because it absorbs light in different concentrations of CuSO4(aq)
the orange region of the visible spectrum. were prepared, as shown in Table 2.
Deep blue is the complimentary colour of Table 2
orange. Volume
(c) Preparation of 250.0 cm3 of 2.00 mol dm–3 of 2.00 Volume of Concentration
Absorbance/
aqueous copper (II) sulfate stock solution: No. mol dm–3 deionised of CuSO4(aq)/
A
CuSO4(aq)/ water/cm3 mol dm–3
Mole of CuSO4·5H2O cm 3
_____
250
= ​ 1000  ​ dm3 × 2.00 mol dm–3 1 0 20 0
= 0.5 mol 2 5 15 0.5
Mr of CuSO4·5H2O = 249.6 3 10 10 1.0
∴ Mass of solid CuSO4·5H2O needed 4 15 5 1.5
= 249.6 × 0.5 5 20 0 2.0
= 124.8 g ② To prepare 0.5 mol dm–3 CuSO4(aq)
① Use a clean dry weighing bottle to (No. 2), add 5.00 cm3 of 2.00 mol dm–3
weigh out about 124.8 g of solid CuSO4(aq) stock solution into a 250 cm3
CuSO4·5H2O using the mass balance. volumetric flask using a burette.
(Do not try to get the exact mass. Instead, ③ Using a separate burette, add 15.00 cm3
settle for any mass within 0.1g of of deionised water into the volumetric
124.8 g and record the actual mass flask. Swirl to get a homogenous
obtained). solution.

Answers to A Level H2 Topical Chemistry A89


© Singapore Asia Publishers Pte Ltd Year 2013

H2 Topical Chemistry 2014.indb 89 3/27/2014 1:57:54 PM


④ Repeat the above steps to obtain diluted ③ Run solution X in the spectrometer and
solutions of different concentrations record the absorbance value, Ax.
using the volumes stated in Table 2. ④ Using the graph drawn earlier, a
Preparation of Copper-Ammonia Complex horizontal line is drawn from this value to
for each solution: intersect the calibration line. By drawing
① Using a measuring cylinder, add 100 cm3 a vertical line from the intersection point,
of 2.00 mol dm–3 NH3(aq) to each flask we will find out the concentration of
and swirl to get a homogenous solution. copper(II) ions in solution X.
Copper-ammonia complex of different 2. [Chemical Energetics/Chemical Equilibrium/
intensity of deep blue colour is observed Electrochemistry]
for solutions 2 to 5. (a) (i) Reaction (1) is responsible for raising
② Overall equation for complex formation the temperature of the furnace as it is
of copper(II) ions with water and the most exothermic reaction i.e. gives
ammonia is: out the most heat.
[Cu(H2O)6]2+ + 4NH3  [Cu(NH3)4(H2O)2]2+ + 2OH– (ii) There is an increase in the disorderliness
deep blue solution in the system for reactions (2) and (4)
Adding of 100 cm3 of NH3(aq) to each because there is an increase of 1 mole
flask ensures that NH3 is always in of gaseous products. There is no change
excess. E.g. for 20 cm3 of 2.00 mol dm–3 in the no. of moles of gaseous products
CuSO4(aq) solution, a minimum volume in other reactions.
of 80 cm3 of 2.00 mol dm–3 NH3(aq) (iii) ∆G = ∆H – T∆S
is required to form copper-ammonia
complex, since [Cu(H2O)6]2+ : NH3 is
= (–24.8) – (500 + 273)​ +​ 1000  ( 
_____
15.8
)
 ​  ​
= –37.0 kJ mol –1

1 : 4. Increasing the temperature has no effect


Using the spectrometer: on the spontaneity of this reaction since
① Spectrometer is set up to absorb the ∆G < O at all temperatures (because ∆H
wavelength of orange. < O and ∆S > 0).
② Each of the five copper-ammonia (iv) Le Chatelier's Principle states that if
complex solutions are run by the a system of a dynamic equilibrium is
spectrometer and the absorbance values subject to a change which disturbs the
are recorded in Table 2. equilirbrium, the system will respond
③ Plot a graph of absorbance value in such a way so as to counteract the
versus the concentration of CuSO4(aq) effect of the change.
solution. An increase in temperature causes
④ Draw the best fit linear line going through the position of equilibrium to shift
the origin. This is the calibration line. to the right, favouring the forward
Absorbance/A endothermic reaction, to absorb the
excess heat.
AX
An increase in pressure causes the
position of equilibrium to shift to the
left which has fewer number of moles
of gaseous molecules, reducing the
pressure in the system.
Conc. of
CuSO4(aq)/ (v) CaCO3(s) → CaO(s) + CO2(g)
0 0.5 1.0 1.5 2.0 mol dm–3 CaO(s) + SiO2(s) → CaSiO3(s)
Analysing solution X: Mass of silicon dioxide from 1 tonne
① Using a burette, measure 20 cm3 of ____
3.5
of ore = ​ 100  ​ × (1 × 106 g)
solution X into a 250 cm3 volumetric
flask. = 35000 g
② Using a measuring cylinder, add 100 cm3 ____________
35000
Mole of SiO2 = ​    
   ​
of 2.00 mol dm–3 NH3(aq) into the flask. 28.1 + 2(16.0)
Swirl to obtain a homogenous solution. = 582.36 mol

Answers to A Level H2 Topical Chemistry A90


© Singapore Asia Publishers Pte Ltd Year 2013

H2 Topical Chemistry 2014.indb 90 3/27/2014 1:57:54 PM


Mole of CaCO3 = Mole of CaO 3. [Hydrocarbons/Hydroxy Compounds]
= Mole of SiO2 (a) (i) reagent A : NaOH(aq)
= 582.36 mol reagent B : (CH3)2CHCOCl
∴ Mass of CaCO3
= Mole × Mr of CaCO3 Common Mistakes
= (582.36) × [40.1 + 12.0 + 3(16.0)] R – COCl + HO → R – COO + HCl
= 58,294 R – COOH + HO → No Reaction
≈ 5.83 × 104 g
(b) (i) 2Fe3+ + Sn2+ → 2Fe2+ + Sn4+ (ii) OH
O 2N OCH3
(ii) Fe3+ + e–  Fe2+ Eθ = +0.77V [R]
Sn + 2e  Sn Eθ = +0.15V [O]
4+ – 2+

∴ ​Eθcell
​  ​ =
​  ​Eθred
​  ​​ – ​EθOX
​  ​  R
= (+0.77) – (+0.15)
Tips
= +0.62V
The –OH group in phenol activates the ring towards
(iii) Fe2+ + 2e–  Fe Eθ = –0.44V [R]
electrophilic substitution and is 2–, 4–directing.
Sn + 2e  Sn Eθ = +0.15V [O]
4+ – 2+
Ignore the effects of ether group (–OCH3) since it is not in
∴ ​Eθcell
​  ​ = ​Eθred
​  ​​ – ​EθOX
​  ​ 
H2 Chemistry syllabus.
= (–0.44) – (+0.15)
= –0.59V (iii) Observations:
Since ​Eθcell​  ​  < 0, ∴ the reaction is not ① Reddish-brown bromine solution
feasible. is decolourised
____________
21.40 + 21.50 ② Steamy white fumes of HBr is
(c) (i) Average titre volume = ​  2 ​
    
= 21.45 cm 3 liberated
(iv)
∴ No. of moles of K2Cr2O7 OH OH
_____
21.45 OCH3 Br OCH3
= ​ 1000 ​ × 0.100
+  Br2 +  HBr
= 2.145 × 10–3 mol
≈ 2.15 × 10–3 mol R R
(b) (i) Phenols are more acidic due to its
Tips relative stability of the phenoxide ion
Usual recommendation is one rough titre (titration number (its conjugate base) as compared to
1) to see roughly where the end point actually is, which we the ethoxide ion. In the phenoxide ion,
will ignore during the calculation of the average volume. the partial overlap of the p orbital on
the oxygen atom with the  electron
(ii) Cr2​O​2–
7
​  ​+ 14H+ + 6Fe2+ → 6Fe3+ + 2Cr3+ cloud of the benzene ring allows for
+ 7H2O delocalisation of the electron pair
(iii) Since 1 mole of Cr2​O​2– 7
​  ​ reacts with 6 dispersing the negative charge on
moles of Fe2+, the O atom, reducing the tendency
∴ Mole of Fe2+ in 25.0 cm3 of phenoxide to attract protons. On
= (2.145 × 10–3­) × 6 the other hand, the ethoxide ion has
= 0.01287 mol an electron donating alkyl group that
Mole of Fe2+ in 250 cm3 increases the negative charge density,
= 0.01287 × 10 making it more ready to accept protons.
= 0.1287 mol Hence, it is a weaker acid.
(ii) HA  H+ + A–
Mass of Fe in the ore [H+][A–]
_______
= 0.1287 × 55.8 Ka = ​  [HA] ​   
= 7.18146 g __________
[H+] = √​ (Ka)(0.001) ​ 
____________

∴ % by mass of Fe in iron ore
= ​√(10–7.4)(0.001) ​
  
_______
7.18146
= ​  11.15 ​ 
 
× 100% = 6.30957 × 10–6
= 64.4% ∴ pH = –1g[H+] = 5.20

Answers to A Level H2 Topical Chemistry A91


© Singapore Asia Publishers Pte Ltd Year 2013

H2 Topical Chemistry 2014.indb 91 3/27/2014 1:57:54 PM


(c) OH OH (ii) In Fe, both the 3d and 4s orbitals can
OCH3 OCH3 contribute to the delocalised ��������
‘�������
sea of
CH3 H
valence electrons�����������������������
’����������������������
for metallic bonding
whereas Ca only uses the 4s orbitals.
C C C C
Hence, stronger metallic bonds exist
H H H CH3
in Fe and it has a higher melting point
cis isomer trans isomer
than Ca.
4. [An Introduction to the Chemistry of Transition (c) (i) Copper is heavy and may cause the
Element/Electrochemistry/Group II] overhead electrical cables to sag
(a) (i) Ca : [Ar] 4s2 between support over time.
Fe : [Ar] 3d54s2
Tips
Cu : [Ar] 3d104s1
Fe and Cu have more protons than Aluminium is commonly used in overhead electrical cables
due to its light weight and moderate electrical conductivity
Ca, and hence, have a higher nuclear (not as good as copper).
charge than Ca. Fe and Cu also have
more electrons than Ca but the increase (ii) Cu : 1s22s22p63s23p63d104s1
in shielding effect is minimal.
Cu2+ : 1s22s22p63s23p63d9
Although Fe and Cu have additional (iii) Impure copper can be purified through
3d electrons in an inner sub-shell, they electrolysis. The impure copper is
provide relatively poor shielding (when placed at the anode while the pure
compared to the 3s and 3p electrons) copper is placed at the cathode. Both
to the outermost 4s electrons because electrodes are placed into CuSO4(aq).
they are occupying highly diffused d At the anode, copper and impurities
orbitals. (e.g.Zn and Fe) with Eθ values which
Consequently, the valence 4s electrons of are less positive than that of copper
Fe and Cu experience greater effective will dissolve as ions. Impurities (e.g.
nuclear charge and are attracted more Ag and Au) with Eθ values which are
strongly to the nucleus, resulting in Fe more positive than that of copper remain
and Cu having smaller atomic radii than undissolved and drop to the bottom of the
Ca. vessel as ‘��������������
���������������
anode sludge��
’�.
_______
Mass
(ii) Density = ​    ​  At the cathode, copper (II) ions from the
Volume
Fe and Cu have larger atomic masses solution are discharged to form copper
than Ca and have smaller atomic radii metal which attached itself to the pure
than Ca (smaller atomic radii → smaller copper cathode.
volume). Hence, the densities of Fe and (d) (i) 2Ca(NO3)2 → ∆ 2CaO + 4NO2 + O2
Cu are significantly higher than that of charge
______
(ii) Charge Density α ​   ​ 
 
Ca. size
Cu has a comparatively higher charge
2+
(b) (i) Calcium has a giant metallic structure
density than Ca2+ due to its smaller ionic
with strong electrostatic forces of
size. Thus Cu2+ has a greater polarising
attraction between the positively
power to polarise the electron cloud of
charged Ca2+ ions and the sea of
N​O​–3​​  ion. This weakens the N-O bond
delocalised valence electrons.
to a larger extent and thus Cu(NO3)2
e– 2+e e 2+e 2+ e–
– – –
decomposes at a lower temperature.
Ca –Ca –Ca e–
e– 2+e 2+e 2+ e– 5. [Nitrogen Compounds]
e–– Ca Ca Ca e–– (a) ① Structural proteins – provide the
e e
e– Ca Ca Ca2+ e–
2+ 2+
framework which defines the size and
e–
e –
shape of cells.
Notes ② Enzymes – to catalyse metabolic
No. of  Ca2+  :  No. of e– processes which produce energy, build
1  :  2
up new cell structures and destroy old
ones.

Answers to A Level H2 Topical Chemistry A92


© Singapore Asia Publishers Pte Ltd Year 2013

H2 Topical Chemistry 2014.indb 92 3/27/2014 1:57:55 PM


③ Transport proteins – to move metabolites (f) H – O – O – H  →  H – O• + •O – H
around the cell or around the whole Free radical is an atom or group of atoms
organism. having an unpaired electrons produced by
(b) (i) cys – tyr – ile – gln – asn – cys – pro the homolytic fission of a covalent bond.
– leu – gly
Tips PAPER 3
The primary structure of a protein shows the order (or 1. [Group VII/Halogen Derivatives/Chemical
sequence) of amino acids in a protein (or polypeptide Equilibria/Electrochemistry]
chain). (a) The halogens are all oxidising agents and
their oxidising power decreases down the
(ii) group.
This is shown by the Eθ values (from the
Data Booklet) becoming less positive on
descending the group.
F2 + 2e–  2F– Eθ = +2.87V
Cl2 + 2e–  2Cl– Eθ = +1.36V
Br2 + 2e  2Br
– –
Eθ = +1.06V
I2 + 2e  2I
– –
Eθ = +0.54V
(b) (i) 2HX(g) → ∆ H2(g) + X2(g)
(ii) As the size of the halogen increases
down the group, the H–X bond
becomes longer and weakens and so,is
easily broken.
Tips
Thermal stability of the hydrides
Phenol is more acidic than water.
decreases down the group due to the
OH + H2O  O – + H 3O +
decrease in the strength of the H–X bond
as the size of the halogen increases.
(c) Tertiary structure refers to the overall 3- Thermal stability:
dimensional shape of the protein involving HF > HCl > HBr > HI
folding or coiling of the chains. It shows HI decomposes easily while HBr
how protein molecules are arranged in decomposes slightly. HCl and HF do not
relation to each other, due to the side-group decompose. They are stable to heat.
interactions. (c) (i) Bimolecular Nucleophilic Substitution
In oxytocin, the following interactions are (SN2) mechanism
involved in maintaining its 3-D shape: –
① Disulfide linkages between the side H δ+ δ– H H
H
groups of 2 cys amino acid. –
I C – Cl I----C----Cl I–C   +  Cl–
H H
② Hydrophobic interactions between CH2CH3 CH2CH3 CH2CH3
the side groups of tyr and ile amino transition state
acids.
(ii) Halide exchange is an equilibrium
(d) Proteins can be denatured by the addition of
reaction, but the reaction can be driven to
heavy metal ions such as Hg2+. The heavy
completion by exploiting of differential
metal ions would form salts or complex ions,
solubility of the halide salts.
thereby breaking the ionic (electrostatic)
NaI is soluble in propanone while NaCl
interactions. Heavy metal ions also break
is almost insoluble.
the disulfide linkages.
The reaction is driven towards the
(e) (i) Hydrolysis
H2N products (RHS) due to precipitation of
(ii)
H2N HO insoluble salt, NaCl.
and Based on Le Chatelier's Principle, if a
O
HO O O product is removed in an equilibrium,
NH2 a new equilibrium is produced which

Answers to A Level H2 Topical Chemistry A93


© Singapore Asia Publishers Pte Ltd Year 2013

H2 Topical Chemistry 2014.indb 93 3/27/2014 1:57:55 PM


contains a higher proportion of the [​I​–3​]​ 
______
product i.e. position of equilibrium shifts (d) (i) KC = ​ [I ][I  –  ​ 
] units = dm mol
3 –1
2
to the right. 2.54 g
______
(ii) Amount of I2 added = ​   
 ​ 
CH3CH2CH2Cl(propanone) CH3CH2CH2I(propanone) 2(127)
+ + = 0.01 mol
NaI(propanone) NaCl(s)

_____
100
Amount of I– added = ​ ​ 1000  ​  ​× 1.00 (  )
Tips
= 0.1 mol
The above reaction is known as Finkelstein Reaction that
involves the exchange of one halogen atom for another. Amount of ​I​–3​​ at equilibrium
The reaction is a continuous one-step process and goes
through SN2 mechanism.

_____
100
(  )
= ​ ​ 1000  ​  ​× 9.98 × 10–2
= 9.98 × 10–3 mol
(iii) H Br H I2(aq) + I–(aq)  ​I​–3​​ (aq)
H–C–C–C–H Initial/mol 0.01 0.1 0
H H H Change/mol –9.98 × 10–3 –9.98 × 10–3 +9.98 × 10–3
Compound A Equilibrium/mol 2.0 × 10–5 0.09002 9.98 × 10–3
• Compound A (a 2º alkyl bromide) ________
2.0 × 10–5
∴ [I2(aq)] = ​   ​ 
 
undergoes nucleophilic substitution
to form compound B (a 2º alcohol).

_____
100
)
​ ​ 1000  ​  ​
= 2.0 × 10–4 mol dm–3
H Br H H OH H
_______
0.09002
[I–(aq)] = ​   ​ 
H–C–C–C–H
H H H
H–C–C–C–H
H H H

_____
100
​ ​ 1000  ​  ​ )
Compound A Compound B = 0.9002 mol dm–3
• Compound B (a 2º alcohol) undergoes = 0.900 (3 s.f.)
oxidation to form compound C (a (9.98 × 10–2)
_________________
KC = ​        ​
ketone) which is neutral. (2.0 × 10–4)(0.9002)
= 554.321 dm mol–1 3
H OH H H O H
≈ 554 dm3 mol–1 (3 s.f.)
H–C–C–C–H H–C–C–C–H
_______
1.00
H H H H H (iii) 554.321 = ​ 1.00[I  ] ​ 
2
Compound B Compound C ∴ [I2(aq)] = 1.804 × 10–3 mol dm–3
• Compound A (a 2º alkyl bromide) ≈ 1.80 × 10–3 mol dm–3 (3 s.f.)
undergoes halide exchange reaction (iv) Since hexane and water are immiscible
with NaI in propanone to form and molecular iodine, I2, is much
compound D (a 2º alkyl iodide) and more soluble in hexane than it is in
NaBr. water, we would expect some of the
molecular iodine to transfer itself from
H Br H H I H
the aqueous layer of ​I​–(aq)   ​ 
/​I–3​ ​(​  aq) to the
H – C – C – C – H (propanone) H – C – C – C – H (propanone)
organic non-polar hexane layer. The
H H H H H H
iodine becomes distributed throughout
Compound A Compound D
(Mr = 123) (Mr = 170)
the mixture of the two layers and reaches
+ + an equilibrium.
NaI(propanone) NaBr(s) As such, the addition of hexane would
________________
Mr of D – Mr of A increase the [I2(hexane)] and decrease
​    
 ​
   × 100% the [I2(aq)] in equilibrium 2. By Le
Mr of A
_________
170 – 123 Chateliers' Principle, a decrease in the
= ​  123 ​   
× 100% [I2(aq)] in the system would cause the
= 38.2% position of equilibrium 2 to shift to the
Hence, Mr of D is 38.2% larger than left hand side so as to replace the I2(aq).
the Mr of A. [​I​–3​(​  aq)] will decrease.

Answers to A Level H2 Topical Chemistry A94


© Singapore Asia Publishers Pte Ltd Year 2013

H2 Topical Chemistry 2014.indb 94 3/27/2014 1:57:56 PM


(iii) Ammonia is acting as an acid.
Tips
Reason: This is a Bronsted-Lowry
Violet Colour Solution: Acid-Base reaction. NH 3
Organic non-polar hexane layer with I2 acts as the acid and donates
i.e. I2(hexane)
a proton to the H – base,
Faint Brown Colour Solution forming the conjugate base
Aqueous layer of I–(aq) & ​I–3​ ​(​  aq) with I2
N​H​–2​​  ion and conjugate acid
i.e. I2(aq)
H2 molecule.
(v) Cu2+ + 2e–  Cu Eθ = +0.34V [O] NH3  +  H–  +  Na+  →  N​H​–2​ ​  +  Na+  +  H2
​I​–3​​ + 2e–  3I– Eθ = +0.536V [R] acid base conjugate conjugate
Overall: I​ ​–3​​ + Cu → 3I– + Cu2+ base acid
​E​θcell  ​ = ​E​   ​ – ​E​   ​ =
θ
Red
θ
Ox
​  (+0.536) – (+0.34) NaH NaNH2
= +0.196V (iv) Ammonia is acting as a reducing
When hexane is added to the ​I​–3​/​ I– half agent.
cell, the position of the equilibrium of​
Reason: It helps to reduce NaOCl
I​–3​​ + 2e–  3I– will shift to the right hand
to NaCl by decreasing the
side to produce more I–.
This causes the Eθ( ​I​–3​/​ I–) to be more oxidation number of chlorine
positive and the overall ​Eθcell ​  ​ value will from +1 in NaOCl to –1 in
also be more positive. NaCl.
(v) Ammonia is acting as a nucleophile.
Tips
Reason: The first stage (the addition
Note that the two equilibriums are related by having the
stage of the reaction) involves
same ions I–(aq) and I​ –3​ ​(​  aq):
I2(aq) + I–(aq)  ​I–3​ ​(​  aq) equilibrium 2
a nucleophilic attack on the
​I​–3​(​  aq) + 2e–  3I–(aq) Eθ = +0.536V fairly positive carbon atom by
the lone pair on the nitrogen
2. [Ionic Equilibria/Nitrogen Compounds] atom in the NH3.
(a) Bronsted-Lowry base is a species which Oδ

accepts protons (H+) from an acid i.e. a δ+

proton acceptor. – C δ–


Cl
Every acid has a conjugate base, this is the
particle left when the acid has given away H N H
its protons.
H
Similarly, every base has a conjugate acid,
this is the particle formed when the base has (c) The strength of a nitrogenous base depends
accepted the proton. on:
A conjugate acid-base is therefore a pair of ① Availability of the lone pair on N atom
species that differs from each other by one to accept a proton, and
proton. ② Stability of its conjugate acid ion.
(b) (i) Ammonia is acting as a base. Order of base strength:
Reason: This is an Bronsted-Lowry ethylamine > ammonia > phenylamine
Acid-Base reaction. H 2 O Ethylamine is a stronger base than
acts as the acid and donates ammonia:
a proton to the NH3 base, CH3CH2NH2 + H2O  CH3CH2N​H​+3​ ​+ OH–
forming the conjugate acid
N​H​+4​ ​ ion and conjugate base NH3 + H2O  N​H​+4​ ​+ OH–
OH– ion. The electron-donating ethyl group in
(ii) Ammonia is acting as a reducing CH3CH2N​H​+3​ ​increases the electron density
agent. on the N atom, making it more available to
Reason: It helps to reduce HNO2 to N2 accept a proton than in ammonia.
by decreasing the oxidation It also stabilises the conjugate acid,
number of nitrogen from +3 CH3CH2N​H​+3​ ​ion, by dispersing the positive
in HNO2 to 0 in N2. charge.

Answers to A Level H2 Topical Chemistry A95


© Singapore Asia Publishers Pte Ltd Year 2013

H2 Topical Chemistry 2014.indb 95 3/27/2014 1:57:56 PM


Phenylamine is a much weaker base than NH2 NH2
ammonia: Br Br
NH3 + H2O  N​H​+4​ ​+ OH–   +  2Br2(aq)   +  2HBr(g)
NH2 NH ​ +3​ ​ ​
CH3 CH3
+ H2O  + OH– white ppt
Tips
The weaker basicity of phenylamine is due
to the delocalisation of the lone pair of Aliphatic amines have no reaction with aqueous bromine.
Aromatic amines undergo electrophilic substitution with
electrons on the nitrogen atom over the
aqueous bromine at room temperature.
aromatic ring, making it less available to
accept a proton. (iii) Route 1:
In addition, the electron-withdrawing CH3 CH3 1. Sn + Excess CH3
conc.HCl
phenyl group also intensifies the positive conc. HNO3 + + reflux
charge on the conjugate acid, C6H5N​H​+3​ ​ion, conc.H2SO4, 30ºC 2. Aq. NaOH
making it less stable than the N​H​+4​ ​ion. NO2 NH2
(d) (i) The aromatic ring (in aromatic amine)
greatly decreases the basicity of the Route 2:
amine and this effect can be either CH3 CH2Cl CH2NH2
excess conc.NH3
strengthened or offset depending on limited Cl2(g) in ethanol
what substituents are on the ring and UV light heat in a sealed tube
on the nitrogen.
4-methylphenylamine is more basic 3. [The Periodic Table: Chemical Periodicity/
than phenylamine: Chemical Bonding/Electrochemistry/Carboxylic
The methyl group substituent on the Acid and Derivatives]
aromatic ring is electron-donating and (a) P4O6(s) + 6H2O(l) → 4H3PO3(aq)
it decreases the electron-withdrawing P4O6 reacts with water to form an acidic
effect of the ring, making the lone pair solution of pH ≈ 3.
of electrons on the nitrogen atom on the Al2O3 is insoluble in water due to its high
aromatic ring to be more available to lattice energy which makes dissolution
accept a proton. difficult
Benzylamine is less basic than (b) Oxide W has a giant ionic structure due to
ethylamine: its high melting point and its good electrical
Both C​H2​–​ ​ and CH3C​H2​–​ ​ groups conductivity when molten. It is a basic oxide
are electron donating groups and that dissolves slightly in water to produce a
both benzylamine and ethylamine are slightly alkaline solution of pH 8, e.g. MgO
stronger bases than ammonia. Oxide X has a giant covalent structure as
However, CH3C​H​2–​ ​ group is more it has a high melting point and does not
electron donating, as the aromatic conduct electricity when molten. It also
ring in the C​H​–2​​  group is slightly has no reaction with water due to its giant
withdrawing. The reduces the network of strong covalent bonds that are
availability of the lone pair of electrons not easily broken, e.g. SiO2.
on the nitrogen atom to be less available Oxide Y has a giant ionic structure due to
to accept a proton. its high melting point and its good electrical
(ii) No reaction between benzylamine conductivity when molten. It is a basic oxide
with bromine water. No visible that dissolves readily in water to produce
observations. a strong alkaline solution of pH 14, e.g.
4-methylphenylamine undergoes Na2O.
electrophilic substitution reaction Oxide Z is a simple covalent compound as
with bromine water to form bromo it has a low melting point due to weak van
substituted product. der Waals������������������������������
’�����������������������������
forces between Z molecules,
Reddish-brown bromine water is and have poor electrical conductivity when
decolourised. A white precipitate is molten. It is an acidic oxide that dissolves
formed. Steamy white funes of HBr is readily in water to produce a strong acidic
liberated. solution of pH 1, e.g. SO2.

Answers to A Level H2 Topical Chemistry A96


© Singapore Asia Publishers Pte Ltd Year 2013

H2 Topical Chemistry 2014.indb 96 3/27/2014 1:57:57 PM


(c) (i)
Tips
SO2 + 2H2O  S​O​24​ ​+ 4H+ + 2e– —– ①

Mn​O​–4​​ + 8H+ + 5e–  Mn2+ + 4H2O —– ② • Compound E is an unsaturated compound with C=C bond
which can undergo electrophilic addition with HCl(g),
(① × 5) + (② × 2) ∴ 2Mn​O​–4​​ + 5SO2 + 2H2O bromine water and H2O.
→ 2Mn2+ + 5S​O​2– ​  ​+ 4H+ • Compound F is an acid chloride which readily hydrolyses
4 in water to form compound G, a carboxylic acid (weak
(ii) The purple KMnO4 solution would acid) with pH of 3.
turn colourless.

_____
20.5

(iii) Mole of Mn​O​–4​​ = ​ ​ 1000  )
 ​  ​× 0.02
4. [Hydrocarbons/Carbonyl Compounds/Chemical
Energetics]
= 4.1 × 10–4 mol (a) Only optical isomerism.

________
4.1 × 10–4
Mole of SO2 = ​ ​  2 ​   ( 
 ​× 5 ) There will be 2 optical isomers.
= 1.025 × 10–3 mol Tips
__________
1.025 × 10–3 Number of optical isomers = 2n = 21 = 2 whereby n is the
∴ [SO2] = ​   ​ 
 
= 0.041 mol dm–3
_____
25
​ ​ 1000  ​  ​(  ) number of chiral carbons (*)

(d) For an acidic buffer: *


[salt]
_____
pH = pKa + lg​    ​ 
[acid]
= –1g(6.3 × 10–5) + 1g​ ​ 0.01 ​  ​
____
0.03
(  ) Common Mistakes
Note that geometric (cis-trans) isomerism cannot exist in
= 4.68 (3 s.f.) cyclohexene structure (present in limonene) due to ring
(e) Ease of hydrolysis: strain. It can only exist in one form. i.e. cis form.
COCl CH2Cl Cl
(b)
> > O
OH , O = C = O and H O H
Cl O
O
Common Mistakes
is an aryl chloride with the p orbitals of
Question asks for “�������������������������������������
��������������������������������������
structural formula of the PRODUCTS”.
the Cl atom overlapping with the  orbitals of Need to include CO2 and H2O.
the benzene ring. This allows the
delocalisation of electrons and strengthens (c) Br
the C–Cl bond.
COCl
Br
is easier to hydrolyse as the carbonyl Tips
C is highly electron deficient as it is bonded Markovnikov’s Rule applies in electrophilic addition.
to two strongly electronegative atoms, O and
Cl, attracting the nucleophiles more strongly, (d)
as compared to that which has only one Observations Deductions
① H and J evolve CO2 • Both H and J have carboxylic acid
strongly electronegative Cl atom. with Na2CO3(aq). group.
Tips • Acid-Base reaction occurs.
② H and J give orange ppt • Both H and J have carbonyl group.
Any hydrolysis reaction which happens involving any of these
with 2, 4–DNPH. • Condensation reaction occurs.
chlorides can be thought of as nucleophilic substitution.
③ H and J do not react with • Both H and J are not aliphatic
H Fehling's solution. aldehydes.
(f) E: C=C=O
H O
H ④ H gives yellow ppt with • H has R – C – CH3 structure and
O
alkaline aqueous iodine hence is a ketone while J does not
F: H – C – C while J does not. O
Cl have R – C – CH3 structure.
H
H ⑤ J is obtained from K • Oxidation of 1º alcohol
O
by reaction with hot → carboxylic acids.
G: H – C – C acidified Cr2​O​2–​  ​ion. • Oxidation of 2º alcohol
Cl 7
→ ketones.
H

Answers to A Level H2 Topical Chemistry A97


© Singapore Asia Publishers Pte Ltd Year 2013

H2 Topical Chemistry 2014.indb 97 3/27/2014 1:57:57 PM


O O OH (ii) Argon exists as a noble gas and its
OH subshells are fully filled and does not
O O form covalent bonds with other argon
J H g-terpinene C10H16
atoms.
Tips (iii) van der Waals����������������������������
’���������������������������
radius is defined as half
• of the distance between the closest
OH O O
hot acidified Cr2​O​2–
7
​  ​ approach of two non-bonded atoms of
OH OH
(Oxidation of alcohols) a given element.
K J Argon has a full electronic configuration
• excess hot O O and their atoms are only attracted due to
OH
conc. Mn​O–4​ ​​  OH + O O van der Waals���������
’ forces.
g-terpinene (oxidative J H For chlorine, each atom has 7 valence
cleavage)
C10H16 electrons and they will be greatly
• g-terpinenene is an isomer of limonene and will have the attracted to come together to form
same molecular formula of C10H16. diatomic molecules.
Therefore we would expect the van der
(e) (i) C10H16 + 14O2 → 10CO2 + 8H2O Waals’ radius of chlorine to be smaller
(ii) Bonds Broken Bonds Formed than argon.
16 × C–H = 6560 20 × C=O = 16100 (b) (i) 2–
8 × C–C = 2800 16 × O–H = 7360 O O
2 × C=C = 1220
(ii) Na2O2 + 2H2O → H2O2 + 2NaOH
14 × O=O = 6944
(c) (i) The order of reaction with respect to a
17524 23460
given reactant is defined as the power to
∴ ∆HC (limonene) = 17524 – 23460 which the concentration of that reactant
= –5936 kJ mol–1 is raised to in the experimentally
Tips determined rate equation.
Structure of limonene (ii) The half-life of a reaction, t__​ 12 ​, is the time
H taken for the initial concentration of a
H H
C reactant to fall to half its value.
(d) (i) See page S16.
C
H C
H
C H (ii) Initial rate = ​  |
______________
0.0200 – 0.0100
190 ​      |
H C H
C = 5.26 × 10 mol dm–3 s–1
–5
H H
C H (iii) Keeping [I –] constant, when [H +]
C C H _____
0.400 __ 4
increases by ​ 0.300 ​ = ​ 3 ​  times, initial
H
C H ________
8.4 × 10–5 __ 4
H H rate increases by ​   ​ 
= ​   ​  times.
6.3 × 10–5 3
___________
–5936 × 1 Hence, reaction is 1st order with respect
(iii) ∆HC (limonene) = ​       ​
(10)(12) + 16 to [H+].
= –43.6 kJg–1 Rate = k[H2O2][H+][I–]x
The enthalpy change of combustion of
k[H2O2][0.300][0.200]x ________
6.3 × 10–5
limonene per gram is less exothermic as ∴ ​ ____________________
    x   ​= ​    ​
compared to that of convertional diesel k[H2O2][0.200][0.100] 2.1 × 10–5
fuel. 2x = 21
∴ x = 1
5. [Chemical Bonding/Reaction Kinetics/Carbonyl Hence, reaction is 1st order with respect
Compounds] to [I–].
(a) (i) Solid silicon lattice consists of (iv) ∴ Rate = ���k[H2O2][H+][I–] mol dm–3 s–1
thousands of silicon atoms held 8.4 x 10 = k(0.0200)(0.400)(0.200)
–5

together by strong covalent bonds to k = 0.0525 mol–2 dm6 s–1


form a giant structure. Covalent radius (e) (i) LiAlH4 in dry ether
of 0.117 nm refers to half the distance (ii) H2SO4(aq), K2Cr2O7(aq).
between the nuclei of silicon atoms in To ensure that the main product is
the covalent bond. propanal, heat and distil immediately.
Answers to A Level H2 Topical Chemistry A98
© Singapore Asia Publishers Pte Ltd Year 2013

H2 Topical Chemistry 2014.indb 98 3/27/2014 1:57:58 PM


Question 5. (d)(i), (ii)

H2O2/mol dm–3

0.0200

0.0180

0.0160

0.0140

0.0120

0.0100

0.0080

0.0060

0.0040

0.0020

0 100 200 300 400 500 600 700 800 time/s

Time taken for 1st half-life = 330s


Time taken for 2nd half-life = 660 – 330 = 330s
Since the half-life for the curve is a constant, the order of reaction with respect to [H2O2] is 1.

Answers to A Level H2 Topical Chemistry A99


© Singapore Asia Publishers Pte Ltd Year 2013

H2 Topical Chemistry 2014.indb 99 3/27/2014 1:57:58 PM

Anda mungkin juga menyukai